Problemas de Eletromagnetismo.pdf

102
Problemas Resolvidos de Eletromagnetismo Prof. Luiz Antˆ onio Righi

Transcript of Problemas de Eletromagnetismo.pdf

Page 1: Problemas de Eletromagnetismo.pdf

Problemas Resolvidos

de Eletromagnetismo

Prof. Luiz Antonio Righi

Page 2: Problemas de Eletromagnetismo.pdf

SUMARIO

1. Sistemas de coordenadas, carga e corrente eletrica . . . . . . . . . . . . . . . . . . . . . . . . 3

2. Lei de Coulomb . . . . . . . . . . . . . . . . . . . . . . . . . . . . . . . . . . . . . . . . . . . . 9

3. Campo e potencial eletrico . . . . . . . . . . . . . . . . . . . . . . . . . . . . . . . . . . . . . 16

4. Lei de Gauss e Capacitancia . . . . . . . . . . . . . . . . . . . . . . . . . . . . . . . . . . . . . 23

5. Campo magnetico . . . . . . . . . . . . . . . . . . . . . . . . . . . . . . . . . . . . . . . . . . 32

6. Fluxo, inducao e forca magnetica . . . . . . . . . . . . . . . . . . . . . . . . . . . . . . . . . . 38

7. Indutancia e circuitos magneticos . . . . . . . . . . . . . . . . . . . . . . . . . . . . . . . . . . 47

8. Fasores . . . . . . . . . . . . . . . . . . . . . . . . . . . . . . . . . . . . . . . . . . . . . . . . 58

9. Campos Dinamicos - Lei de Faraday . . . . . . . . . . . . . . . . . . . . . . . . . . . . . . . . 62

10. Corrente de Deslocamento . . . . . . . . . . . . . . . . . . . . . . . . . . . . . . . . . . . . . . 71

11. Ondas eletromagneticas . . . . . . . . . . . . . . . . . . . . . . . . . . . . . . . . . . . . . . . 79

12. Linhas de Transmissao . . . . . . . . . . . . . . . . . . . . . . . . . . . . . . . . . . . . . . . . 90

13. Carta de Smith . . . . . . . . . . . . . . . . . . . . . . . . . . . . . . . . . . . . . . . . . . . . 95

14. Equacoes de Maxwell . . . . . . . . . . . . . . . . . . . . . . . . . . . . . . . . . . . . . . . . . 100

Page 3: Problemas de Eletromagnetismo.pdf

1. SISTEMAS DE COORDENADAS, CARGA E CORRENTE ELETRICA

PR 1.1: Explicar a diferenca entre submeter um BOM condutor e um BOM isolante (mau condutor)

num campo eletrico.

Resposta: O bom condutor tem grande numero de eletrons livres de conducao, e eles se deslocam

com determinada velocidade escalar de deriva, enquanto que o bom isolante tem baixıssimo numero de

eletrons livres, e quando submetido ao campo eletrico acontece a polarizacao das moleculas. Os atomos

mais positivos (Kations) deslocam-se no sentido do campo, e os mais negativos (Anios), vice-versa. A

forca eletrica faz a molecula girar, segundo o sentido do campo externo.

PR 1.2: Fazer a analogia entre circuitos eletricos e hidraulicos, citando as grandezas fundamentais de

potencial e fluxo.

PR 1.3: Um motor de corrente contınua de 10 CV solicita uma corrente de 40 A quando operado a

plena carga ligado a uma rede de 220 V em corrente contınua (CC). Determinar: (a) o rendimento deste

motor; (b) qual a potencia perdida.

Solucao:

(a) Com o motor operando a plena carga (potencia nominal), a potencia na saıda (no eixo do motor)

e PS = 10× 736 = 7360W. A potencia na entrada (fornecida pela rede) e PE = 220× 40 = 8800W. O

rendimento e η = 83, 64%.

(b) As perdas no motor sao: PPERDA = PE − PS = 1140W. ♦

PR 1.4: Durante a operacao de descarga de uma bateria, a corrente eletrica foi de 50 A e a tensao 11,5

V. Sabendo que a taxa de transferencia de calor e de 30 W, qual e a taxa de diminuicao da energia

interna da bateria?

Solucao: Como as variacoes de energia potencial e cinetica nao sao significativas, a equacao do

equilıbrio energetico da bateria e:

Q =dU

dt+ W

onde W = 50× 11, 5 = 575W. Portanto

dU

dt= −30− 575 = 605 W. ♦

PR 1.5: O que e Efeito Joule? Qual a equacao para a potencia e a energia de um resistor?

Page 4: Problemas de Eletromagnetismo.pdf

1. Sistemas de coordenadas, carga e corrente eletrica 4

PR 1.6: Quais sao as unidades de resistencia, diferenca de potencial e intensidade de corrente?

PR 1.7: Utilizando os dados disponıveis em tabelas, calcule a resistencia de 1 m de um fio de ferro

envolto com alumınio, se o diametro do nucleo de ferro e 0.25 pol e o diametro externo e 0.50 pol. Se

o condutor transporta uma corrente contınua de 50 A, determine a potencia dissipada por polegada

quadrada de superfıcie do condutor externo.

PR 1.8: O elemento de aquecimento de uma certa torradeira eletrica consiste de uma tira de certa

qualidade de Nicromo, cujo comprimento e 1.5 m e a secao reta mede 0.05 m por 0.8 mm, com uma

resistividade de 1.1E-4Ω.cm. Encontre a corrente que circula no elemento quando ligamos entre os seus

terminais, uma fonte de 120 Vcc. Determine tambem a potencia dissipada.

PR 1.9: Em uma casa, abastecida com tensao de 110 V, seus moradores utilizam um chuveiro com duas

temperaturas: inverno e verao. Quando a chave esta acionada, o chuveiro trabalha com 5600 W. No

verao, opera com 3000 W. Qual e a diferenca de resistencia entre as duas faixas de temperatura? (R:

1,873 Ω)

PR 1.10: Um chuveiro eletrico possui tres opcoes de configuracao: quente, morno e desligado. Na opcao

A, o aquecimento d’agua se da por meio de uma resistencia de secao 1 mm2 e comprimento de 2 m. Na

opcao B utiliza-se a mesma secao, porem com 1 m de comprimento. Considerando a resistividade de 1

Ω mm2/m, pergunta-se: a) Qual a resistencia eletrica do chuveiro nas tres configuracoes? (R: 1 Ω e 2

Ω e infinito (circuito aberto). b) Qual a potencia de cada opcao, sabendo que o chuveiro esta ligado em

110 V? (R:12100 W e 6050 W).

PR 1.11: Qual e a resistencia de uma lampada em cujo bulbo se le 60 W e 110 V?

PR 1.12: Por que as linhas de transmissao de energia a longas distancias operam sob altas tensoes?

PR 1.13: Um chuveiro eletrico submetido a tensao constante, pode ser regulado para fornecer agua a

maior ou menor temperatura (inverno e verao respectivamente). A resistencia eletrica do chuveiro e

maior quando se deseja agua mais aquecida (inverno)? Por que?

PR 1.14: Um chuveiro eletrico foi construıdo para operar sob a tensao de 110 V. Para opera-lo a uma

tensao de 220 V , sem modificar a potencia de aquecimento, de quanto deve-se alterar a sua resistencia?

PR 1.15: Suponhamos que se necessita construir uma resistencia eletrica de 500 ohm com um condutor

de comprimento 100 m. Qual o valor da queda de tensao em cada espira, sabendo-se que a corrente

total e 2 A e que cada espira possui 1 cm de diametro?

Page 5: Problemas de Eletromagnetismo.pdf

1. Sistemas de coordenadas, carga e corrente eletrica 5

PR 1.16: Ao realizar um experimento em laboratorio, um estudante submeteu um resistor a diversas

diferencas de potencial V , e para cada caso mediu a corrente eletrica i. Com esses dados tracou um

grafico de V em funcao de i, onde os pontos lidos foram: Qual a resistencia eletrica desse resistor?

V(Volt) i(Ampere)

5 0,1

10 0,2

20 0,4

30 0,6

PR 1.17: Considerando um resistor de fio com resistividade ρ = 1/σ, comprimento `, secao A, e re-

sistencia R, demonstrar a lei de Ohm sob a forma local, quando ele e percorrido pela corrente I, e possui

uma tensao ou d.d.p. V nos seus terminais.

Solucao: Partindo da lei de Ohm

R =V

I= ρ

`

A

e, considerando que V = E ` e I = J A, encontra-se

E `

J A= ρ

`

A

que simplificando os termos ` e A resulta:

ρ =E

Jou ~J = σ ~EC ♦

PR 1.18: Demonstrar e explicar a equacao da refracao da corrente eletrica.

R: Vamos supor que a corrente passa de um meio 1 para um meio 2, com condutividades σ1 e σ2,

respectivamente, onde α e o angulo de ~E ou ~J com a normal nos meios 1 ou 2. No contorno de dois

materiais com condutividades diferentes σ1 e σ2, o princıpio da continuidade da corrente (a integral da

densidade de corrente numa superfıcie fechada e igual a zero):∮~J · d~S = 0

garante a continuidade da componente normal da densidade de corrente

Jn1 = Jn2 (1.1)

e o princıpio da circulacao do campo eletrico nao conservativo (a integral de linha fechada e igual a

zero), pois a diferenca de potencial na fronteira e igual para os dois meios:∮~E · d~= 0

garante a continuidade da componente tangencial do campo

Et1 = Et2 (1.2)

Page 6: Problemas de Eletromagnetismo.pdf

1. Sistemas de coordenadas, carga e corrente eletrica 6

ouJt1

σ1=Jt2

σ2(1.3)

Dividindo (1.3) por (1.1) temos

Jt1

σ1Jn1=

Jt2

σ2Jn2

outanα1

tanα2=σ1

σ2(1.4)

PR 1.19: Energia de uma bateria. Considere uma fonte de tensao contınua V , como uma bateria,

por exemplo. Certamente, Existe energia potencial eletrica armazenada? Calcular o seu valor, em

Joules, caso seja possıvel.

Solucao: Com certeza, existe energia potencial, mas nao temos dados para calcular sua quantidade.

Vamos ver que a energia potencial e W = 0, 5QV J, ou seja, precisamos da quantidade de carga, ou da

corrente eletrica e do tempo. ♦

PR 1.20: Explicar o que e, e estimar a velocidade escalar de deriva dos eletrons livres de conducao de

um fio de cobre com A = 2, 5 mm2 de secao, quando passa uma corrente de 1,3 A.

Resposta: Consideremos, como uma analogia, um grande numero de pessoas correndo em direcoes

aleatorias, e empurrando-se constantemente umas contra as outras. Elas seriam os eletrons livres ou

eletrons de conducao. Se esta multidao estiver sobre uma superfıcie inclinada (campo ou diferenca de

potencial eletrico), prosseguira lentamente no seu caminho segundo a direcao e o sentido da inclinacao.

Uma pequena, mas direcionada ‘velocidade de deriva’ (corrente eletrica) sera superposta ao movimento

aleatorio dos empurroes (agitacao termica). No cobre, existe em media um eletron livre de conducao

por atomo. O numero n de eletrons e, portanto igual ao numero de atomos por unidade de volume.

n =NA ρ

M= 8, 47× 1028eletrons/m3

onde:

NA - numero de avogadro (6, 02× 1023 eletrons/m3);

ρ - densidade do cobre (9, 0× 103 kg/m3); e,

M - massa molar do cobre (64× 10−3 kg/mol).

A carga livre total num trecho de fio de comprimento L e:

∆q = (n A L) e

e a velocidade de deriva

vd =L

∆t=

L

∆q/i=

i

A

L

L n e

Page 7: Problemas de Eletromagnetismo.pdf

1. Sistemas de coordenadas, carga e corrente eletrica 7

vd =J

ne

que para o nosso exemplo fica em torno de 14 cm/hora.

Obs.: cuidado para nao confundir velocidade escalar de deriva com velocidade de propagacao do

campo eletromagnetico, que e da ordem da velocidade da luz (c = 3× 108 m/s. ♦.

PR 1.21: Uma tira de silıcio (semicondutor tipo n, ‘dopado’ com uma impureza controlada de fosforo),

tem largura w = 3, 2 mm; espessura t = 250µm; e, n = 1, 5× 1023m−3. Determinar: a) a densidade de

corrente na tira; e, b) a velocidade escalar de deriva.

Resposta:

J =i

wt= 6500 A/m2 ou 0, 0065 A/mm2

vd =J

ne= 0, 27 m/s

Este valor e muito menor do que no cobre, porque . . .♦

PR 1.22: Vetor deslocamento infinitesimal de uma carga puntiforme Sejam duas cargas pun-

tiformes Q1 = 5µC e Q2 = 3µC, localizadas em P1 = (1, 1, 1)m e P2 = (2,−3, 4)m, respectivamente.

Qual e a energia necessaria para deslocar a carga Q2 para o ponto P3, localizado no meio do segmento

de reta P1P2?

Solucao: Calculou-se a energia potencial em P1P2 e P1P3

U12 = 0, 02647 J

U13 = 0, 05295 J

logo, a energia de um agente externo, necessaria para deslocar Q2 de P2 para P3 e 0, 02648 joules.

Pode-se calcular esta energia de uma maneira mais geral, calculando o trabalho ao longo do trecho

P2P3. Sabemos que o trabalho e o produto escalar do vetor forca pelo vetor deslocamento. E, quando

a forca nao e constante, precisamos usar a integral de linha, com o vetor deslocamento infinitesimal d~.

O vetor deslocamento infinitesimal, em coordenadas cartesianas, e sempre dado por:

d~= dxax + dyay + dzaz

onde precisamos deixa-lo em termos de apenas uma variavel, que denominaremos t.

Fazendo t = 0 no ponto inicial, e t = 1 no ponto final, podemos escrever a equacao da reta como

x = a1t+ b1

y = a2t+ b2

z = a3t+ b3

Page 8: Problemas de Eletromagnetismo.pdf

1. Sistemas de coordenadas, carga e corrente eletrica 8

PR 1.23: Tres cargas puntuais de 4µC, localizam-se nos vertices de um triangulo equilatero de 0, 5mm,

situado no vacuo. Que trabalho deve ser realizado para deslocar uma das cargas ate o ponto medio do

segmento determinado pelas outras duas cargas? Sugestao: calcular a diferenca de potencial entre o

ponto final e inicial (R: 575 J)

Page 9: Problemas de Eletromagnetismo.pdf

2. LEI DE COULOMB

PR 2.1: Lei da atracao das massas e das cargas. Sabendo que a distancia media entre o proton e

o eletron no atomo de hidrogenio e de 5, 3× 10−11m, compare as forcas eletrica e gravitacional.

Solucao: Considerando-se duas massas m1 e m2, afastadas de uma distancia d, o modulo da forca

gravitacional vale

Fg = 6, 67× 10−11m1 m2

d2Newton

Fg = 6, 67× 10−11 (9, 11× 10−31) (1, 67× 10−27)(5, 3× 10−11)2

Fg = 3, 6× 10−47N.

O modulo da forca eletrica vale

Fe = 9, 0× 109 (1, 61× 10−19)2

(5, 3× 10−11)2= 8, 2× 10−8N

A forca gravitacional e muitas vezes mais fraca do que a forca eletrostatica. Entretanto, a forca grav-

itacional e sempre atrativa, e pode agregar grandes massas, enquanto a eletrostatica pode ser repulsiva.

PR 2.2: Campo eletrico. Considere uma carga puntiforme Q1 C, situada no ponto P1(x1, y1, z1) m e

outra carga puntiforme Q2 C, situada no ponto P2(x2, y2, z2) m. Escrever a equacao do vetor campo

eletrico produzido por esta carga Q em qualquer ponto P (x, y, z) do espaco.

Solucao: Vamos supor que no ponto P tenhamos uma carga de prova q. Usando a Lei de Coulomb,

vamos escrever a equacao da forca entre as duas cargas Q1 e q; e depois Q2 e q. Para isto, podemos

seguir a sequencia abaixo:

- Vetor deslocamento de P1 ate P , que denominaremos vetor ~r1

~r1 = (x− x1)ax + (y − y1)ay + (z − z1)az

- Modulo ou intensidade do vetor deslocamento |~r1| ou simplesmente r1

|~r1| = r1 =√

(x− x1)2 + (y − y1)2 + (z − z1)2

- Vetor unitario ~ur1

~ur1 =~r1|~r1|

Page 10: Problemas de Eletromagnetismo.pdf

2. Lei de Coulomb 10

- Vetor campo eletrico ~E1, que e a forca dividida pela carga de prova q.

~E1 = kQ1

r21~ur1

~E1 = k Q1~r1r31

Da mesma forma obtemos ~E2, e o campo resultante

~E = ~E1 + ~E2 ♦

PR 2.3: Potencial eletrico de um dipolo. Considere um dipolo, formado por duas cargas pun-

tiformes +Q = 10µC e −Q = −10µC, localizadas em +P = (1, 0, 0)m e −P = (−1, 0, 0)m, respectiva-

mente. Explicar o que e potencial eletrostatico, e escrever sua equacao no plano z = 1 metros, referente

ao dipolo?

Solucao: O potencial de 1 Volt e a energia potencial de 1 Joule para deslocar uma carga de prova

unitaria q = 1 Coulomb desde o infinito ate o ponto de raio R. O potencial num ponto distante R de

uma carga puntiforme de fonte Q e:

W = −∫Fdr = −

∫ R

∞KQ q

r2dr

V = K Q

∣∣∣∣1r∣∣∣∣R∞

V = K Q1R

O potencial resultante e a soma dos potenciais das cargas positiva e negativa.

V = V + + V − ♦

PR 2.4: Energia armazenada em cargas puntiformes. Demonstrar a equacao da energia ar-

mazenada em cargas puntiformes.

**Sistema com duas cargas puntiformes

Consideremos um sistema com duas cargas puntiformes Q1 e Q2, situadas a uma distancia infinita,

tal que nao exista forca em nenhuma carga, e elas estao em equilıbrio. De acordo com a definicao de

potencial: uma intensidade de trabalho igual a Q2 vezes o potencial de Q1 deve ser aplicado sobre a

carga Q2 e para aproxima-la de Q1. Entao a energia potencial do sistema e aumentada da quantia

W2 = Q2V12

onde V 12 e o potencial de Q1 no local que o agente externo fixou Q2.

**Sistema com tres cargas puntiformes

Consideremos, agora, um sistema com tres cargas puntiformes Q1, Q2 e Q3, situadas a uma distancia

infinita, tal que nao exista forca em nenhuma carga, e elas estejam em equilıbrio. De acordo com a

Page 11: Problemas de Eletromagnetismo.pdf

2. Lei de Coulomb 11

definicao de potencial: uma intensidade de trabalho igual a Q3 vezes o potencial de Q1, mais Q3 vezes

o potencial de Q2 deve ser aplicado sobre a carga Q3 e para aproxima-la de Q1 e Q2. Entao a energia

potencial do sistema e aumentada da quantia

W3 = Q3V13 +Q3V

23

Entao o trabalho total necessario para aproximar Q2 e Q3 de Q1 e

We = W2 +W3 = Q2V12 + (Q3V

13 +Q3V

23 )

**Sistema com n cargas puntiformes

Assim, o trabalho para aproximar n− 1 cargas da carga Q1

We = W2 +W3 + . . .+Wn

We = Q2V12 + (Q3V

13 +Q3V

23 ) + (Q4V

14 +Q4V

24 +Q4V

34 +) + . . .

ou

We =n∑

i=2

i−1∑j=1

QiVji (2.1)

onde

QiVji = Qi

Qj

4πε0Rji= Qj

Qi

4πε0Rij= QjV

ij

Tambem pode-se escrever a energia como:

We = W1 +W2 + . . .+Wn−1

We = Q1V21 + (Q1V

31 +Q2V

32 ) + (Q1V

41 +Q2V

42 +Q3V

43 +) + . . .

ou

We =n∑

i=2

i−1∑j=1

QjVij (2.2)

Somando-se 2.1 e 2.2, tem-se

2We = Q1(V 21 + V 3

1 + V 41 + . . .) +Q2(V 1

2 + V 32 + V 4

2 + . . .) + . . .

Que resulta na energia potencial armazenada num sistema de n cargas puntiformes:

We =12

n∑i=1

Qi Vi

onde Vi e o potencial criado por todas as outras cargas j 6= i.

Observamos que quando se transfere uma carga positiva de um ponto de menor potencial para outro

ponto do espaco com maior potencial, um agente externo precisa realizar um trabalho. ♦

Page 12: Problemas de Eletromagnetismo.pdf

2. Lei de Coulomb 12

PR 2.5: Energia de duas cargas puntiformes. Sejam duas cargas puntiformes Q1 = 5µC e Q2 =

3µC, localizadas em P1 = (1, 1, 1)m e P2 = (2,−3, 4)m, respectivamente. Qual e a energia potencial

armazenada no campo eletrostatico?

Solucao: Vamos calcular a energia potencial pelo trabalho realizado por um agente externo para

deslocar uma das cargas (vamos escolher Q2) desde o infinito ate o ponto final (P2).

W = −∫Fdr = −

∫ d12

∞KQ1 Q2

r2dr

W = K Q1 Q2

∣∣∣∣1r∣∣∣∣d12

W = 0, 02647 J

Tambem podemos calcular a energia potencial usando o potencial eletrico, como feito na rotina EP2Q.for,

descrita abaixo.

c Duas cargas puntiformes

Q1=5.0E-6

Q2=3.0E-6

x1=1.

y1=1.

z1=1.

x2=2.

y2=-3.

z2=4.

c Distancia entre 1 e 2

d12=sqrt((x2-x1)**2+(y2-y1)**2+(z2-z1)**2)

c Potencial da carga 1 em 2

V12= 9.0E9*Q1/d12

c Potencial da carga 2 em 1

V21= 9.0E9*Q2/d12

c Energia potencial de duas cargas

EP2q=0.5*V12*Q2+0.5*V21*Q1

write(*,*) EP2q

stop

end

PR 2.6: Carga total de um fio. Um fio retilıneo, com 3 m de comprimento, esta situado sobre a reta

x = 2 e y = 3, desde z1 = 0 ate z2 = 3m. A densidade de carga linear ρz = 4zµC/m. Qual a carga do

fio?

Page 13: Problemas de Eletromagnetismo.pdf

2. Lei de Coulomb 13

Solucao: Q =∫ z2z1ρz dz

Q =∫ 3

04, 0E − 6 z dz = 18 µC. ♦

PR 2.7: Carga de uma figura bidimensional. Calcular a carga compreendida na superfıcie delimi-

tada pelas curvas y = x/2 e y =√x, desde x = 2 a x = 4, quando a carga superficial ρs = xy µC/m2.

Solucao: Vamos encontrar inicialmente a densidade de carga linear para cada valor de x, que de-

nominaremos q(x). Assim:

q(x) =∫ √

x

x/2xy dy =

x2

2− x3

8µC/m

Agora, podemos calcular a carga total, fazendo a integral em x.

Q =∫ x2

x1

q(x) dx =∫ 4

2(x2

2− x3

8) dx =

116µC. ♦

PR 2.8: Carga total de um disco. Um disco de raio R, centrado na origem, esta situado sobre no

plano x− y, e possui densidade de carga superficial ρs = r2 µC/m2. Qual a carga total do disco?

Solucao: Q =∫ ∫

ρs ds

Q = 4∫ y2=0

y1=R

∫ x2=√

R2−y2

x1=0(x2 + y2) dx dy

Q = 4∫ y2=0

y1=R

((R2 − y2)3/2

3+ y2(R2 − y2)1/2

)dy

Q =πR4

2µC.

Lembrete: Sempre que voce encontrar uma expressao envolvendo x2 + y2 no integrando, precisa

considerar a possibilidade de converter para coordenadas polares. Vejamos como ficaria a solucao deste

exemplo:

dS = r dφ dr

Q =∫ r2=R

r1=0

∫ φ2=2π

φ1=0r2 r dφ dr =

πR4

2µC. ♦

PR 2.9: Densidade de carga volumetrica. Demonstrar a equacao do volume de uma esfera de raio

R, e a seguir, fazer o calculo da carga total, quando a densidade volumetrica e ρ = 3r cosφ C/m3.

Solucao: Em coordenadas esfericas, o volume infinitesimal e

dv = dr r dθ r sen θ dφ

e a integral em dv vale

V =∫ θ2=π

θ1=0

∫ φ2=2π

φ1=0

∫ r2=R

r1=0r2 sen θ dφ dr dθ

Page 14: Problemas de Eletromagnetismo.pdf

2. Lei de Coulomb 14

V =43πR3

Observacao: se tivermos duvidas sobre os limites de integracao, podemos calcular o volume, a area ou

o comprimento da figura cuja resposta ja seja conhecida. Agora que acertamos os limites de integracao,

podemos fazer o calculo da carga com seguranca. ♦

PR 2.10: Produtos escalar e vetorial. Estas duas operacoes com vetores sao muito usadas no

eletromagnetismo, pois estao presentes em todas as equacoes de Maxwell. Sejam dois vetores ~A =

A1ax +A2ay +A3az e ~B = B1ax +B2ay +B3az, defasados de um angulo θ, explicar os produtos escalar

e vetorial.

Produto escalar - esta associado ao movimento de translacao, isto e, quanto um vetor contribui com

o outro para modificar o seu modulo. O produto escalar e utilizado para calcular o fluxo de um vetor,

ou o trabalho realizado por uma forca ao longo de um percurso. O resultado e um escalar, que vale zero

quando os vetores sao ortogonais.

~A · ~B = A1B1 +A2B2 +A3B3 = | ~A| | ~B| cos θ (2.3)

Produto vetorial - esta associado ao movimento de rotacao, isto e, quanto um vetor contribui com

o outro para modificar o seu angulo. O produto vetorial e usado para calcular um momento angular.

O resultado e um vetor ortogonal ao plano formado pelos dois vetores que estao sendo multiplicados.

Quando os dois vetores sao paralelos, o resultado e o vetor nulo.

~A× ~B =

∣∣∣∣∣∣∣∣∣ax ay az

A1 A2 A3

B1 B2 B3

∣∣∣∣∣∣∣∣∣ = |~A| | ~B|sen θ~n (2.4)

onde ~n e um vetor ortogonal ao plano formado por ~A e ~B, e sentido dado pela regra da mao direita (ou

do parafuso), de ~A e ~B. ♦

PR 2.11: Equacao do trabalho realizado por uma forca. Qual e o trabalho realizado por uma

forca do agente externo de 20 Newton na direcao 45o Nordeste, que movimenta um corpo por 0,3 metros

na direcao Oeste para Leste?

Solucao: O trabalho W e definido como uma forca ~F agindo atraves de um deslocamento infinitesi-

mal d~, onde a forca e aplicada na direcao do deslocamento. Assim, temos a integral do produto escalar

entre os dois vetores:

W =∫ 2

1

~F · d~ (2.5)

Para o nosso exemplo

W = 20 . 0, 3 . cos 45o = 4, 24 J

Page 15: Problemas de Eletromagnetismo.pdf

2. Lei de Coulomb 15

W = ~F · δ~= 20(cos 45ax + sen 45ay) ·~(0, 3ax)

W = 20× 0, 3(cos 45ax · ax + sen 45ay · ax = 4, 24) J.

Embora a forma vetorial nos pareca mais difıcil, ela simplifica muito o estudo, a representacao e o

calculo de campos eletromagneticos. ♦

Page 16: Problemas de Eletromagnetismo.pdf

3. CAMPO E POTENCIAL ELETRICO

PR 3.1: Dipolo eletrico - Considere duas cargas puntiforme +Q, situada em (+d/2, 0, 0), e −Q, situada

em (−d/2, 0, 0). Obter a equacao do campo eletrico num ponto P (xp, yp, 0).

Solucao: O campo resultante das duas cargas e ~E = ~E+ + ~E−.

~E =Q

4πε0r2+~ur+ −

Q

4πε0r2−~ur−

Denominando ~ur o vetor unitario radial, onde ~ur · ~ur+ = cosα+ e ~ur · ~ur− = cosα−, inserindo o angulo

α formado entre ~r com a linha do dipolo, e desenvolvendo a componente radial tem-se a componente

radial do campo tem-se

Er =Q

4πε0r2+cosα+ −

Q

4πε0r2−cosα−

Er∼=

Q

4πε0r32 d cosα

O produto Q d e conhecido como momento de dipolo p. Duas cargas de mesma intensidade e opostas

mantidas a certa distancia d uma da outra, possuem momento de dipolo eletrico (nao e momento

mecanico) ~p, que e um vetor de modulo igual ao produto da intensidade das cargas pela distancia entre

elas.

~p = Q~d

onde ~d aponta da carga negativa para a positiva. As componentes do campo eletrico em um ponto

generico P situado a uma distancia r do centro do dipolo sao dados por:

EN =2p cosα4πε0r3

ET =psen α4πε0r3

E assim, o modulo do campo eletrico do dipolo vale

~E =p

4πε0r3(2 cosα~ur + 2sen α~uα)

Observa-se que o campo e inversamente proporcional ao cubo da distancia. Se tivessemos 3 cargas,

o campo seria inversamente proporcional a r4, e assim por diante.

Fazendo r = x, temos o campo na direcao do eixo

E =2p

4πε0x3♦ (3.1)

Page 17: Problemas de Eletromagnetismo.pdf

3. Campo e potencial eletrico 17

PR 3.2: Tres cargas puntuais iguais a 20pC localizam-se, no vacuo, sobre o eixo x em x = −1, x = 0

e x = 1. (a) Determine a forca resultante que age sobre uma carga de 1C situada em P (1, 10, 2). (b)

Substitua as tres cargas por uma unica carga igual a 60pC localizada na origem, e determine a forca na

carga de 1C. (c) Por que as respostas dos itens (a) e (b) sao semelhantes?

PR 3.3: Fio infinito carregado - Imaginemos um fio infinito carregado com uma carga definida por sua

densidade linear de carga qL, conforme a Fig. 3.1.

qL

R d~Er

d ~E

QQ

QQ

QQ

QQ

QQ

QQ

r

O

PL

dLdQ

QQ

QQQs

-

Fig. 3.1: Fio infinito carregado.

O fio e constituıdo de uma infinidade de cargas puntuais dQ. Cada carga dQ produzira um vetor

campo eletrico d ~E no ponto P , que tem modulo

dE =1

4πεdQ

r2

Tendo em vista que a distribuicao de cargas e uniforme, escreve-se

dQ = qL dL

Por outro lado, observa-se que a componente vertical de d ~E sera anulada pela componente correspon-

dente quando considerarmos um outro dQ situado simetricamente em relacao ao ponto O. A componente

horizontal, que nos interessa, e

dEr =1

4πεqL dL

r2cos θ

As variaveis L, r e θ sao interdependentes; e pode-se reescrever dEr como:

dEr =1

4πεqL dL

r2cos θ

Agora, pode-se fazer o somatorio dos campos dEr causados pelas cargas dQ, usando os angulos +π/2 e

−π/2 como limites de integracao. Observa-se que +π/2 e −π/2 sao os angulos que definem a dimensao

infinita do condutor. Caso o fio fosse finito, seriam outros os angulos limites que determinariam o inıcio

e o fim do condutor, bem como a posicao do ponto P em relacao ao segmento de condutor em questao.

Er =∫ +π/2

−π/2

14πε

qL dL

r2cos θ =

qL2πεR

V/m

Page 18: Problemas de Eletromagnetismo.pdf

3. Campo e potencial eletrico 18

B

A

P

R

Na continuidade do curso, calcularemos este campo de um modo bem mais simples, usando a lei de

Gauss. ♦

PR 3.4: Deduzir a equacao do campo eletrico produzido por um segmento de fio retilıneio, uniforme-

mente carregado com carga total Q, de comprimento A+B, em um ponto P , situado longitudinalmente

a uma distancia R do fio.

PR 3.5: Campo de um cabo coaxial. Considere um cabo coaxial, onde o condutor interno, de raio a,

esta carregado com densidade de carga +ρs, o condutor externo, de raio b, esta carregado com densidade

de carga linear −ρL coulomb/metro. Determinar o campo no isolante situado entre os dois condutores,

a uma distancia r do centro (a < r < b).

Resposta: Este problema classico, muito importante, tem solucao

E = Er =ρL

2πε0εrr

A constante εr e a constante dieletrica do isolante, e e apenas um fator que altera a intensidade do

campo, como veremos mais adiante. Observamos tambem que o campo e maximo quando o raio for

mınimo, isto e r = a, e vice-versa quando r = b. ♦

PR 3.6: Campo de um anel carregado de raio R. Determinar o campo num ponto situado no eixo do

anel a uma distancia z do seu centro.

Resposta: A componente radial vale

E =Q z

4πε0(z2 +R2)3/2

Exercıcio: dividir o anel em N cargas puntiformes, e fazer uma rotina para calcular as componentes e

a resultante do campo num ponto qualquer do espaco. ♦

Page 19: Problemas de Eletromagnetismo.pdf

3. Campo e potencial eletrico 19

PR 3.7: Disco carregado. Determinar o campo eletrico num ponto situado ao longo do eixo de um disco

de raio R carregado uniformemente com carga ρs.

Resposta: Situando o disco no plano ‘xy’, com centro coincidente com a origem do sistema de

coordenadas, substituindo os elementos infinitesimais e integrando, obtemos

E = Ez =ρs

2ε0(1− z√

z2 +R2)

PR 3.8: Chapa plana infinita. Determinar o campo num ponto situado a uma distancia d de uma

chapa infinita, carregada uniformemente com densidade de carga superficial ρs.

Resposta: Este e um problema muito importante, devido sua grande aplicac ao. E importante obter

a resposta

E = En =ρs

2ε0onde observamos que a intensidade e direcao do campo e independente da distancia d do ponto ate a

chapa. Inicialmente, isto poderia parecer impossıvel, mas e bom destacar que estamos tratando de uma

chapa plana infinita. ♦

PR 3.9: Meia-casca esferica. Determinar o campo num ponto situado no centro de uma meia-casca

esferica de raio R, carregada uniformemente com densidade de carga superficial ρs.

Resposta: Este e outro problema classico de Eletromagnetismo, que tem como solucao

E = Er =ρs

4ε0

Sugestao: utilizar coordenadas esfericas, usando dS = R2senθdφdθ. ♦

PR 3.10: Dado o campo eletrico ~E = zax−3y2ay+xaz V/m, calcular o trabalho realizado por um agente

externo, para mover uma carga de 7µC ao longo de um caminho incremental de 1mm de comprimento,

na direcao do vetor 2ax − 6ay − 3az m, localizado em: a) PA(1, 2, 3) (R: 511 nJ) e em b) PB(2, 0,−4)

(R:-98 nJ)

PR 3.11: Para o campo eletrico ~E = zax − 3y2ay + xaz V/m, determinar o trabalho realizado por um

agente externo, para deslocar uma carga de 7µC ao longo de um caminho retilıneo desde A(1, 0, 2) ate

B(3, 2, 2), fazendo o calculo analıtico e numerico, dividindo o trecho de reta em 05(cinco) segmentos.

(R: 28 µJ)

Page 20: Problemas de Eletromagnetismo.pdf

3. Campo e potencial eletrico 20

PR 3.12: A Fig. abaixo ilustra o movimento de uma carga q0, na presenca do campo eletrostatico

produzido por outra carga q. O trabalho realizado sobre a carga q0, num deslocamento infinitesimal d~

e

dW = −~F · d~= −q0 ~E · d~♦

O sinal e negativo quando a forca for interna ou de reacao ao agente externo.

PR 3.13: Consideremos inicialmente o trecho 1 → 2. A variacao da energia cinetica da carga q0 neste

trecho e

T2 − T1 =∫ 2

1q0 ~E · d~= k q0 q

∫ 2

1

~r · d~r2

T2 − T1 = k q0 q

∫ 2

1

dr

r2= −k q0 q

(1r2− 1r1

)Suponhamos agora que a carga q0 percorra todo o trajeto mostrado na figura, retornando ao ponto 1

de partida. Caso sua energia cinetica fosse, por exemplo, maior que a inicial, terıamos uma forma de

produzir energia do nada! Sabemos que isto nao e possıvel, pois nao existe um moto perpetuo. Portanto,

devemos ser capazes de demonstrar que o trabalho realizado ao longo de qualquer trajetoria

fechada e nulo. Caso uma determinada trajetoria resultasse em um trabalho negativo (diminuindo a

energia cinetica da carga q0), poderıamos inverter o sentido da trajetoria obtendo assim um ganho de

energia cinetica. Vamos primeiro mostrar que o trabalho e de fato nulo para a trajetoria simples vista

na figura. Note que, nos trechos 2→ 3, 4→ 5, 6→ 7 e 8→ 1, a carga q0 desloca-se perpendicularmente

a direcao do campo radial ~E. Portanto, o trabalho e nulo nestes trechos (dW = ~E ·d~= 0). Nos trechos

onde o trabalho nao e nulo temos

W12 = k q0 q

∫ 2

1

dr

r2= −k q0 q

(1r2− 1r1

)

W34 = k q0 q

∫ 4

3

dr

r2= −k q0 q

(1r4− 1r3

)W56 = k q0 q

∫ 6

5

dr

r2= −k q0 q

(1r6− 1r5

)W78 = k q0 q

∫ 8

7

dr

r2= −k q0 q

(1r8− 1r7

)O trabalho total e a soma dos trabalhos em cada trecho;

W = −k q0 q(

1r2− 1r1

+1r4− 1r3

+1r6− 1r5

+1r8− 1r7

)notando que r2 = r3, r4 = r5, r6 = r7 e r1 = r8, concluımos facilmente que W = 0.

A curva utilizada no exemplo anterior pode parecer muito especial. Deixamos como trabalho, veri-

ficar o que acontece em uma situacao mais geral. Se o trabalho total para deslocar uma carga ao longo

de uma superfıcie fechada for nulo, chamamos este campo de campo conservativo.

Concluımos este item dizendo que o trabalho realizado por uma forca conservativa so depende da

posicao dos pontos inicial e final. ♦

Page 21: Problemas de Eletromagnetismo.pdf

3. Campo e potencial eletrico 21

PR 3.14: Tres cargas puntuais QA = 5nC, QB = −2nC e QC = 1nC se localizam no vacuo, nos pontos

PA(1, 2,−2), PB(−2, 4, 1) e PC(3, 0, 2), respectivamente. Determine o potencial em P (−3,−2, 4) se: (a)

V = 0 no infinito (R: 4,15 V); (b) V = 5 V no infinito (R: 9,15 V); e, (c) V = 0 em P (0, 5,−2) (R:-6,52

V).

PR 3.15: Uma distribuicao linear e uniforme de carga, de 0, 6 nC/m, esta situada ao longo do eixo z

no vacuo. Determine o potencial em P (3, 4, 2) se: (a) V = 0 em A(2,−9, 3) (R: 6,6 V); (b) V = 24 V

em B(10, 24, 1) (R:41,8 V).

PR 3.16: Um longo fio cilındrico, reto, de diametro muito pequeno, e colocado no espaco livre, e possui

uma carga Q por unidade de comprimento. Usando a lei do inverso do quadrado das distancias, entre

um ponto distante x do centro do fio, que e muito maior que o diametro do fio, definir qual e a forma

das superfıcies equipotenciais, e qual e a diferenca de potencial entre dois pontos distantes x1 e x2 do

fio condutor? Se Q = −0.10E− 9 coulomb por metro, x1 = 0.40 cm e x2 = 15.70 cm, qual e a diferenca

de potencial entre os dois pontos?

PR 3.17: Uma distribuicao superficial plana e uniforme de carga ρS = 40ε0 C/m2, localiza-se no vacuo,

no plano x = 0. Qual e o potencial em B(7,−3, 1) se V = 0 volt em A(2, 4, 6). (R: -100 V)

PR 3.18: Fazer uma tabela resumo com o campo eletrico e o potencial para as principais distribuicoes

de carga, estudadas ate o momento, e compara-las.

PR 3.19: Dado o campo eletrico

~E =10xax

x2 + y2+

10yay

x2 + y2− 2az V/m

e sabendo-se que o potencial no ponto (3, 4, 5) metros vale 10 volt, determine o potencial no ponto

(6,−8, 7). (R:7,07 V)

PR 3.20: Qual e o trabalho necessario para movimentar uma carga q = 100µC ao longo da circunferencia

de raio r = 0, 1m centrada na origem, no campo eletrico ~E = (40/r)~uφ V/mm? Existe diferenca de

potencial entre o inıcio e o fim do percurso? (R: 25,13 mJ)

PR 3.21: Um campo escalar e representado por T = 2xy − 5z. (a) Determine o campo vetorial

~S = (∂T/∂x)ax + (∂T/∂y)ay + (∂T/∂z)az

para o ponto P (1, 2, 3), e determine: (b) T ; (c) ~S; e (d) ~uS .

PR 3.22: Admitindo que o campo eletrico seja uniforme na direcao ox, calcular o campo eletrico ~E

entre duas placas metalicas planas, espacadas de uma distancia l.

Page 22: Problemas de Eletromagnetismo.pdf

3. Campo e potencial eletrico 22

-x6y

V=1000V V=600V

- -l=0,2 mm

Solucao: O modulo do campo eletrico e 400V/0, 0002m = 2 × 106V/m. Como o campo eletrico

somente possui a componente horizontal, resulta

~E = Exax = 2× 106axV/m

Concluımos que, estabelecendo V em todo o domınio, estamos estabelecendo tambem o campo

eletrico ~E.

O componente de ~E em qualquer direcao e o negativo da taxa de variacao do potencial eletrico com

a distancia naquela direcao. Se tomarmos o sistema de coordenadas cartesianas, o campo eletrico em

qualquer ponto e

Ex = −∂V∂x

Ey = −∂V∂y

Ez = −∂V∂z♦

PR 3.23: Existe relacao entre gradiente e integral de linha? Gradiente do potencial = campo e∫

campo

= potencial ?

PR 3.24: Qual e o campo eletrico no interior de um condutor ideal? Por que?

PR 3.25: Considerando que o potencial eletrico de um ponto qualquer seja dado pela equacao V (x, y) =

3xy2, obter a equacao para o campo eletrico.

PR 3.26: Suponha que o potencial eletrico aumente 100 kV/m na direcao Norte-Sul e diminua 50 kV/m

na direcao Leste-Oeste. Qual e o modulo e a direcao do campo eletrico?

PR 3.27: Se o potencial eletrico e dado por V (r, φ, z) = 24rsen (φ+ π/9) V, qual e o valor do modulo

e a direcao do campo eletrico em x = 2, y = 1 e z = 0?

PR 3.28: Sabendo-se que nos pontos P1(2, 1), P2(7, 4) e P3(3, 8) os potenciais sao respectivamente V1 =

5V, V2 = 10V e V3 = 19V, obter os coeficientes a, b e c para o potencial V = a + bx + cy. Qual e o

campo eletrico correspondente a este potencial?

PR 3.29: Demonstrar que o potencial num ponto P devido a duas cargas −Q e +Q e a soma dos

potenciais devidos as duas cargas, isoladamente, e

V =Q

4πε~p · ~rr2

e que o campo eletrico tambem pode ser obtido pelo gradiente de V :

~E = −~∇V

Page 23: Problemas de Eletromagnetismo.pdf

4. LEI DE GAUSS E CAPACITANCIA

PR 4.1: Um capacitor de placas planas paralelas tem uma area de 1.51 m2 distantes de 1.0 mm, com

um isolante de constante 3,2. Este capacitor esta ligado a uma d.d.p. de 13800 Volts. Determinar os

valores de:

• Intensidade do campo eletrico;

• Intensidade do vetor inducao eletrica;

• Intensidade do vetor polarizacao eletrica.

• Densidade de carga;

• Carga livre acumulada; e,

• Capacitancia.

PR 4.2: Duas placas planas paralelas tem uma area de 1,51 m2 e separacao de 10 mm (com ar εr = 1),

e estao submetidas a uma d.d.p. de 50 V. Desprezando o efeito de borda, calcular: (a) O campo

eletrostatico E; (b) A inducao eletrostatica D; (c) A polarizacao P ; e, (d) A carga livre Q e polarizada

q. A seguir, inseriu-se um dieletrico com constante dieletrica εr = 2, 5. Quais serao os novos valores de

E, D e P?

Solucao: O campo eletrostatico vale

E = 50/10/0, 001 = 5000 V/m

A inducao eletrostatica D e

D = ε0 εr E = 8, 85× 10−12 × 1× 5000 = 44, 25 nC/m2

A polarizacao P vale

P = D − ε0E = 0

A carga livre nas placas e

Q = D S = 44, 25× 1, 51 = 66, 81 nC

e nao existe carga polarizada. Este resultado era esperado, pois o dieletrico inicial e o ar.♦

Page 24: Problemas de Eletromagnetismo.pdf

4. Lei de Gauss e Capacitancia 24

PR 4.3: Para o exemplo anterior, com um dieletrico de constante dieletrica εr = 2, 5, temos ainda

E = 5000 V/m, pois a tensao aplicada se manteve constante. A nova inducao eletrostatica D sera

D = ε0 εr E = 8, 85× 10−12 × 2, 5× 5000 = 110, 625 nC/m2

e a polarizacao P

P = D − ε0E = 110, 625− 44, 25 = 63, 375 nC/m2

A carga livre nas placas e

Q = D S = 110, 625× 1, 51 = 167, 04 nC

e a carga polarizada.

q = P S = 63, 375× 1, 51 = 95, 69 nC

Observamos que houve um aumento da carga livre nas placas, mas a diferenca Q− q permaneceu con-

stante. O estudo dos dieletricos adquire grande relevancia na construcao de dispositivos armazenadores

de energia eletrica, tambem conhecidos como condensadores ou capacitores, os quais constam basica-

mente de duas placas condutoras com potencial eletrico distinto, entre as quais se intercala a substancia

dieletrica. A capacidade de armazenamento de um condensador se avalia mediante um coeficiente -

conhecido como capacitancia - que depende de suas caracterısticas fısicas e geometricas. Essa grandeza

tem dimensoes de carga por potencial eletrico e se mede comumente em farads (coulombs por volts). ♦

PR 4.4: A constante dieletrica da Ebonite e 2.8, e sua rigidez dieletrica vale 18106 V/mm. Qual a

menor area que podem ter as placas de um capacitor plano de 7.0E-2 microfarad, usando a ebonite

como dieletrico, para que este suporte uma diferenca de potencial de 4000 V.

PR 4.5: O campo eletrico, num ponto qualquer do espaco, e ~E = 2ax + 3ay − 5az V/m. Sabendo que

a permissividade relativa do meio e 4, 5, qual e o vetor e o modulo da densidade de fluxo eletrico, ou

intensidade do vetor deslocamento eletrico, ou inducao eletrica ~D, neste ponto?

PR 4.6: Uma esfera metalica tem 0,5 m de raio com 20 µC distribuıdo em sua superfıcie. Qual a

densidade de fluxo eletrico na superfıcie externa da esfera? Qual e a densidade de fluxo a 0,5 m distante

da esfera? (R: 6,36 µC/m2 e 1,59 µC/m2)

PR 4.7: Uma esfera condutora metalica de raio R = 0, 5 m foi carregada com uma carga de 1 µC.

Como se distribuem as cargas nesta esfera? Qual a densidade superficial de carga nessa esfera? (R:

1/πµC/m2)

PR 4.8: Em um ponto P (−2, 5,−4) em uma superfıcie condutora esferica, a densidade superficial de

carga e 75 nC/mm2. Se o condutor esta isolado no vacuo, encontre ~E fora e dentro do condutor nas

vizinhancas do ponto P .

Page 25: Problemas de Eletromagnetismo.pdf

4. Lei de Gauss e Capacitancia 25

PR 4.9: A densidade de carga eletrica de uma casca de raio interno 10 cm e raio externo 11 cm e dada

por ρ(r, θ, φ) = 2× 10−10/r2 C/m3. Pergunta-se: a) Qual a carga total da casca carregada? b) Qual a

intensidade, direcao e sentido do campo eletrico na superfıcie interna; e, c) na superfıcie externa?

PR 4.10: Fio infinito carregado - Este problema pode ser resolvido de duas maneiras distintas: imagi-

nando uma superfıcie simetrica ou fazendo a integral dos fluxos.

Se um condutor longo, reto e cilındrico tem uma carga eletrica uniforme ao longo de seu comprimento

e esta isolado de outras cargas de modo que sua carga esteja uniformemente distribuıda em sua periferia,

o fluxo sera radial. Por simetria, todos os pontos equidistantes desse condutor tem a mesma densidade

de fluxo eletrico. Assim, a densidade de fluxo eletrico a r metros do condutor pode ser calculada

imaginando uma superfıcie gaussiana cilındrica concentrica ao condutor, de raio r. A area da superfıcie

lateral do cilindro com 1m de altura e 2πr m2. Considerando a carga qL Coulombs por metro de

condutor, o modulo da densidade de fluxo eletrico e

D =qL2πr

coulombs/metro quadrado

Finalmente, dividindo D pela permissividade do meio ε, encontramos o campo eletrico no ponto P .

E =qL

2πε0rV/m

que e igual ao valor obtido pela lei de Coulomb. ♦

PR 4.11: Cabo coaxial - Demonstre a equacao do campo eletrico de um cabo coaxial.

E =qL

2πε0εrr

O resultado e igual ao campo de um condutor retilıneo, obtido pela lei de Coulomb. ♦

PR 4.12: Superfıcie plana infinita. Se cortarmos um cilindro (superfıcie gaussiana) exatamente no seu

centro, com o plano das cargas, o fluxo total do cilindro e igual a duas vezes o fluxo de cada base.

2×D × S = ρS × S

isolando D e dividindo pela permissividade

E =ρS

2ε0εr

O resultado indica que o campo nao varia com a distancia. Esta equacao pode ser usada para calcular

o campo de duas placas paralelas. ♦

Page 26: Problemas de Eletromagnetismo.pdf

4. Lei de Gauss e Capacitancia 26

PR 4.13: Calcular para o ponto P (0.1,−4.3, 0.6), a divergencia de cada um dos seguintes campos:

~F1 = xze2y(zax + xzay + xaz)

~F2 = (xax + yay + zaz)/√x2 + y2 + z2

~F3 = 0.2ax − 0.6ay + 0.35az

~F4 = xy2z3(ax + 2.0ay + 3.0az)

PR 4.14: Considere-se que um cilindro longo e oco esteja cheio de ar sob pressao; e a tampa de uma

extremidade seja retirada rapidamente. A velocidade v do ar tem divergente; pois se colocarmos um

pequeno volume num ponto, vemos que a quantidade de ar que entra nao e igual a que sai pelo lado

oposto. Esta e a forma de ver se um campo tem divergente: colocar um pequeno volume, e ver se o

balanco lıquido e diferente de zero. ♦

PR 4.15: Comprovar o teorema da divergencia para ~D = 2xyax +x2ay e o paralelepıpedo formado pelos

planos x = 0, x = 1, y = 0, y = 2, z = 0 e z = 3.

Com integral de superfıcie:

∮~D · d~S =

∫ 3

0

∫ 2

0(2yax + ay) · (dydzax)

+∫ 3

0

∫ 1

0(x2ay) · (dxdzay) +

∫ 3

0

∫ 1

0(4xax + x2ay) · (dxdzay)

=∫ 3

0

∫ 2

02y dydz = 12

Com integral de volume:~∇ · ~D =

∂x(2xy) +

∂y(x2) = 2y

∫~∇ · ~Ddv =

∫ 3

0

∫ 2

0

∫ 1

02y dx dy dz = 12. ♦

PR 4.16: V = 1000rc2 V em coordenadas cilındricas.

1. Se a regiao 0.1 < rc < 0.3 m e vacuo e as superfıcies 0.1 e 0.3 sao condutoras, especifique a

densidade superficial de carga de cada condutor.

2. Qual e a carga ao longo de 1 m de comprimento da regiao onde ha vacuo?

3. Qual e a carga total ao longo de 1 m de comprimento, incluindo ambas as cargas superficiais?

PR 4.17: Um campo potencial eletrico e dado por V = x4 + y4 − 1 V.

1. Esboce as superfıcies equipotenciais V = 0 e V = 100 V. Estas sao duas superfıcies condutoras.

Page 27: Problemas de Eletromagnetismo.pdf

4. Lei de Gauss e Capacitancia 27

2. Determine a densidade volumetrica de carga no ponto P (2, 1, 3), situado no vacuo, entre as su-

perfıcies.

3. Encontre a densidade superficial de carga no ponto C(1,0,0) em um dos condutores.

PR 4.18: A superfıcie x + 2y2 + 4z3 = 100 e o contorno de um objeto condutor situado no vacuo. A

origem esta situada no interior do condutor e o ponto A(18,−5, 2) em sua superfıcie. Se | ~E| = 50 V/m

e o campo esta orientado para fora do condutor, determine ~E, ~D, e ρS neste ponto.

PR 4.19: Um campo potencial e dado por

V = 100 ln(x+ 1)2 + y2

(x− 1)2 + y2V.

Sabendo que o ponto P (2, 1, 1) esta na superfıcie do condutor e que ele esta situado no vacuo, determinar

o vetor unitario normal a superfıcie bem como a densidade superficial de carga no condutor.

PR 4.20: Uma carga puntual Q localiza-se a uma distancia h de um plano condutor. Determinar o

lugar geometrico dos pontos do condutor para os quais a densidade superficial de carga e 0.1Q/h2.

PR 4.21: Duas cargas puntuais de −100πµC estao localizadas em (2,−1, 0) e (2, 1, 0). A superfıcie

x = 0 e um plano condutor.

1. Determine a densidade superficial de carga na origem.

2. Determine ρS no ponto P (0, h, 0).

PR 4.22: Seja um sistema de coordenadas esfericas e uma densidade volumetrica de carga variando

linearmente com o raio, ρV = ρ0 r/a (ρ0 e a constantes). Determine a carga contida: (a) na esfera

r ≤ a; (b) no cone r ≤ a, 0 ≤ θ0, 1π; (c) na regiao r ≤ a, 0 ≤ θ ≤ π; 0 ≤ φ ≤ 0, 2π.

PR 4.23: Seja ρ = (10−6/r) C/m3 em coordenadas esfericas. Determine: (a) a densidade volumetrica

de carga na origem; (b) a carga contida em uma esfera de 1mm de raio centrada na origem; (c) a carga

contida na regiao 10 ≤ r ≤ 20mm; 28o ≤ θ ≤ 31o; 0, 9π ≤ φ ≤ 0, 96π.

PR 4.24: Calcular a funcao para a densidade volumetrica de cargas ρ quando o potencial eletrico seja

V = 8(x2 + y2). (R: ρ = −32ε0εr)

PR 4.25: Um capacitor, formado por duas placas paralelas, distantes 0,1mm, e cujas areas sao 200

cm2, esta ligado a uma fonte de tensao contınua de 12 V. Admitindo que o isolante entre as placas

seja plastico, com permissividade relativa igual a 3,0, calcular a carga acumulada em cada placa do

capacitor? (R: 63,7 nC)

Page 28: Problemas de Eletromagnetismo.pdf

4. Lei de Gauss e Capacitancia 28

PR 4.26: Um capacitor tem capacitancia de 0,1µF, e esta ligado a uma rede eletrica de 220 V. Qual a

carga contida neste capacitor? O que significa este valor ... e a carga total das duas placas? (R: 22µC)

PR 4.27: Um capacitor a ar de placas planas paralelas (considere campo uniforme entre as placas), tem

uma area de 1,51 m2 e separacao entre as placas de 10 mm, e ligado a uma fonte de tensao contınua de

240 Volts. A seguir todo o espaco livre entre as placas e preenchido com oleo (εr = 2, 5). Determinar o

que acontece com: a) a carga do capacitor; b) o campo eletrico entre as placas; c) a carga de polarizacao;

d) a inducao ou a densidade de fluxo eletrico entre as placas.

PR 4.28: Considerando um capacitor de placas paralelas contendo um dieletrico de permissividade ε,

de espessura d, area de placas A, e capacitancia C, demonstrar que ~D = ε ~E e a forma local da equacao

Q = CV .

Solucao: Partindo da equacao da capacitancia

C =Q

V= ε

A

d

e, considerando que V = E d e Q = D A, encontra-se

D A

E d= ε

A

d

que simplificando os termos d e A resulta:

ε =D

Eou ~D = ε ~E ♦

PR 4.29: Um capacitor de placas paralelas com area de 0.30 m2 e separacao de 5.5 mm, contem tres

dieletricos com interfaces normais ~D e ~E, como segue: εr1 = 3.0 , d1 = 1.0 mm; εr2 = 4.0 , d2 = 2.0

mm; εr3 = 6.0 , d3 = 2.5 mm. Encontre a sua capacitancia.

PR 4.30: Um capacitor a ar possui capacitancia de 10 pF. Quando ele e submerso em oleo seu valor de

capacitancia passa a 18 pF. Qual e a permissividade do oleo? Por que a capacitancia aumentou? (R:

15, 93× x10−12 F/m)

PR 4.31: Um capacitor a ar (tipo variavel) e carregado e depois desligado da fonte. A seguir ele e

imerso em um recipiente com oleo. A energia armazenada no capacitor aumenta ou diminui? Por que?

PR 4.32: Um capacitor a ar com capacitancia 0.005 microfarad e conectado a uma linha de corrente

contınua de 500 volts, desconectado, e entao imerso num oleo com constante dieletrica 2.5. Calcular a

energia armazenada no capacitor antes e depois da imersao no oleo.

Page 29: Problemas de Eletromagnetismo.pdf

4. Lei de Gauss e Capacitancia 29

PR 4.33: Submetendo um capacitor de 6 micro Farad a uma tensao de 50 V e uma frequencia de 1000

Hz, tem-se a perda de 0,5 W no mesmo. Qual e o seu fator de dissipacao tan δ?

Solucao:

tan δ =PC

V 2ωC=

0, 5502 2π1000 6E − 6

= 0, 0053

δ = 0, 303o ♦

PR 4.34: Um capacitor de 70 µF num desfibrilador esta com carga de 5.000 V, e a energia armazenada

no capacitor e

U = 1/2CV 2 = (1/2)(70× 10−6)(5.000)2 = 875J.

Aproximadamente 200J dessa energia sao enviados atraves da vıtima durante um pulso de cerca de 2,0

ms. A potencia do pulso e

P =W

t=

2002× 10−3

= 100kW,

muito maior que a potencia da propria bateria. ♦

PR 4.35: Tres cargas puntiformes de valor 1, 2 e 3 C estao situadas nos nos de um triangulo equilatero

com 1 m de lado. Calcular o trabalho realizado para deslocar estas cargas para um triangulo equilatero

com 1/2 m de lado.

Solucao: A energia potencial armazenada no sistema de tres cargas no triangulo equilatero maior e

We =12

3∑i=1

Qi Vi =12[5 + 8 + 9

4πε0] =

114πε0

J

A energia potencial armazenada no triangulo menor e o dobro deste valor, pois todos potenciais sao

a metade. O aumento da energia potencial e igual a 11/(4πε0)J.

Obviamente, este aumento de energia deve ser suprido por um agente externo. ♦

PR 4.36: Uma isolacao e composta de tres camadas, dispostas como um sanduıche, com: 2 mm de oleo

(εr = 2, 2, Vruptura = 80 kV/cm); 1,5 mm de papel (εr = 6, 0, Vruptura = 200 kV/cm); e, 2 mm de oleo

novamente. Qual a tensao de isolacao desta configuracao? O que acontece com a tensao admissıvel se

variarmos a espessura relativa entre os dois isolantes?

PR 4.37: Qual e a equacao da capacitancia de um cabo coaxial, de raio interno a e externo b? (R:

C = 2πεL/ ln(b/a))

PR 4.38: Qual e a capacitancia entre duas esferas concentricas, com raio interno a e raio externo b?

(R: C = 4πε[ab/(b− a)])

Page 30: Problemas de Eletromagnetismo.pdf

4. Lei de Gauss e Capacitancia 30

PR 4.39: Considere dois fios infinitos paralelos, com raio r0 e separados por uma distancia d entre seus

centros, e carregados uniformemente com densidade +ρL e −ρL. Determinar: a. A equacao do campo

eletrico; b. A equacao do potencial eletrico; c. A capacitancia unitaria (F/m) entre os dois condutores;

PR 4.40: Considere um fio infinito com raio r0, paralelo e separado por uma distancia d/2 entre seu

centro e o solo (V = 0 como uma superfıcie infinita), carregados uniformemente com densidade +ρL.

Determinar a capacitancia unitaria entre o fio e o solo.

PR 4.41: Dado que ~E1 = 2ax − 3ay + 5az V/m, chegando no plano 4x − 3y + z = 2, para a interface

dieletrica sem cargas livres, encontre ~D2, sabendo que εr1 = 3 e εr2 = 1.

PR 4.42: Demonstre a equacao da capacitancia de um capacitor constituıdo por dois cilindros concentricos

de raios a e b (a < b) e de altura h. O dieletrico entre os dois cilindros possui constante dieletrica

εr = 3, 0.

PR 4.43: O isolamento de um dispositivo eletromagnetico, como e o caso de um transformador, se con-

stitui numa das principais, senao a maior, preocupacao para o projetista. Em princıpio o transformador

opera em tensoes elevadas, pois e esta a caracterıstica que o torna util nos sistemas eletricos. A operacao

em tensoes sempre maiores tem feito do isolamento o principal topico de pesquisas e de desenvolvimento

de novos materiais isolantes. O isolamento de um transformador compreende:

1. Isolamento entre bobinas de uma mesma fase;

2. Isolamento das cabeceiras das bobinas;

3. Isolamento entre a bobina interna e o nucleo;

4. Isolamento entre a bobina externa e a caixa (tanque);

5. Isolamento entre fases;

6. Isolamento entre espiras entre camadas e entre discos;

7. Isolamento dos terminais e buchas passantes; e,

8. Isolamento das ligacoes, comutadores e paineis de ligacao.

Considere o problema do isolamento entre as bobinas de AT e BT com um cilindro isolante dividindo

o canal de dispersao de fluxo eletrico em dois canais de oleo e uma camada de papelao, formando um

‘sanduıche’. Sendo εoleo e εp as constantes dieletricas, respectivamente, do oleo e do papelao (cilindro) e

Vp a tensao de prova (ensaio), pode-se calcular os gradientes de potencial Eoleo e Ep no oleo e papelao,

Page 31: Problemas de Eletromagnetismo.pdf

4. Lei de Gauss e Capacitancia 31

admitindo-se que o campo, entre AT e BT seja uniforme. Fazendo a associacao de capacitores em serie,

pode-se demonstrar que:

Eoleo =Vp

εoleo

(a1

εoleo+ a2

εp+ a3

εoleo

)Ep =

Vp

εp(

a1εoleo

+ a2εp

+ a3εoleo

)Estas expressoes determinam os gradientes de potencial no oleo e no papelao. Estes gradientes nao

podem ser superiores aos gradientes de ruptura do oleo e/ou do papelao nas condicoes de sobretensoes de

ensaio, e consequentemente de operacao. Utiliza-se um coeficiente de seguranca que depende do projeto.

Tambem observa-se pelas expressoes que o gradiente sera maior no material com menor constante

dieletrica. ♦

PR 4.44: Considerando o problema do isolamento entre as bobinas de AT e BT de um transformador,

calcular a tensao de prova suportada para duas camadas de oleo com 2cm de espessura, e uma camada

de papelao com 1,5cm. Sabemos que as permissividades relativa do oleo e do papelao valem 2,2 e 6,0

respectivamente; e a rigidez dieletrica do oleo e 80 kV/cm e do papelao 200 kV/cm.

Solucao: Se aplicarmos uma tensao de prova de 100 kV, teremos os campos eletricos

Eoleo =100 000

2, 2(

0,22,2 + 0,15

6 + 0,22,2

)Eoleo = 219, 78 kV/cm

e, de forma semelhante

Epapel = 80, 586 kV/cm

Observa-se que, nesta configuracao, o oleo romperia primeiro. Assim, fazemos uma regra de tres para

calcular a tensao de prova, que sera 36,4 kV. ♦

Page 32: Problemas de Eletromagnetismo.pdf

5. CAMPO MAGNETICO

PR 5.1: Campo magnetico devido a um condutor longo retilıneo. Determine o campo magnetico ~H

num ponto P distante R metros de um condutor infinitamente longo, percorrido por uma corrente de

I amperes. A seguir, calcule o campo a uma distancia de 10 cm do condutor quando ele for percorrido

por uma corrente de 0,1A.

Solucao: Usando a lei de Biot-Savart, a contribuicao δ ~H para o campo no ponto P , devido ao

elemento de corrente iδ` e:

δ ~H =1

4πr3iδ~× ~r

A componente δ ~H tera sempre a mesma direcao e sentido, dados pela regra da mao direita. Considerando

o angulo θ, a componente de ~H torna-se

δH =1

4πr3I δ` r sen (90− θ)

Lembrando que arco = angulo x raio, podemos escrever a relacao entre δ` e δθ:

r δθ = δ` cos θ

ou seja

δ` = rδθ

cos θ= R

δθ

cos2 θSubstituindo δ` em δH:

δH =I cos θδθ

4πRAgora, integrando esta expressao desde θ = −π/2 ate θ = π/2 obtem-se o campo total H.

H =∫ π/2

−π/2

I

4πRcos θdθ

que resulta

H =I

2πRamperes/metro (5.1)

Entao, se R = 0, 1m e I = 0, 1A, a intensidade do campo e H = 0, 159A/m. ♦

PR 5.2: Dois fios retilıneos paralelos estao afastados de d = 40 cm, e sao percorridos por correntes

I1 = 100A e I2 = 60A, em sentidos opostos. Encontrar a distancia x de um ponto P ao primeiro

condutor, onde o campo magnetico total seja nulo.

Page 33: Problemas de Eletromagnetismo.pdf

5. Campo magnetico 33

6 I1

?

I2

rP

Solucao: Aplicando a eq. (5.1) para os dois condutores, e igualando a zero

I12πx

=I2

2π(x− d)

1002πx

=60

2π(x− 0, 4)

que resulta x = 1, 0 metro do primeiro condutor. ♦

PR 5.3: Uma espira circular, de raio R, e percorrida pela corrente I. Obter a equacao do campo

magnetico no centro da mesma.

Solucao: O produto vetorial do numerador da lei de Biot-Savart e

d~× ~r = R dφ R = R2 dφ

e

H =∫IR2 dφ

4π R3=

I

4πR

∫ 2π

0dφ =

I

2Rque e o campo no centro da espira circular. ♦

PR 5.4: Campo magnetico de uma espira circular - Neste exemplo, calcularemos o valor do campo

magnetico em um ponto generico P , situado no eixo de uma espira circular percorrida por uma corrente

constante I, conforme esquema da Fig.

Como d~ e ~r, da lei de Biot-Savart, sao perpendiculares entre si, o modulo do produto vetorial e

simplesmente d` r. Temos entao o modulo de d ~H:

dH =I d`

4π r2

Na integracao ao longo da espira, cada valor do componente radial dHR e anulado pelo seu oposto de

1800. Portanto, esses componentes nao entram no calculo de H para um ponto situado no eixo da

espira. Mas o componente axial

dHA = dH cosα = dHR

r=R I d`

4π r3

Page 34: Problemas de Eletromagnetismo.pdf

5. Campo magnetico 34

Integrando d` = Rdθ:

H =∫dHA =

R I

4π r3

∫d`

=R I

4π r32πR =

R2 I

2 r3

Mas como

r =√R2 + x2

temos, portanto

H = IR2

2 (R2 + x2)3/2(5.2)

Este resultado e fundamental para calcular o momento de dipolo magnetico. ♦

PR 5.5: As bobinas de Helmotz sao duas bobinas circulares coaxiais, onde seus raios R sao iguais

a distancia d entre elas, isto e R = d. Elas sao muito conhecidas pelo fato de que o campo magnetico

uniforme ao longo do seu eixo. Fazer um grafico com a amplitude do campo ao longo do eixo das

bobinas.

Solucao: Situando o eixo x, tal que x = 0 no ponto equidistante das duas bobinas, o modulo do

campo magnetico dado por (5.2) torna-se

H = NIR2

2 (R2 + (x− d/2)2)3/2+NI

R2

2 (R2 + (x+ d/2)2)3/2

como d = R escreve-se

H = NIR2

2 (R2 + (x−R/2)2)3/2+NI

R2

2 (R2 + (x+R/2)2)3/2

que resulta

H =NI

2R

(1 +(x−R/2)2

R2

)−1,5(1 +

(x+R/2)2

R2

)−1,5 (5.3)

e, para x = 0 o campo vale

H0 = 0, 7155NI

R

A eq. (5.3) pode ser expandida em serie, para x,

H(x) = H0(1 + c4x4 + c6x

6 + ...)

e para valores de x proximos de zero, a derivada dH/dx ∼= 0, e H ∼= H0, comprovando que a intensidade

de ~H permanece constante para um grande intervalo. ♦

Page 35: Problemas de Eletromagnetismo.pdf

5. Campo magnetico 35

PR 5.6: Campo de um solenoide finito. Verificar a validade da equacao do campo magnetico

H =NI√

4R2 + L2=

NI√D2 + L2

onde R e o raio do solenoide e L e o seu comprimento.

Resposta: Quando L → 0, H → NI / 2R; e, quando L → ∞, H → NI / L, que demonstra a

validade desta equacao. ♦

PR 5.7: Calcule a intensidade do campo magnetico no centro de uma espira quadrada com 32 cm de

lado, quando e percorrida por uma corrente de 10 A. (Resposta: 59 A/m)

PR 5.8: Um solenoide com 64 cm de comprimento e 2,54 cm de diametro, tem uma distribuicao uniforme

de 600 espiras de condutor de cobre isolado. Calcular a intensidade do campo magnetico no centro do

solenoide, quando e percorrido por uma corrente de 2 A. (Resposta: 1970 A/m)

PR 5.9: Desenhar diagramas mostrando o campo magnetico ao redor de dois condutores retilıneos

paralelos percorridos por corrente: (a) no mesmo sentido; e, (b) em sentidos opostos. Se os dois

condutores estiverem afastados de 7,62 cm e tiverem uma corrente de 1000 A no mesmo sentido, qual

a forca por metro de comprimento? (Resposta: 2,6246 N/m, de atracao.)

PR 5.10: Montou-se duas bobinas com 20 cm de diametro e 100 espiras de um condutor muito fino,

e dispostas coaxialmente a uma distancia de 10 cm uma da outra. Ligou-se uma fonte de corrente de

1 A, que passa nas duas bobinas em serie, de forma que os campos das duas bobinas sejam aditivos.

Plotar curvas mostrando a intensidade do campo magnetico ao longo do eixo das bobinas. (Demonstrar

qualquer formula utilizada).

PR 5.11: Tem-se dois fios infinitos separados por uma distancia A, nos quais passam as correntes I e

2I. Calcular o campo devido aos dois fios H, no ponto P , situado a uma distancia x do fio 1. (R:

H = I2π

A+xx(A−x)A/m)

PR 5.12: Uma barra condutora condutora infinita, de secao retangular, com expessura e e largura 2L,

possui uma corrente I fluindo transversalmente.

a) Calcule o modulo da densidade de corrente J (A/m2) na barra; (R: J = I2 e LA/m2)

b) Considere que o afastamento de um ponto ate a barra x << L, e calcule o campo magnetico H

criado pela barra no ponto P , cuja projecao sobre a mesma se situa na sua linha mediana.

PR 5.13: Tendo 3 fios infinitos percorridos por I, formando um triangulo equilatero com afastamento l

entre os centros, calcular o campo total no baricentro do triangulo, devido aos tres fios (R: Ht = 0)

PR 5.14: Calcule o campo magnetico num ponto P , situado no mesmo plano de uma espira circular de

raio R, que e percorrido corrente I, a uma distancia x do centro da mesma. (R: H = IR2

2√

(R2+x2)3A/m)

Page 36: Problemas de Eletromagnetismo.pdf

5. Campo magnetico 36

PR 5.15: Calcule o campo magnetico em um ponto P , que se situa no encontro das diagonais de uma

espira retangular de largura 2b e altura 2a, criado por uma corrente I. (R: H =√a2 + b2 I

πabA/m)

PR 5.16: Calcular a intensidade do campo magnetico no centro do sistema de coordenadas, se uma

bobina retangular com uma unica espira esta colocada no plano xy, passando 47.8 ampere nos lados

colocados em x = −17.3cm, x = −2.8cm, y = −12.5cm, e y = 3.2cm.

PR 5.17: Helmholtz, ligou duas bobinas circulares com mesmo raio, em serie, dispostas com os seus

eixos em comum, e observou que a intensidade do campo magnetico no eixo, e na metade entre as duas

bobinas possui gradiente nulo num trecho ao longo do eixo. Considerando N = 167 espiras, i = 13.4A,

calcular o campo quando o raio a = 0.6m. Observacoes: a. As duas bobinas estao associadas para

superposicao do campo. b. A distancia entre as duas bobinas e igual ao raio.

PR 5.18: Campo magnetico de um solenoide. Forma-se um campo magnetico ao redor de uma bobina de

fio de cobre, chamada solenoide, cujo comprimento e muito maior do que o seu raio, e consideraremos

o solenoide infinito. Usando argumentos de simetria e facil mostrar que sao nulos os campos entre os

fios e na parte externa do solenoide. No interior do solenoide o campo tem o sentido indicado pela regra

da mao direita.

Vamos usar a lei de Ampere para calcular o modulo de H no interior do solenoide. Vamos escolher

um retangulo como uma linha fechada amperiana, que e cortado pela linha que une os condutores de

um lado do solenoide. A corrente que atravessa o retangulo a amperiana selecionada) e igual a corrente

i, multiplicada pelo numero de espiras que atravessa a amperiana. Como o solenoide tem um numero

infinito de espiras (na pratica, um numero muito grande de espiras), a corrente que entra na lei de

Ampere e calculada em termos da densidade de espiras n espiras por metro de solenoide. Supondo que

temos n espiras por metro, num determinado comprimento L, a corrente que atravessa a amperiana

sera n L i. Assim, ∮LHd` = n L i

A integral fechada pode ser desdobrada, tomando H = 0 na regiao externa ao solenoide, de modo que

H = n i

E claro que para um solenoide finito, o valor do campo e menor. Mas, este valor e uma referencia, ou

um valor limite que nunca sera ultrapassado! ♦

PR 5.19: Campo de um toroide. No interior do toroide, aplicando a lei de ampere, e integrando na

linha amperiana circular de raio r, e isolando H, temos:

H =1

2π rN I (5.4)

onde N e o numero de espiras do toroide. ♦

Page 37: Problemas de Eletromagnetismo.pdf

5. Campo magnetico 37

PR 5.20: Campo magnetico dentro de um fio. Consideremos o fio condutor como um cilindro infinito,

de raio R, transportando uma corrente I0, com densidade uniforme.

Solucao: Vamos escolher uma linha amperiana circular, com raio r ≤ R. Como a corrente tem

densidade uniforme, temos a corrente I envolvida pela linha amperiana

I = I0r2

R2

Este resultado sera usado para calcular a indutancia interna de um fio. ♦

PR 5.21: Comparar, citando vantagens e desvantagens, de usar ou nao usar; ferramentas matemati-

cas mais avancadas (gradiente, divergente, rotacional, integral de linha, . . . ) para os problemas de

engenharia.

PR 5.22: Calcular a intensidade do vetor campo magnetico a 0,5 m do centro de um condutor de cobre,

com 25 mm2, imerso em oleo, que e percorrido por uma corrente de 100 A. A condutividade do cobre e

58 mm2/m/Ω. (R: 31,8 A/m)

PR 5.23: Considerando uma corrente i fluindo na direcao positiva ao longo do eixo z de −∞ a +∞,

calcular o campo magnetico num ponto P afastado de uma distancia r do condutor, usando: (a) a lei

de Biot-Savart; e, (b) a lei circuital de Ampere. (R: ~H = i2π r~uφ)

PR 5.24: Uma superfıcie infinita esta situada no plano z = 0, e e percorrida por uma corrente superficial

uniforme ~J = kay A/m. Escolher um percurso fechado e calcular o campo magnetico a uma distancia

z da superfıcie. (R: Hx = k/2. O campo para uma superfıcie de corrente nao varia com a distancia.)

PR 5.25: Uma densidade de corrente ~J = Jzaz, centrada no eixo z, passa atraves de um condutor de

raio R. Calcular o campo magnetico: (a) no interior do condutor; e, (b) externo ao condutor. (R:

Hφ = Jz r/2 e Hφ = Jz R2/(2r))

PR 5.26: Calcule o campo, dentro, no meio e fora de um cabo cilındrico oco de raio interno a e raio

externo b, percorrido por uma corrente I, de maneira uniforme. (R: a) dentro H = 0, b) no meio

H = I2πr

r2−a2

b2−a2 e c) fora H = I/(2πr))

Page 38: Problemas de Eletromagnetismo.pdf

6. FLUXO, INDUCAO E FORCA MAGNETICA

PR 6.1: A magnetizacao de saturacao do ferro e 1, 7×106A/m, e sua densidade e 7970 kg/m3. Sabendo

que o numero de Avogadro vale 6, 025 × 1026 kg-atomo, e a massa atomica relativa do ferro e 56,

calcular o momento magnetico de cada atomo de ferro, em Am2. Solucao: Um metro cubico tem massa

de 7970kg, e contem o seguinte numero de atomos:

N =7970× 6, 025× 1026

56= 8, 58× 1028 atomos

E o modulo do momento magnetico ~m por atomo vale

m =1, 7× 106

8, 58× 1028= 1, 98× 10−23 Am2 ♦

PR 6.2: Considere que o campo magnetico passa do ar para um meio contendo µr = 850, incidindo a

45o com a normal. Qual o angulo no meio 2?

Solucao: Isolando θ2 encontra-se

tg θ2 = tg 45o 8501

= 850

θ2 = 89, 93o

Observa-se uma grande variacao angular na passagem do ar para um meio ferromagnetico. ♦

PR 6.3: Considere um fio retilıneo, situado no ar, proximo a um material com permeabilidade µr. Fazer

um esboco das linhas de fluxo para: (a) µr = 1; (b) 1 < µr = 1 <<∞; e, (c) µr →∞.

Solucao: Quando µr = 1, as linhas sao circulares, em volta do fio. Quando 1 < µr = 1 << ∞, as

linhas tem o formato mostrado na figura ... Quando µr → ∞, a espessura do fluxo e a intensidade do

campo magnetico no ferro cai para zero. ♦

PR 6.4: A tensao de Hall do semicondutor com RH = 0, 63 × 10−3m3/As, espessura ` = 10−3m, e

corrente I = 100 mA e:

VH = 63 mV/T ♦

PR 6.5: Dado o campo vetorial ~F = x3ax + y3ay + z3az, calcular:

Page 39: Problemas de Eletromagnetismo.pdf

6. Fluxo, inducao e forca magnetica 39

1. O fluxo de ~F atraves de uma esfera de raio R centrada na origem.

2. A circulacao de ~F atraves de uma circunferencia de raio R centrada na origem e situada no plano

xy.

Solucao: Fazendo o divergente de ~F :

~∇ · ~F = 3 (x2 + y2 + z2) = 3 r2

De acordo com o teorema da divergencia podemos escrever:∫ ∫S~F · d~S =

∫ ∫ ∫V (~∇ · ~F ) dV

=∫ R0 (3r2) (4πr2dr) = 12πR5

5

Como ~∇× ~F = 0, a circulacao de ~F e nula para qualquer contorno fechado C. ♦

PR 6.6: Representar o campo vetorial ~A = 2yax − zay + 3xaz em coordenadas esfericas, especificando

Ar, Aθ, e ~Aφ; calcular o fluxo de ~A atraves de uma esfera de raio R = 5 centrada na origem; e, calcular

a circulacao de ~A atraves de uma circunferencia de raio R = 2 centrada na origem e localizada no plano

xy.

Solucao: As coordenadas esfericas de ~A sao:

Ar = 2rsen 2θ cosφsen φ

−r cos θsen θsen φ+ 3rsen θ cos θ cosφ

Aθ = 2rsen θ cos θsen φ cosφ− r cos2 θ cosφ− 3rsen 2θ cosφ

Aφ = −2rsen θsen 2φ− r cos θ cosφ

Como ~∇ · ~A = 0, implica∫ ∫

~A · d~S = 0

O rotacional ~∇× ~A = ax − 3ay − 2az, e o teorema de Stokes permite obter:∮~A · d~=

∫ ∫S(~∇× ~A)× d~S = −8π ♦

PR 6.7: Seja o campo vetorial: ~F = r~ur +R cosφ~uφ, calcule:

1. O fluxo de ~F atraves do cilindro fechado de altura 2h, raio R e centrado na origem.

2. A circulacao de ~F atraves da circunferencia C : r = 1, θ = π/3.

Page 40: Problemas de Eletromagnetismo.pdf

6. Fluxo, inducao e forca magnetica 40

Solucao: Fazendo uso da equacao:~∇ · ~F = 3− Rsen φ

rsen θe pelo teorema da divergencia teremos:∫ ∫

S

~F · d~S =∫ ∫ ∫ (

3− Rsen φrsen θ

)rcdrc dφ dz = 6πR2H

(Tambem pode-se fazer diretamente, sem usar o teorema da divergencia).

Agora, calculamos diretamente a integral de linha:∮~F · d~=

∫ 2π

0R cosφ (rsen θ)dφ = 0 ♦

PR 6.8: Dado o campo vetorial:

~A =(xz2 + xy2/2, x2y + yz2/2, y2z + x2z/2

)calcular:

1. O rotacional de ~A no ponto r = 5, θ = π/2 e φ = π/2;

2. O divergente de ~A no ponto rc = 5, φ = φ/2 e z = 1; e,

3. A circulacao de ~A atraves do retangulo com vertices: (2, 2, 1); (2,−2, 1); (−25, 2, 1); (−2,−2, 1).

Solucao: Usando diretamente a expressao do rotacional em coordenadas cartesianas:

~∇× ~A = yzax + zxay + xyaz =⇒ ~∇× ~A(0,5,0) = 0

Como rc = 5, φ = φ/2 e z = 1, e o mesmo que (x, y, z) = (0, 5, 1), calculamos o divergente:

~∇ · ~A =32(x2 + y2 + z2) =⇒ ~∇ · ~A(0,5,1) = 39

Pelo teorema de Stokes, a circulacao de ~A e a integral de superfıcie do rotacional de ~A:∮~A · d~=

∫ ∫(~∇× ~A) · d~S =

∫ 2

−2

∫ 2

−2xydxdy = 0 ♦

PR 6.9: Seja o campo vetorial: ~F = k~r, onde k e uma constante.

1. Obtenha o divergente e o rotacional de ~F .

2. Ache um campo escalar f tal que ~F = ~∇ f ; e se nao for possıvel, explicar a razao.

3. De uma expressao para um campo vetorial ~A tal que ~F = ~∇× ~A; e se nao for possıvel, explicar a

razao.

Page 41: Problemas de Eletromagnetismo.pdf

6. Fluxo, inducao e forca magnetica 41

Solucao: Aplicacao direta das expressoes de divergencia e rotacional em coordenadas cartesianas:

~∇ · ~F = 3k

~∇× ~F = 0

O incremento infinitesimal do campo escalar f e tal que:

df = ~∇ f · d~= ~F · d~r

df = k(x, y, z) · (dx, dy, dz) = k(xdx+ ydy + zdz)

e, fazendo a integral de df encontra-se f :

f =∫df =

∫k(xdx+ ydy + zdz)

f =∫kxdx+

∫kydy +

∫kzdz

f =k

2(x2 + y2 + z2) + cte

Nao e possıvel encontrar uma expressao para um campo vetorial ~A tal que ~F = ~∇ × ~A porque~∇× ~F = 0, e ~F e um campo com divergencia (pois o rotacional do divergente e sempre nulo). ♦

PR 6.10: Demonstrar que o campo vetorial ~F = yzax + zxay + xyaz e irrotacional (~∇ × ~F = 0) e

solenoidal (~∇ · ~F = 0); encontre um campo ~A tal que ~∇× ~A = ~F .

Solucao: De acordo com a expressao para o rotacional em coordenadas cartesianas teremos:(∂Az∂y −

∂Ay

∂z

)= yz;(

∂Ax∂z −

∂Az∂x

)= zx; e(

∂Ay

∂x −∂Ax∂y

)= xy.

Pode-se escrever as parcelas das derivadas parciais como:

∂Az

∂y= 2yz; e

∂Ay

∂z= yz

∂Ax

∂z= 2zx; e

∂Az

∂x= zx

∂Ay

∂x= 2xy; e

∂Ax

∂y= xy

Integrando estas equacoes obtemos:

~A =

(xz2 +

xy2

2

)ax +

(x2y +

yz2

2

)ay +

(y2z +

zx2

2

)az ♦

Page 42: Problemas de Eletromagnetismo.pdf

6. Fluxo, inducao e forca magnetica 42

PR 6.11: Comprovar o teorema da divergencia para o campo vetorial ~F = r~ur + φrsen θ~uφ, atraves da

superfıcie S definida pela esfera de raio R centrada na origem. Explicar o que acontece.

Solucao: Primeiro, fazemos a integral de superfıcie de ~F ao longo da superfıcie da esfera:∫ ∫~F · d~S =

∫ ∫(R~ur) cot(R2sen θdθdφ~ur) = 4πR3 (6.1)

Agora, num segundo passo, fazemo o divergente de ~F encontra-se

~∇ · ~F = 4; (6.2)

e, fazendo sua integracao no volume da esfera, obtem-se:∫ ∫ ∫V

(~∇ · ~F ) dV =163πR3

Como (6.1) nao e igual a (6.2), aparentemente ~F estaria violando o teorema da divergencia. Mas o

que acontece e que o campo ~F nao e uma funcao, porque ~F assume mais de um valor para um mesmo

angulo. Por exemplo, ~F (φ = 0) 6= ~F (φ = 2π), e portanto ~F nao e um campo vetorial. ♦

PR 6.12: Calcular a integral de linha∫~F · d~ desde (0, 0, 0) ate (1, 2, 4) se ~F = x2ax + yay +(xz− y)az:

1. Ao longo do segmento de reta que une os pontos dados; e,

2. Ao longo da curva x = t2; y = 2t; z = 4t3

Solucao: Substituindo ~F e d~= dxax + dyay + dzaz, a integral de linha de ~F ao longo de um caminho

qualquer C fica sendo ∫C

~F · d~=∫

Cx2dx+ ydy + (xz − y)dz

O segmento de reta que une os pontos (0, 0, 0) a (1, 2, 4) tem a seguinte equacao parametrica:

x = t, y = 2t; e z = 4t,

com respectivas derivadas:

dx = dt, dy = 2dt; e dz = 4dt.

Substituindo estes valores na integral de linha, ela resulta:∫C

~F · d~=∫

tt2dt+ (2t)2dt+ (4t2 − 2t)4dt

=∫ t=1

t=0

(17t2 − 4t

)dt =

113

Ao longo do caminho x = t2; y = 2t; z = 4t3, tem-se dx = 2tdt; dy = 2dt; e, dz = 12t2dt.

Substituindo na integral de linha, tem-se:∫C

~F · d~=∫ t=1

t=0

(48t7 + 2t5 − 24t3 + 4t

)dt =

73

Observa-se que os dois valores sao diferentes, isto e, a integral de linha por um caminho e diferente

da integral de linha pelo outro, significando que o campo vetorial ~F nao e um campo conservativo. ♦

Page 43: Problemas de Eletromagnetismo.pdf

6. Fluxo, inducao e forca magnetica 43

PR 6.13: Dado o campo vetorial ~F = r cosφ~ur +zsen φaz, expresso em coordenadas cilındricas, calcular

a circulacao de ~F atraves de uma circunferencia de raio R centrada no eixo z e em um plano paralelo

ao plano xy. ~F e um campo vetorial conservativo?

Solucao: ∮~F · d~=

∫ φ=2π

φ=0(r cosφ~ur + zsen φaz) · (Rdφ~uφ) = 0

Por esta integral, dirıamos que ~F e conservativo. Mas, de acordo com a equacao do rotacional (nao

nulo),~∇× ~F = z cosφ~ur − zsen φ~uφ + rsen φaz 6= ~0

afirma-se que ~F nao e um campo vetorial conservativo. ♦

PR 6.14: Calcular∫ ∫

S(~∇× ~F ) · d~S quando F = (x2 + y − 4; 3xy; 2xz + z2) e S e a superfıcie:

1. O hemisferio x2 + y2 + z2 = 16 sobre o plano xy; e,

2. A parabola z = 4− (x2 + y2) sobre o plano xy.

Solucao: Pelo teorema de Stokes: ∫ ∫S(~∇× ~F ) · d~S =

∮~F · d~.

Na circunferencia x2 + y2 = 16, e z = 0, tem-se∮~F · d~=

∮(x2 + y − 4)dx+ 3xydy

e, fazendo a troca de variaveis x = 4 cosφ, y = 4sen φ, dx = −4sen φdφ e dy = 4 cosφdφ obtem-se∮~F · d~=

∫ φ=2πφ=0 [(4 cosφ)2 + 4sen φ− 4)

(−4sen φdφ) + 3(4 cosφ)(4sen φ)4 cosφdφ]

∮~F · d~=

∫ φ=2πφ=0 [−64 cos2 φsen φ

−16sen 2φ+ 16sen φ+ 192 cos2 φsen φ]dφ = −16π

Agora, fazendo o mesmo procedimento para a circunferencia de integracao x2 + y2 = 4∮~F · d~= −4π ♦

PR 6.15: Usando os teoremas da divergencia e de Stokes (se for conveniente), calcular as seguintes

integrais:

1.∫ ∫

S(xdydz + ydzdx+ zdxdy onde S e uma esfera de raio R centrada na origem.

2. A circulacao de ~F , onde ~F = −3yax + 3xay + az, e o contorno C e a circunferencia de raio R = 1

localizada no plano z = 2y centrada no eixo z.

Page 44: Problemas de Eletromagnetismo.pdf

6. Fluxo, inducao e forca magnetica 44

Solucao: A primeira integral pode ser escrita na forma:∫ ∫S(xdydz + ydzdx+ zdxdy =

∫ ∫S~r · d~S

=∫ θ=2π

θ=0

∫ φ=2π

φ=0R(R2sen θdθdφ) = 4πR3

Para a segunda integral, usando o teorema de Stokes, com ~∇× ~F = 6az, encontra-se∮~F · d~= 6π ♦

PR 6.16: Um material com vetor magnetizacao ~M = 1, 7× 106A/m, tem 8, 1× 1028 atomos por metro

cubico. Qual e o momento de dipolo de cada atomo, em Am2?

PR 6.17: Qual e a corrente que seria necessaria para produzir um campo magnetico de 1, 7 × 106A/m

em uma barra de ferro cilındrica, de comprimento ` = 10cm e diametro D = 3cm, que esta envolvida

por um solenoide de 100 espiras?

PR 6.18: Calcular a inducao magnetica de uma peca de ferro que tem permeabilidade relativa igual a

2500, quando o campo magnetico e de 300A/m.

PR 6.19: As linhas de fluxo de ~B podem fazer dobras, ou mudar abruptamente sua direcao? Por que?

PR 6.20: Considerando ~H = 1400.0ax − 4500.0ay A/m, e o tensor de relutividade igual a

||ν|| =

η −ξξ η

=1||µ||

onde η = .700E+04 m/H, e ξ = .610E+03 m/H. Determinar o vetor de magnetizacao ~M .

PR 6.21: A curva de desmagnetizacao de uma amostra de ıma-permanente, apos imantado, e apresen-

tada na tabela.

Ponto Induc~ao (T) H desmag. (A/cm)

1 0.65 4.0

2 0.59 12.0

3 0.52 20.0

4 0.43 28.0

5 0.31 36.0

6 0.14 44.0

A densidade de fluxo usada no entreferro de um instrumento de medida e escolhida para 0,09 weber por

metro quadrado. O comprimento do entreferro unico e 0,12 cm, e a area do entreferro e 10 centımetros

quadrados. Considerando que o fluxo de dispersao seja igual ao fluxo util no entreferro, calcule as

dimensoes do ıma permanente com menor custo.

Page 45: Problemas de Eletromagnetismo.pdf

6. Fluxo, inducao e forca magnetica 45

PR 6.22: O campo vetorial ~F = 10xax poderia ser o vetor inducao magnetica ~B? Por que?

PR 6.23: Uma carga puntual de 1,2µC tem uma velocidade de 5ax − 3az m/s. Encontre a intensidade

da forca sobre ela no campo: (a) ~E = −18ax + 5ay − 10az kV/mm; (b) ~B = −0, 4ax + 0, 4ay + 0, 3az T;

e, (c) ~E e ~B juntos.

PR 6.24: Um fio retilıneo e percorrido por corrente I, perpendicularmente a um fluxo magnetico uni-

forme. Usando o princıpio da superposicao, mostre como ficam as linhas de fluxo proximo do condutor.

PR 6.25: Qual e a forca para movimentar ou fixar um fio condutor com secao 16mm2 e 2m de compri-

mento, passando 100A, quando o mesmo esta imerso num campo de 10000A/m? (Resp.: 2,51 N)

PR 6.26: Considere um contator, contendo material magnetico ideal (µr = ∞), que tem entreferro

`ar = 1mm, secao magnetica S = 4cm2, e uma bobina de N = 500 espiras percorrida por I = 2A. Qual

e a energia magnetica armazenada no contator? (Resp.: 126mJ)

PR 6.27: Demonstre a equacao da forca entre dois condutores paralelos.

PR 6.28: Uma fita de corrente de largura igual a 1,8cm conduz uma corrente de 25A na direcao positiva

do eixo x. Calcular a forca sobre a fita por unidade de comprimento se a inducao magnetica uniforme

for ~B = 0, 5ay T.

PR 6.29: Um momento magnetico de 1 Am2 equivale a um conjugado de 1 Nm quando a espira estiver

orientada perpendicular a inducao magnetica de 1 Tesla, ou seja

~τ = ~m× ~B

onde ~m = i~S. Demonstrar esta equacao e determinar o conjugado de uma espira de S = 20 × 8 cm2

numa inducao de 0,9 T e corrente i = 1 A.

PR 6.30: Enquanto que a unidade basica da eletricidade e a carga eletrica, a unidade basica do mag-

netismo e o dipolo magnetico. Entao, usando a expressao µ0p, alguns pesquisadores calculam o monopolo

magnetico ou carga magnetica, analogo a carga eletrica. Qual e a carga magnetica Q∗ de um dipolo

p = 3, 0 × 10−9 Am? Qual e a expressao do campo magnetico criado pelo dipolo num ponto qualquer

do espaco? (Resp.: ~H = Q∗~ur/(4πµ0 r2) A/m)

PR 6.31: O plano y+ 2z = 3 divide o espaco em duas partes: a regiao 1, contendo a origem do sistema

de coordenadas, possui µr1 = 5; e, a regiao 2, com µr2 = 20. Dado ~B1 = ax + 0, 5ay T, obter H1, H2 e

B2.

Page 46: Problemas de Eletromagnetismo.pdf

6. Fluxo, inducao e forca magnetica 46

PR 6.32: Dois condutores paralelos estao afastados 5,5 cm, a uma diferenca de tensao 80 V eficaz, na

frequencia de 60 Hz, e angulo de fase 0 graus. O primeiro condutor e percorrido por uma corrente

alternada, de 5,8 A, com angulo de fase 55,3 graus. O segundo condutor e percorrido por 8,9 A, com

angulo de fase 173,7 graus, tomando o mesmo sentido das correntes no circuito. Qual a intensidade e o

sentido das forcas atuando em cada condutor?

PR 6.33: Duas bobinas sao dispostas uma ao lado da outra, com seus eixos alinhados, colocadas no ar.

As duas bobinas sao percorridas por uma corrente I, de forma que a forca entre elas e de repulsao. Um

agente externo realiza forca em sentido contrario a forca magnetica.

Caso Bob1 Bob2 Fexterna

Inverte I2

Aproxima Bob2

Page 47: Problemas de Eletromagnetismo.pdf

7. INDUTANCIA E CIRCUITOS MAGNETICOS

PR 7.1: Qual e a tensao induzida num enrolamento com 1 cm2 de secao, quando a inducao magnetica

varia uniformemente de 2.5 T para 1.0 T em 1 ms?

PR 7.2: Como funciona um transformador ideal? Quais suas equacoes basicas?

PR 7.3: Deduzir a equacao da indutancia por metro de comprimento, para uma linha de transmissao

constituıda por um cabo coaxial com raio interno a e raio externo b.

PR 7.4: Deduzir a indutancia mutua entre um fio infinito e um circuito retangular.

PR 7.5: Duas linhas de transmissao a dois fios condutores paralelos, tem afastamento d entre os centros

dos quatro condutores. Qual a indutancia mutua entre as duas linhas? (Resp.: M = 2µ/(15πd))

PR 7.6: Uma espira de secao ~S = 0, 02m2 e atravessada pela inducao magnetica variavel no tempo, e

uniforme no espaco~B = 0, 5ax + 2, 0t2ay T

Qual e a equacao da tensao induzida na espira?

PR 7.7: Uma antena circular, de area A e resistencia R e perpendicular a um fluxo magnetico uniforme,

que varia desde zero ate B num intervalo de tempo ∆t. Qual e a expressao para a energia termica

dissipada na antena?

PR 7.8: Qual deve ser a relacao de espiras de um transformador de 220/5 Volts? Qual deve ser a secao

magnetica mınima se a densidade de fluxo de pico for 1,5T e a frequencia for 60Hz?

PR 7.9: Uma bobina retangular, de comprimento a e largura b e girada numa frequencia f numa inducao

uniforme ~B. Qual e a tensao induzida na bobina?

PR 7.10: Um pedaco de fio retilıneo esta colocado no eixo ox, desde x1 = 0 ate x2 = 3m. Qual e o

campo eletrico induzido no fio quando a densidade de fluxo e dada por ~B = 1, 4 cos 2π50t Tesla? Qual

a f.e.m. nos terminais do fio?

Page 48: Problemas de Eletromagnetismo.pdf

7. Indutancia e circuitos magneticos 48

PR 7.11: Qual deve ser a inducao magnetica ~B(t) num ponto onde o campo eletrico seja dado por

~E(t) = −y cosωtax + xsen ωtay V/m?

PR 7.12: Uma fonte chaveada tem um transformador operando a 50 kHz. Qual e a vantagem desta

frequencia, em relacao a uma fonte de 60 Hz?

PR 7.13: Uma maquina eletrica tem potencia P1 e um volume X1. Qual sera o volume X2 para a

potencia P2, mantendo as mesmas caracterısticas e proporcoes?

PR 7.14: Por que o acionamento de uma maquina eletrica tem por princıpio manter a relacao V/f

constante?

PR 7.15: Sendo ~B = 2 cos(3×105πt−πy)az Wb/m2, ache a fem V (t) induzida no sentido generico +aφ

ao longo do caminho fechado:

1. (0,0,0) a (1,0,0) a (1,1,0) a (0,1,0) a (0,0,0);

2. (0,0,0) a (1,0,0) a (1,2,0) a (0,2,0) a (0,0,0).

PR 7.16: Considere uma inducao ~B = 0, 4xaz Wb/m2 e dois trilhos paralelos posicionados em x = 0 e

x = a = 5cm. Um voltımetro liga os dois trilhos com um fio reto desde o ponto (0, 0) ate (0, a), e uma

barra deslizante paralela ao eixo y fecha o circuito. A posicao da barra e dada por x = 5, 4t−3t2 metros,

e se estende ate 2,4m alem do voltımetro.

1. Ache Vab quando a barra deslizante se encontra em x = 1m;

2. Esboce Vab em funcao do tempo.

PR 7.17: Um nucleo magnetico em forma de C, tem secao magnetica de 4cm2 e comprimento medio

do caminho magnetico de 12cm, e foi alimentado com tensao senoidal eficaz de 58,80 V, em 52,60 Hz,

e a corrente eficaz circulante e 25,70 A. O enrolamento possui 929 espiras, e entreferro e = 0, 91mm.

Pergunta-se: (a) O fluxo magnetico circulante de pico. (b) A inducao magnetica de pico no ar. (c) A

inducao magnetica de pico no ferro. (d) A forca magneto motriz de pico. (e) O campo magnetico de

pico no ar. (f) O campo magnetico de pico no ferro. (g) A permeabilidade magnetica relativa do ferro.

(h) A maxima energia magnetica armazenada. (i) A indutancia nos terminais do enrolamento.

PR 7.18: Uma bobina, com N = 220 espiras, foi ligada a rede com tensao V = 220 Volts eficaz, e

frequencia f = 60 Hz. Sabendo-se que esta bobina foi enrolada num nucleo de ferro tipo ‘C’, e separado

por um entreferro, com as seguintes caracterısticas: Secao magnetica = 4 cm2; Comprimento do ferro

= 13 cm; Permeabilidade relativa do ferro = 500; e, Entreferro = 0,5 mm. Determinar:

Page 49: Problemas de Eletromagnetismo.pdf

7. Indutancia e circuitos magneticos 49

• O fluxo magnetico;

• A relutancia equivalente do circuito;

• A forca magneto motriz; e,

• Corrente eficaz na bobina.

PR 7.19: Um disco de cobre com 150 mm de diametro esta girando com ω = 5 rpm, numa densidade

de fluxo ortogonal B = 0, 8 T. Ligou-se uma escova de carvao no centro e outra na periferia do disco, e

conectou-se a um voltımetro. Qual a leitura do instrumento? E um valor cc ou ca?

PR 7.20: Um condutor retilıneo com 0, 2 m de comprimento gira no plano xy com um terminal colocado

na origem e com velocidade angular ω = 200 rpm, numa inducao magnetica ~B = −0, 5az T. Qual a

f.e.m. induzida nos terminais do fio?

PR 7.21: O polo Norte de um ıma, que possui fluxo total de 0, 048Wb, e aproximado de uma bobina

de 500 espiras, num intervalo de 0,1s. Qual e a tensao induzida na bobina?

Solucao:

V = 5000, 0480, 1

= 240 V. ♦

PR 7.22: Considere uma barra condutora, que se move com velocidade ~v numa inducao magnetica ~B.

B = B0 cosωt

determinar a equacao da tensao induzida V . Solucao: Devido ao movimento, tem-se

V =∫

(~v × ~B) · d~= vbB0 cosωt

A f.e.m. induzida pela variacao de ~B no tempo e dada por Stokes, e vale

V = −∫

S

∂ ~B

∂t· d~S = ωabB0sen ωt

Somando as duas parcelas resulta

V = vbB0 cosωt+ ωabB0sen ωt

V = B0b√v2 + (ωa)2sen (ωt+ δ) V.

onde δ = tan−1(v/ωa) ♦

Page 50: Problemas de Eletromagnetismo.pdf

7. Indutancia e circuitos magneticos 50

PR 7.23: Uma leve moldura de fibra, retangular, de area A = 0,01 m2 funciona como carretel onde se

enrolam N = 42 espiras de fio de cobre esmaltado. Esse quadro e posto a girar com frequencia f = 60Hz

(r.p.s.) em um campo de inducao uniforme de intensidade B = 1, 00Wb/m2 (ou, o mesmo que 1,00

tesla). Determinar a lei de variacao da forca eletromotriz induzida, em funcao do tempo.

Solucao: A velocidade angular do quadro e aproximadamente:

ω = 2πf = 377 rad/s

resultando

V = 158 cos 377 t

sendo V em volts e t em segundos. ♦

PR 7.24: Esse experimento tem por objetivo evidenciar o princıpio de funcionamento dos transfor-

madores.

1. Construa duas bobinas toroidais, ambas com fio de cobre esmaltado #24; uma com 200 a 300

espiras e a outra com 100 a 150 espiras. A bobina grande leva, em serie, um soquete para lampada

incandescente comum. Ligue o cordao de forca na tomada eletrica domiciliar e observe o brilho

da lampada pequena. Faca essa observacao usando no soquete comum lampada de 40W, 60W,

100W e 200W. Nao demore demasiado nessas observacoes para evitar aquecimento exagerado na

bobina grande.

2. Coloque a bobina pequena dentro da bobina grande, ajustando bem. Aos terminais A e B da

bobina pequena ligue uma lampada para 6V (usadas em lanternas de 4 pilhas). Sempre e bom

dispor de soquete para tal lampada para facilitar as ligacoes e as trocas. Na falta dele, basta

soldar as extremidades dos fios A e B (devidamente lixadas) aos terminais da lampada.

3. Varie a posicao relativa entre as duas bobinas e verifique a tensao no terminais da bobina menor.

A relacao entre as duas bobinas e chamada de indutancia mutua.

4. Depois, no interior do conjunto passe um feixe de laminas de ferro-silıcio. Repita todo o proced-

imento anterior para essa nova situacao ... nucleo de ferro dentro das bobinas. Se as laminas

envolvem as bobinas e, com isso, ocorre maior concentracao das linhas de inducao, aumentando

a corrente induzida no secundario (bobina menor). Repita todo o procedimento anterior nova-

mente, observando o brilho da lampadinha. Troque a lampada (40W, 60W, 100W e 200W) em

cada observacao. ♦

PR 7.25: Considerando que a dimensao a = 5cm, calcular a potencia do transformador, quando f =

60Hz, B = 1T, e J = 4A/mm2. Considere um fator de empilhamento das laminas do nucleo igual a

0,95 e fator de ocupacao da janela pelo enrolamento de 0,35.

Page 51: Problemas de Eletromagnetismo.pdf

7. Indutancia e circuitos magneticos 51

Solucao: A tensao e a corrente nos enrolamentos e

V = 4, 44 f N B Am Cm

I = J Aw Cw / N

e a potencia do transformador S(VA) e

S = V I = 4, 44 Am Aw Cm Cw f B J

As secoes magnetica e de janela sao

Am = 2a× 2a = 4a2 = 100× 10−4 m2

Aw = 1, 5a× 3a = 4, 5a2 = 112, 5× 10−4 m2

Assim, temos a potencia

S = 4, 44× 100× 10−4 × 112, 5× 10−4

×0, 95× 0, 35× 60× 1× 4× 106 = 9965 VA ♦

PR 7.26: Considere um transformador de potencia de 50 kVA que tem uma relacao de tensao de

10000V/200V. A partir do kVA e da tensao, a corrente de carga do enrolamento da maior tensao e

50000VA/ 10000V = 5A, e do enrolamento de menor tensao e 50000VA / 200V = 250 A. Tal transfor-

mador com carga maxima com seu enrolamento conectado ao terminal sem ponto do outro enrolamento.

O circuito secundario de 10000 V pode ser carregado com no maximo 250 + 5 = 255 A sem que um dos

enrolamentos tenha uma corrente de sobrecarga. Sendo que a corrente da fonte e 250 A, o transformador

pode fornecer 10200 x 255 = 2550kVA. Isto pode ser tambem determinado pelo circuito do secundario:

10000 x 255 = 2550 kVA. Na verdade, a conexao como autotransformador teve um aumento na potencia

de 50 para 2550 kVA.

A explicacao para esse aumento e que o transformador original de 50 kVA nao tem conexao metalica

entre os dois enrolamentos, e entao o 50 kVA devem ser transmitidos atraves do transformador pelo

acoplamento magnetico. Mas com os enrolamentos conectados para fornecer a operacao de auto - trans-

formacao, existe uma conexao metalica entre os dois enrolamentos que transmite 2550 - 50 = 2500 kVA

sem ser transformada magneticamente. Assim, essa conexao metalica e que fornece o aumento de kVA.

Embora vantajoso a esse respeito, tal conexao destroi a propriedade de isolacao dos transformadores

convencionais, o significa que a auto - transformacao nao pode ser aplicada em qualquer transformador.

PR 7.27: Demonstrar a expressao

vL = LdiLdt

Page 52: Problemas de Eletromagnetismo.pdf

7. Indutancia e circuitos magneticos 52

PR 7.28: Demonstrar a equacao da energia armazenada num indutor.

PR 7.29: Considere uma bobina de indutancia igual a 1 Henry, que esta sendo percorrida pela corrente

I = 20A, e que a corrente seja interrompida a zero num intervaldo de 0,1µs. O que acontece com esta

bobina? O que poderia ser feito para amenizar este efeito?

PR 7.30: Indutancia entre dois fios paralelos - Considerem-se dois fios condutores paralelos, que sao

percorridos por correntes eletricas com sentidos opostos e mesma intensidade i1 = i2 = i. Nestas

condicoes, a intensidade do campo magnetico gerado por qualquer um dos dois condutores num ponto

P do plano (no plano definido pelos dois condutores) e dada pela expressao

H =i

2πd

em que d = d1 ou d = d2 define a distancia entre o condutor 1 ou 2 e o ponto. Pode-se calcular o fluxo

por unidade de comprimento, considerando µr = 1, e tendo em conta os sentidos opostos das correntes,

a integral da densidade do fluxo magnetico criado pelos dois fios conduz ao resultado

φ =µ0 i

∫ d+r/2

r/2(1r

+1

d− r) dr

φ =µ0 i

πln(

d

r)

e na qual se inscreve a indutancia por unidade de comprimento:

L =µ0

πln(

d

r) H/m

Este procedimento pode ser adotado para calcular a indutancia de outras estruturas de corrente eletrica.

PR 7.31: Indutancia de um cabo coaxial - Calcular a indutancia linear de cabo coaxial reto e infinito.

Resposta: O valor da indutancia por metro e

L =µ0

2ln(

rext

rint) H/m ♦

PR 7.32: Indutancia de uma bobina com nucleo toroidal - Nos exemplos considerados, calculou-se o

fluxo magnetico em superfıcies convencionais, como sejam, por exemplo, o plano definido pelos dois

condutores paralelos e o plano no qual se inscreve o diametro dos condutores concentricos caracterısticos

do cabo coaxial. No entanto, no caso das bobinas com N espiras e nucleo cilındrico ou toroidal, o fluxo

magnetico deve ser multiplicado por N espiras, para obter-se o fluxo concatenado.

Page 53: Problemas de Eletromagnetismo.pdf

7. Indutancia e circuitos magneticos 53

O campo magnetico e dado pela equacao 5.4. A inducao magnetica no nucleo e igual ao produto

do campo magnetico multiplicado pela permeabilidade magnetica do material do toroide. O fluxo

magnetico e o produto da secao Sm do toroide pela inducao. Assim, o fluxo concatenado vale

ψ = µ0 µr Sm1

2π rN2 I

e a indutancia resulta

L = µ0 µrSm

LmN2

Observacoes:

a) A indutancia e diretamente proporcional ao quadrado do numero de espiras;

b) A indutancia e diretamente proporcional a secao transversal, e inversamente proporcional ao

comprimento do caminho magnetico, que neste caso vale Lm = 2πr.

c) A indutancia e diretamente proporcional a permeabilidade magnetica.

d) Obviamente, se quizermos aumentar ou diminuir a indutancia, devemos variar estes parametros.

e) Normalmente, quando desejamos diminuir a corrente de uma bobina devemos aumentar a

indutancia.

f) Normalmente, quando aumenta a potencia de um equipamento, diminui a indutancia. ♦

PR 7.33: Considerando um indutor, com N espiras, envolvendo um circuito magnetico simples, com

um material de permeabilidade µ, de comprimento medio `m, area media da secao transversal Sm, e

indutancia L, demonstrar que ~B = µ ~H e a forma local da equacao N φ = L I.

Solucao: Partindo da equacao da indutancia

L = Nφ

I= µ N2Sm

`m

e, considerando que H = N I/`m; φ = B Sm, encontra-se

L = NB Sm

H `m/N= µ N2Sm

`m

que simplificando os termos Sm e `m resulta:

µ =B

Hou ~B = µ ~H ♦

PR 7.34: Ao longo do percurso da ciencia, definiram-se muitas unidades e leis. As definicoes basicas ou

fundamentais sao os materiais, e suas relacoes de causa-efeito. Quais os principais meios ou materiais

existentes na natureza e estudadas no eletromagnetismo, e suas relacoes constituintes?

Resposta: Os tres meios sao: condutores, dieletricos e magneticos, que sao resumidos na tabela 7.1.

Page 54: Problemas de Eletromagnetismo.pdf

7. Indutancia e circuitos magneticos 54

Tab. 7.1: Meios condutores, dieletricos, e magneticos.

Fonte Fluxo Relac~ao

Condutor V ou ~E I ou ~J R ou σ

Dieletrico V ou ~E Q ou ~D C ou ε

Mangetico I ou ~H φ ou ~B L ou µ

PR 7.35: Qual e o campo magnetico no interior de um material magnetico ideal? Por que?

PR 7.36: Um fluxo magnetico de 24000 linhas (maxwell) atravessa uma superfıcie de 6 cm2. Qual o

valor da densidade de fluxo ou inducao magnetica? (R:0,4 T)

1 maxwell = 1 linha de campo.

1 Gauss = 1 linha de campo / cm2.

1 Tesla = 1 T = 1 Wb/m2 = 104 Gauss.

PR 7.37: Quantas espiras sao necessarias para produzir uma inducao de 1 T num entreferro de 5 mm,

quando pelo condutor se deslocam 1 A? (R:3979 espiras)

PR 7.38: Considerando um circuito magnetico simples, pretende-se obter no entreferro de 2mm, uma

inducao magnetica de 0,95 Teslas. Quanto deve valer a corrente, se o numero de espiras for N = 1000?

(R: I ∼= 1, 6A)

PR 7.39: Um nucleo ferromagnetico toroidal, com permeabilidade relativa igual a 1000, com raio medio

de 10cm e secao de 3cm2, deve ser atravessado por uma densidade de fluxo magnetico igual a 1,2 T.

Qual a forca magnetomotriz necessaria?

PR 7.40: Qual a inducao magnetica num toroide de ferro, com raio medio de 10 cm e permeabilidade

magnetica relativa igual a 100, que e envolvido por 200 espiras, percorrido por 40 A. (R: 1,6 T)

PR 7.41: Um nucleo laminado possui comprimento medio de 12,6 cm, e permeabilidade relativa µr =

4890. Qual o numero de espiras necessario para produzir uma inducao de 1,3T, quando pelo condutor

se delocam 1,4A?

PR 7.42: Um nucleo ferromagnetico retangular simples, sem entreferro, possui tres lados com secao de

150 cm2, e comprimento total de 130 cm, e o quarto lado possui secao de 100 cm2 e comprimento de 45

cm. O numero de espiras e 200 e a permeabilidade relativa µr = 2500. Qual e o fluxo produzido por 1

A de corrente na bobina? (φ ∼= 0, 0048 Weber)

PR 7.43: Um nucleo ferromagnetico quadrado, tem um comprimento medio de 55 cm e uma area de

150 cm2. A bobina tem 200 espiras. Escolher o material e considerando sua curva B −H, calcule:

Page 55: Problemas de Eletromagnetismo.pdf

7. Indutancia e circuitos magneticos 55

1. Qual a corrente requerida para produzir 0,012 Wb de fluxo no nucleo?

2. Qual e a permeabilidade relativa do ferro neste nıvel de corrente?

3. Qual e a relutancia do ferro?

PR 7.44: Um circuito magnetico simples tem dois materiais de mesma secao colocados em serie. Qual

deve ser a relacao entre os comprimentos do caminho magnetico para que a energia total nos dois

materiais sejam iguais.

PR 7.45: Dado o circuito magnetico,que tem quatro entreferro com espessura e = 1mm, e secao

magnetica S = 4cm2. Escreva as equacoes de circulacao de campo e continuidade de fluxo. Cal-

cule os campos H nos entreferros quando NI = 1000A, e o fluxo total gerado pela bobina. (Resp.:

428,719A/m e 3,3E-4Wb)

PR 7.46: Calcular a indutancia mutua M12 entre dois aneis com seus eixos coindicentes, e afastados de

uma distancia c, sendo o primeiro de raio a e o segundo de raio b. Considere a >> b e n1 = n2 = 1.

1o. Calculo do campo magnetico

O campo magnetico no eixo de um anel e

H =I R2

2(R2 + x2)3/2

Nesse caso, H = H1, I = I1, R = a e ,x = c, de modo que

H1 =I1 a

2

2(a2 + c2)3/2

2o. Calculo do fluxo

Sendo a >> b, pode-se considerar o campo ~H da primeira bobina de raio a constante na segunda

de raio b. Assim, o fluxo da primeira bobina, que passa atraves da segunda e

φ12 =∫ ∫

S2

µ0 H1 dS = µ0 H1 S2

φ12 = µ0I1 a

2

2(a2 + c2)3/2π b2

φ12 =πµ0 I1 a

2 b2

2(a2 + c2)3/2

3o. Calculo da indutancia mutua

A indutancia mutua e igual ao fluxo concatenado (produto do fluxo pelo numero de espiras) dividido

pela corrente que originou este fluxo.

M12 = n2φ12

I1

Page 56: Problemas de Eletromagnetismo.pdf

7. Indutancia e circuitos magneticos 56

Como n2 = 1, a indutancia mutua e

M12 =πµ0 a

2 b2

2(a2 + c2)3/2

Este e o procedimento padrao para o calculo da indutancia: calculo do campo magnetico; calculo do

fluxo e calculo da indutancia mutua. Observa-se que, considerou-se a >> b, para calcular o valor da

indutancia mutua. Entretanto, se desejarmos um valor mais exato, precisaremos fazer uso de metodos

numericos, como o arquivo MUTUA.exe, disponıvel na pasta DEMAG. ♦

PR 7.47: Consideremos duas bobinas proximas, percorridas por correntes I1 e I2, variaveis no tempo.

Solucao: Dividindo o numero de linhas pela corrente, temos as indutancias:

L11 = 5 linhas / 5 A = 1,00 linha / ampere;

L12 = 2 linhas / 8 A = 0,25 linha / ampere;

L22 = 6 linhas / 8 A = 0,75 linha / ampere; e,

L21 = 3 linhas / 5 A = 0,60 linha / ampere. ♦

PR 7.48: Coeficiente de acoplamento. Vamos supor que as duas correntes I1 e I2 estejam ‘ligadas’ ao

mesmo tempo. Determinar os fluxos nas bobinas e o coeficiente de acoplamento k.

Solucao: Considerando positivo o fluxo o produzido pela corrente propria, temos

φ1 = 5− 2 = 3 linhas

φ2 = 6− 3 = 3 linhas

Observa-se que 2/5 das linhas se perdem em (a) e 4/6 se perdem em (b). Caso nenhuma linha se

dispersasse, o coeficiente de acoplamento seria k = 1. Mas, com a dispersao, temos a media geometrica

k =√

25× 4

6= 0, 516 ♦

PR 7.49: Escrever as equacoes das tensoes V1 e V2 e das correntes I1 e I2 das bobinas, que variam

senoidalmente com frequencia ω.

Solucao: Considerando positivo o fluxo produzido pela corrente propria, temos para correntes quais-

quer

V1 = ωL11I1 − ωL12I2

V2 = −ωL21I1 + ωL22I2

Com as correntes e indutancias dadas

V1 =dφ1

dt= ω(1× 5− 0, 25× 8) = 3ωV

V2 =dφ2

dt= ω(0, 75× 8− 0, 60× 5) = 3ωV

Page 57: Problemas de Eletromagnetismo.pdf

7. Indutancia e circuitos magneticos 57

Observa-se que, a relacao 1:1 nao significa transformador ideal, pois:

S1 = V1I1 = 3ω × 5 = 15ω

S2 = V2I2 = 3ω × 8 = 24ω ♦

PR 7.50: Duas bobinas chatas com eixos coinncidentes tem raios R1 e R2, estao afastadas uma da outra

da distancia d. Usando o programa MUTUA.EXE, disponıvel na pasta DEMAG, completar a tabela

com os valores da indutancia mutua e explicar o seu comportamento.

R1 (cm) R2 (cm) d L12

0,1 0,35 0,2 3,6677E-10 H1,0 3,5 2,0 3,6677E-9 H10 35 20 3,6677E-8 H100 350 200 3,6677E-7 H

Solucao: A indutancia mutua diminui proporcionalmente com as dimensoes, porque o fluxo tambem

diminui proporcionalmente com as dimensoes, para uma mesma corrente. Quando diminui as dimensoes

o campo magnetico aumenta proporcionalmente. Embora diminua o caminho magnetico e aumente o

campo 10 vezes, com a reducao de 10 vezes das dimensoes, a area diminui 100 vezes com o quadrado

do raio, e assim o fluxo diminui 10 vezes. ♦

PR 7.51: Um indutor com nucleo de ar, na forma de um solenoide de comprimento d = 3cm, area media

A = πr2 = 12cm2 e com N = 1000 voltas, tem uma indutancia

L = µ0N2A

d= 50 mH.

O perımetro medio de cada espira e 2πr = 10, 3cm, o que da um comprimento total ltot = 123 metros. Se

o fio e de cobre (resistividade ρ = 1, 8× 10−6Ωcm, de diametro D = 0, 25mm (area da secao transversal

S = πD2/4), entao a resistencia serie desse indutor e

rs = ρltot/S = 45Ω.

Para uma frequencia de 100 Hz, a reatancia e XL = 2πfL = 31, 4Ω, que e menor que a sua resistencia

interna. Por outro lado, para uma frequencia de 10 MHz, XL = 188MΩ >> rs (mesmo considerando

o efeito pelicular, que daria rs = 130Ω). Apesar disto, em certos casos, principalmente em circuitos

ressonantes, rs nao podera ser ignorada, mesmo que a frequencia seja alta. A frequencias mais altas e

necessario considerar a capacitancia parasita entre as espiras da bobina, cp, em paralelo com o indutor.

Page 58: Problemas de Eletromagnetismo.pdf

8. FASORES

PR 8.1: (Ulaby Ex 7.1) O campo eletrico de uma onda eletromagnetica e

E(z, t) = 10 cos(π × 107t+ π z/15 + π/6) V/m.

Determinar: (a) a direcao de propagacao da onda; (b) a frequencia da onda; (c) o comprimento de onda

λ; e, (d) a velocidade de propagacao ou de fase up.

Solucao:

(a) −z porque os dois coeficientes de t e z tem mesmo sinal (neste caso positivo).

(b) f =ω

2π=π × 107

2π= 5 MHz

(c)2πλ

15→ λ = 30 m

(d) up = λ f = 1, 5× 108 m/s ♦

PR 8.2: (Ulaby Ex 7.2) Uma onda eletromagnetica propaga-se na direcao +z num meio com constante

de atenuacao α = 0, 5 Np/m. Se a amplitude do campo eletrico e 100 V/m em z = 0, qual e a distancia

para que a amplitude seja atenuada para: (a) 10 V/m; (b) 1 V/m; e, (c) 1 µV/m?

Solucao:

(a) 100e−0,5z = 10 → z = 4, 6 m

(b) 100e−0,5z = 1 → z = 9, 2 m

(c) 100e−0,5z = 10−6 → z = 37 m ♦

PR 8.3: (Ulaby Ex 7.3) Representar as funcoes complexas z1 = (4− j3)2 e z2 = (4− j3)1/2 na forma

polar.

Solucao:

z1 = (4− j3)2 =[(42 + 32)1/2 6 − tan−1 3/4

]2= [5 6 − 36, 87]2 = 256 − 73, 7

z2 = (4− j3)1/2 =[(42 + 32)1/2 6 − tan−1 3/4

]1/2= [5 6 − 36, 87]1/2 =

√5 6 − 18, 4 ♦

PR 8.4: (Ulaby Ex 7.4) Demonstrar que√

2j = ±(1 + j).

Solucao:

ejπ/2 = 0 + jsen (π/2) = j√2j = [2ejπ/2]1/2 = ±

√2ejπ/4 = ±(1 + j). ♦

Page 59: Problemas de Eletromagnetismo.pdf

8. Fasores 59

PR 8.5: (Ulaby Ex 7.5) Um circuito RL serie (R = 400 Ω, L = 3mH e ω = 105rad/s) e conectado

a uma fonte de tensao v(t) = 150 cosωt Volts. Qual e: (a) o fasor de corrente Is; e, (b) a corrente

instantanea?

Solucao:

Is =Vs

R+ jωL=

150400 + j300

= 0, 36 −36, 9o A

i(t) = <[Is ejωt] = <[0, 3 e−j36,9oejωt] = 0, 3 cos(105t− 36, 9o) A ♦

PR 8.6: (Ulaby Ex 7.6) O fasor da tensao e Vs = j5 V. Encontre v(t).

Solucao:

v(t) = <[Vs ejωt] = <[5 ejπ/2 ejωt] = −5 sen ω t; V ♦

PR 8.7: Um predio e alimentado com tres fios vivos de 127 V (eficazes) e fases (vivos) R, S e T . A

diferenca de fase entre dois fases quaisquer e de 120 graus. Represente as tres tensoes no plano complexo

e mostre que a diferenca de potencial entre dois vivos quaisquer e DV cos(ωt+2π/3), onde DV = 311, 1

Volts (pico) ou 220 Volts eficazes.

Solucao: As tres fases estao defasadas de 120 eletricos, e formam uma estrela, que ligam o baricentro

ate cada ponto de um triangulo equilatero. Se este triangulo for cinscunscrito num cırculo de raio igual

a 127 Volts, podemos demonstrar facilmente que os seus lados correspondem a 220 Volts.♦

PR 8.8: Suponha que a tensao v(t) e a corrente i(t) ‘senoidais’ sejam v(t) = 110 cos 377t volts e

i(t) = 8, 39 cos(377t− π/6) amps. Comprovar que a equacao da potencia ativa P = V I cosφ e a media

de p = vi num perıodo.

Solucao: A potencia ativa P e a media das potencias instantaneas p(t) = v(t) i(t) num perıodo

P =1T

∫ T

0p(t)dt

=3772π

∫ 2π/377

0924 cos 377t cos(377t− π/6)dt

= 5, 53× 104∫ 2π/377

0cos 377t(cos 377t cosπ/6 + sen 377tsen π/6)dt

= 5, 53× 104∫ 2π/377

0(0, 866 cos2 377t+ 0, 217sen 754t)dt

= 400 watts ♦

PR 8.9: Mediu-se a tensao v(t) e a corrente i(t) de um dispositivo desconhecido, encontrando-se v =

220sen 377t Volts e i = 11 cos 377t Amperes. Determine o circuito equivalente do elemento desconhecido.

Solucao: Escrevendo-se os fasores de tensao e corrente, obtem-se a impedancia:

Z =2206 − 90

116 0= −j20 Ω

Como Z e puramente imaginario negativo, corresponde a uma capacitancia pura, onde:

C =1

ωXC=

1377 . 20

= 133 µFarads. ♦

Page 60: Problemas de Eletromagnetismo.pdf

8. Fasores 60

PR 8.10: Mediu-se a tensao v(t) e a corrente i(t) de um dispositivo desconhecido, encontrando-se

v = 220sen 377t Volts e i = −11 cos 377t Amperes. Determine o circuito equivalente do elemento

desconhecido.

Solucao: Escrevendo-se os fasores de tensao e corrente, obtem-se a impedancia:

Z =220 6 − 90

116 180= 206 − 270 = 206 90 = +j20 Ω

Como Z e puramente imaginario positivo, corresponde a uma indutancia pura, onde:

L =XL

ω=

20377

= 53 milihenrys. ♦

PR 8.11: Um motor eletrico alimentado com tensao alternada de 110 V consome uma corrente de 5 A

atrasada de 20 graus eletricos da tensao. Desenhar as formas de onda de v(t) e i(t) e calcular a potencia

media num perıodo. Qual e a equacao da potencia?

Solucao: Sabemos que a potencia e o produto de V e I eficazes pelo fator de potencia cos θ:

P = V I cos θ = 110× 5× cos 20 = 516, 83 Watts ♦

PR 8.12: Demonstrar que a soma de duas impedancias complexas em corrente alternada e identica ao

caso de resistencias em corrente contınua.

Solucao: Duas impedancias em serie tem a mesma corrente, e a tensao total e a soma das tensoes

de cada impedancia.

V = V1 + V2 + . . . = Z1 I + Z2 I + . . . = (Z1 + Z2 + . . .) I = Z I

Z = Z1 + Z2 + . . . ♦

PR 8.13: Demonstrar que a soma de duas admitancias complexas em corrente alternada e identica ao

caso de duas condutancias em corrente contınua.

Solucao: Duas admitancias em paralelo tem a mesma tensao, e a corrente total e a soma das correntes

de cada admitancia.

I = I1 + I2 + . . . = Y1 V + Y2 V + . . . = (Y1 + Y2 + . . .) V = Y V

Y = Y1 + Y2 + . . . ♦

PR 8.14: Dado um circuito serie Z = R + jωL Ω, encontre o seu circuito equivalente paralelo Y =

G− j/(ωL) Siemens?

PR 8.15: Dado um circuito paralelo Y = G + jωC Siemens, encontre o seu circuito equivalente serie

Z = R+ jωL− j/(ωC) Ω?

Page 61: Problemas de Eletromagnetismo.pdf

8. Fasores 61

Fig. 8.1: Circuito RLC, para calculo da impedancia de entrada usando a Carta de Smith.

PR 8.16: Determinar a impedancia de entrada ou equivalente do circuito eletrico de componentes con-

centrados apresentado na Fig. 8.1, na frequencia de 100 MHz. Sugestoes: normalizar as impedancias e

admitancias, usando a referencia base de 50 Ω, somar impedancias em serie e admitancias em paralelo.

Solucao: Inicialmente, calculamos as impedancias e admitancias normalizadas:

C1= 4,00E-11 Farads

YC1= 2,51E-02 Siemens = 1,256637061 p.u.

ZC1= -3,98E+01 Ohms = -0,795774715 p.u.

L1= 5,30E-08 Henrys

ZL1= 3,33E+01 Ohms = 0,666017643 p.u.

YL1= -3,00E-02 Siemens = -1,501461727 p.u.

C2= 1,38E-10 Farads

YC2= 8,67E-02 Siemens = 4,34E+00 p.u.

ZC2= -1,15E+01 Ohms = -2,31E-01 p.u.

C3= 3,60E-11 Farads

YC3= 2,26E-02 Siemens = 1,130973355 p.u.

ZC3= -4,42E+01 Ohms = -0,884194128 p.u.

No passo seguinte, partimos da carga para a fonte . . . :

ZA= 1 .+j. -0,795774715

YA= 0,612273363 .+j. 0,487231661

YB= 0,612273363 .+j. -1,014230066

ZB= 0,436234662 .+j. 0,722622175

ZC= 0,436234662 .+j. 4,92E-01

YC= 1,009036072 .+j. -1,137938575

YD= 1,009036072 .+j. -6,97E-03

ZD= 0,990997627 .+j. 0,006840703

ZIN= 49,54988134 .+j. 0,342035157

Correspondendo ao valor de 50 Ω. ♦

PR 8.17: Escreva a impedancia complexa para cada caso do arquivo Impedancia.xls.

Page 62: Problemas de Eletromagnetismo.pdf

9. CAMPOS DINAMICOS - LEI DE FARADAY

PR 9.1: Considerando a densidade de fluxo magnetico ~B = B0ektaz, situado na regiao cilındrica ρ < b.

Qual e a f.e.m. induzida e o campo eletrico em uma espira de raio ρ = a, onde a < b, no plano z = 0?

Solucao: A f.e.m. dada pela Lei de Faraday

f.e.m. = −∫d ~B

dt· d~S = −k B0 ektπ a2

e, devido a simetria do problema, a f.e.m. e o campo multiplicado pelo perımetro,

f.e.m. = 2πa Eφ

logo o campo eletrico induzido e:~E = −1

2k B0 ektρaφ ♦

PR 9.2: Hayt 10.1 Na Fig. 9.1, seja B(t) = 0, 2 cos 120πt T, assumindo um resistor perfeito de 250 Ω,

e desconsiderando o campo magnetico produzido pela corrente induzida I(t), determine: a) Vab(t); e,

b) I(t).

Solucao: A f.e.m. e dada pela Lei de Faraday

f.e.m. = Vba(t) = −dφ/dt = (120π)(1, 41× 10−2)sen 120πt = 5, 33sen 120πt V.

e, a corrente induzida e:

I(t) = Vba(t)/R =5, 33sen (120πt)

250= 21, 3sen (120πt) mA ♦

PR 9.3: Hayt 10.2 Sendo a inducao magnetica variavel no tempo ~B = (0, 5ax+0, 6ay−0, 3az) cos 5000t

T, e uma espira retangular com cantos em(2,3,0), (2,-3,0), (-2,3,0), e (-2,-3,0), determine a corrente

induzida se a resistencia total da espira e 400 kΩ.

Solucao: A f.e.m. e dada pela Lei de Faraday

f.e.m. = emf =∫~E · d~L = −dφ/dt =

∫ ∫superf.

~B · azdS

f.e.m. =d

dt(0, 3)(4)(6) cos 5000t = −7, 2(5000)sen 5000t V.

e, a corrente induzida e:

I(t) =f.e.m.

R=−7, 2(5000)sen 5000t

400× 103= −90sen 5000t mA ♦

Page 63: Problemas de Eletromagnetismo.pdf

9. Campos Dinamicos - Lei de Faraday 63

Fig. 9.1: Espira circular no campo magnetico.

PR 9.4: Hayt 10.3 Seja ~H = 300az cos(3× 108t− y) A/m no vacuo, determinar a f.e.m. desenvolvida

na espira retangular situada nos cantos (0,0,0), (1,0,0), (1,1,0), e (0,1,0).

Solucao: O fluxo magnetico e

φ =∫ 1

0

∫ 1

0300µ0 cos(3× 108t− y) dx dy = 300µ0sen (3× 108t− y)|10

φ = 300µ0sen (3× 108t− 1)− sen (3× 108t) Wb

A tensao induzida e

f.e.m. = −dφdt

= −300 (3× 108)(4π × 10−7) = cos(3× 108t− 1)− cos(3× 108t)

f.e.m. = −1, 13× 105[cos(3× 108t− 1)− cos(3× 108t)

]V ♦

PR 9.5: Hayt 10.5 A posicao da barra deslizante na Fig. 9.2 e dada por x = 5t + 2t3, e a separacao

das duas barras fixas (trilhos) e 20 cm. Considerando ~B = 0, 8x2az T, determine a leitura no voltımetro

no instante de tempo t = 0, 4 segundos.

Solucao: O fluxo magnetico e

φ =∫ 0,2

0

∫ x

00, 8(x′)2dx′dy =

0, 163

x3 =0, 16

3(5t+ 2t3)3 Wb

A tensao induzida e

f.e.m. = −dφdt

= 0, 16 (5t+ 2t3)2(5 + 6t2) = −4, 32 V ♦

PR 9.6: (Wentworth 5.1) Qual o tempo gasto para que uma densidade de carga decaia de um valor

qualquer para 1% do seu valor inicial, no material poliestireno?

Page 64: Problemas de Eletromagnetismo.pdf

9. Campos Dinamicos - Lei de Faraday 64

Fig. 9.2: Barra deslizante no campo magnetico.

Solucao: O poliestireno tem εr = 2, 56 e σ = 10−17 Siemens/metro.

ρv = ρ0e−(σ/ε)t

0, 01 ρ0 = ρ0 exp

(−10−17

2, 56× 8, 854× 10−12

)= ρ0 e−4,41×10−9

t = 10, 4× 106 segundos = 120 dias

PR 9.7: (Wentworth 5.5) A densidade de fluxo magnetico aumenta a taxa de 10 Wb/m2/s na direcao

z. Uma espira condutora quadrada com 10x10 cm2 possui uma resistencia distribuıda de 10 Ω. Deter-

mine a direcao (por meio de um desenho) e a intensidade da corrente induzida na espira condutora.

Solucao:d ~B

dt= 10

Wbm2 s

az

Vemf = −∫d ~B

dt· d~S = −

∫10

Wbm2 s

az · dx dy az

Vemf = −0, 1Wbs

V sWb

= −0, 1 V

I =0, 1 V10 Ω

= 10 mA

Sentido horario, quando visto do eixo +z.

Page 65: Problemas de Eletromagnetismo.pdf

9. Campos Dinamicos - Lei de Faraday 65

PR 9.8: (Wentworth 5.7) No ar, uma espira condutora estacionaria e posicionada proxima a uma

linha infinita de corrente como ilustrado na Fig.5.17. A corrente linear aumenta com o tempo pela

relacao i(t) = 110(A/s) t. Se a distancia a e igual a 4 cm, calcule a tensao VR na espira.

Solucao: Adotaremos i(t) = A t, onde A = 100 A/s. Agora podemos resolver

Vemf = − ∂

∂t

∫~B · d~S

Sabendo-se que Vemf = VR, devemos fazer a integracao no sentido horario, adotando a regra da mao

direita d~S = dy dz (−ax). Da linha de corrente de comprimento infinito, tem-se:

~B =µ0 i

2πρaφ =

µ0 A t

2π y(−ax)

E entao∂ ~B

∂t=µ0 A t

2π y(−ax)

VR = −∫µ0 A t

2π y(−ax) · dy dz (−ax) = −µ0 A

∫ 2a

a

dy

y

∫ a

0dz = −µ0 A a

2πln(2)

Substituindo-se os valores numericos, VR = −610 nV.

PR 9.9: (Wentworth 5.8) Considerando a Fig.5.2, suponha que a area de uma unica espira de um par

seja 100 cm2 e que a densidade de fluxo magnetico seja constante sobre a area das espiras, mas que se

modifique com o tempo como ~B = B0 e−χtaz, onde B0 = 4 mWb/m2 e χ = 0, 30 Np/s. Determine VR

em t = 1, 10, e 100 segundos.

Solucao:

Vemf = −N∫∂ ~B

∂t· d~S

∂ ~B

∂t= −χ B0 e−χtaz

Vemf = 2χ B0 e−χt S = 24× 10−6e−0,30 t

para t = 1 s, VR = 17, 8 µV para t = 10 s, VR = 1, 20 µV para t = 100 s, VR = 2, 25× 10−18Volts.

PR 9.10: (Wentworth 5.9) Um transformador e algumas vezes utilizado como um conversor de impe-

dancia, onde ela e dada por V/I. Determine a expressao para a impedancia Z1 vista pelo lado primario,

para um transformador que possui uma impedancia Z2 na carga do lado secundario.

Solucao: Sabendo-se que i2 = N1N2i1 , v2 = N2

N1v1, e Z1 = v1

i1, tem-se:

Z2 =v2v1

=N2N1v1

N1N2i1

=(N2

N1

)2

Z1

Z1 =(N1

N2

)2

Z2 ♦

Page 66: Problemas de Eletromagnetismo.pdf

9. Campos Dinamicos - Lei de Faraday 66

PR 9.11: (Wentworth 5.10) Um fio de cobre com 1,0 mm de diametro e modelado como uma espira

quadrada de lado 4,0 cm. Ela e colocada em um plano normal a um campo magnetico que aumenta

com o tempo pela expressao ~B = 1, 0 t az Teslas, onde t e dado em segundos. (a) Calcule a magnitude

da correne induzida e indique sua direcao, com um desenho. (b) Calcule a densidade de fluxo magnetico

no centro da espira resultante da corrente induzida, e compare com a densidade de fluxo magnetico

original responsavel por gerar a corrente induzida em t = 1, 0 segundos.

Solucao: A corrente induzida e Vemf dividida pela resistencia distribuıda da espira.

R =1σ

`

A=

1Ωm5, 8× 107

4× 0, 04mπ (0, 0005m)2

= 3, 5 mΩ

d ~B

dt= 1

Wbm2s

az

;

Vemf = −∫d ~B

dt· d~S = −1

Wbm2s

∫ 0,04

0dx

∫ 0,04

0dy = −1, 6 mV

Iind =1, 63.5

= 0, 46 A

Observamos que esta resposta nao depende do tempo.

(b) O campo no centro da espira, devido a um lado, e dado pela equacao:

~H =I ρ

[z

ρ2√z2 + ρ2

]2

(−az) = −2, 59azAm

~B = 4µ0~H = −13 µTaz.

Este valor e muito menor do que o valor original (fonte). ♦

PR 9.12: (Wentworth 5.11) O comprimento medio do nucleo de nıquel de um transformador e 16

cm e a area da secao transversal e igual a 1 cm2. Existem 30 espiras no lado primario e 45 espiras no

lado secundario. Se a corrente no primario e 1, 0 sen(20π × 106t) mA, (a) calcule a amplitude do fluxo

magnetico no nucleo na ausencia da bobina de saıda. (b) Considerando a presenca da bobina de saıda,

calcule i2.

Solucao:

φ =Vm

R=N1 I1

`µrµ0A

= 14× 10−9 Wb

i2 =N1

N2i1 =

23sen(20π × 106t) mA ♦

PR 9.13: Demonstre a equacao da tensao induzida Vrms = 4, 44 f N φpico.

PR 9.14: Demonstre a equacao da tensao de um indutor VL = L di/dt.

Page 67: Problemas de Eletromagnetismo.pdf

9. Campos Dinamicos - Lei de Faraday 67

PR 9.15: Usando uma tensao sinosoidal qualquer, com velocidade angular ω, demonstre a equacao da

impedancia de um indutor ZL = jωL.

PR 9.16: Um condutor perfeito une as extremidades a e b de um resistor de 100Ω, formando um circuito

fechado circular de raio R = 20cm. A densidade de fluxo magnetico atraves do circuito e dada por~B = 0, 4sen (120πt) az T, perpendicular ao plano do circuito. Despreze a indutancia da espira e calcule

em funcao do tempo: (a) a diferenca de potencial sobre o resistor; e, (b) a corrente no circuito.

PR 9.17: Uma inducao magnetica de 1,5 T, e frequencia de 60 Hz, esta incidindo numa chapa de cobre

com espessura de 15 mm. Qual e a inducao no outro lado? E qual e a defasagem em relacao a incidente?

PR 9.18: Qual deve ser a espessura de uma chapa de alumınio para reduzir 99% dos campos em uma

blindagem? Qual seria a espessura de uma chapa de ferro com µr = 500?

PR 9.19: Suponha que exista um campo oscilante na superfıcie superior de uma arruela metalica. Con-

siderando: Ri = 1.0cm, Re = 1.7cm, B0 = 1.0T, σ = 0.57E + 08(Ωm)−1, f = 64.80Hz, e = 1.70mm, e

µr = 1.60; calcule: (a) a espessura de efeito pelicular dos campos δ; (b) a potencia media dissipada por

efeito Joule na peca.

PR 9.20: Um disco de cobre, se encontra sob a acao da inducao ~B(r, t) = B0(r/R)senωt. Calcule:

(a) a densidade de corrente induzida J(r, t); (b) a corrente induzida I(t) no disco (B0 = 0, 5T, σ =

5, 8.107(Ωm)−1, R = 1cm, e = 1mm, f = 50Hz); (c) a potencia media dissipada no disco por efeito joule

com os dados numericos acima.

PR 9.21: Um condutor perfeito circular, de raio r = 20cm, colocado num fluxo magnetico perpendicular

uniforme (no espaco) e variavel no tempo, une as duas extremidades de um resistor de 100Ω. Sendo

B = 0, 4sen 120πWb/m2 e, considerando desprezıvel o fluxo produzido pela propria corrente da espira

(desprezando a indutancia propria da espira), determine: (a) Vab(t); e, b) I(t).

PR 9.22: Os lados de uma espira quadrada, situada no plano z = 0, estao localizados em x = 0, 6

y = 0, 6m. Ha um campo magnetico ~B = (0, 2ax − 0, 4ay + 0, 4az) cos 2000 t Wb/m2, nesta regiao. Se

a resistencia da espira e 1kΩ, qual e a corrente no sentido horario (quando observada da parte positiva

do eixo z) que se faz presente no circuito? Considere desprezıvel o fluxo o fluxo produzido pela propria

corrente da espira, isto e, despreze a indutancia propria da espira.

PR 9.23: Uma espira condutora situada no plano z = 0 e limitada por x = 0, x = 0, 2m, y = 0, 3m, em

t = 0. A espira se desloca no sentido +ax com uma velocidade uniforme de 6m/s, e possui um pequeno

resistor de 20Ω. Sabendo que existe um campo magnetico nao uniforme, porem constante no tempo

nesta regiao, representado por ~B = 2x2yaz Wb/m2, esboce a curva que indica a potencia dissipada em

R como funcao do tempo, para o intervalo 0 < t < 100ms.

Page 68: Problemas de Eletromagnetismo.pdf

9. Campos Dinamicos - Lei de Faraday 68

PR 9.24: A partir de qual frequencia o efeito pelicular deve ser levado em consideracao para um fio de

grafite (condutividade 0,12 S/m) de 1 mm de diametro?

PR 9.25: Para diminuir as perdas ohmicas em instalacoes de alta potencia e redes de transmissao de

energia eletrica, se utilizam cabos de cobre grossos. Se a frequencia e de 60 Hz, a partir de que valor,

aproximadamente, nao adianta aumentar o diametro do cabo?

PR 9.26: Uma aruela de cobre (σ=5,8E7 (Ωm)−1) com espessura de 1mm tem raio interno de 1cm e

raio externo de 3cm, esta situada no plano xy. Um fluxo magnetico variavel tem Bz = B0 cosωt, sendo

B0 = 0, 3T e f = 60Hz. Calcular a espessura de efeito pelicular e a potencia media dissipada na peca

por efeito Joule. (R:8,53mm e 103,88W)

PR 9.27: Um disco de cobre (σ=5,8E7 (Ωm)−1) com espessura de 1mm tem raio R = 1cm, esta situado

no plano xy. Um fluxo magnetico variavel tem Bz(r, t) = B0(r/R)sen ωt, sendo B0 = 0, 5T e f = 50Hz.

Calcular a espessura de efeito pelicular; a corrente induzida I(t); e a potencia media dissipada na peca

por efeito Joule. (R:0,832W)

PR 9.28: Um anel, considerado filiforme, possui secao S = 1mm2, raio R = 2cm, condutividade

σ=1,0E7 (Ωm)−1, esta situado no plano xy. Na superfıcie interna deste anel existe um fluxo magnetico

variavel dado por Bz(r, t) = B0(1−r/R)sen ωt. Calcular a corrente i(t) que ira circular no anel, quando

B0 =1T e f =400Hz. (R:-83,78cosωt A)

PR 9.29: Uma espira, considerada filiforme, possui secao S = 1 mm2, raio R = 2 cm, condutividade

σ=5,8E7 (Ωm)−1, esta situado no plano xy. Na superfıcie interna deste anel existe um fluxo magnetico

variavel dado por

Bz(r, t) = B0(r2/R2)(1− e−t/T ).

Calcular a energia dissipada na espira desde T = 0 ate T =0,01s, quando B0 =0,2T. (R:0,364E-3J)

PR 9.30: Estime a capacitancia, C, a indutancia, ls, e resistencias serie, rs, e paralelo, rp, de um

capacitor de laminas de alumınio ρ = 2, 8 × 10−6Ωcm) de w = 2cm de largura, t = 5µm de espessura,

` = 2m de comprimento separadas por um filme plastico (ε = 30pF/m, ρ = 1, 2×1018Ωcm) de espessura

d = 10µm. Note que a indutancia parasita depende de se os contatos forem soldados as laminas de Al

pelos extremos ou pelos lados (apos enrolado); calcule ls nos dois casos.

PR 9.31: Um material magnetico laminado tem as seguintes perdas na frequencia de 50 Hz: a) Histerese

(5 W/kg); e, b) Foucault (3 W/kg). Determinar as perdas na frequencia de 200 Hz.

Solucao: Considerando o modelo de perdas

P (W/kg) = WH f +KCB2m f2

Page 69: Problemas de Eletromagnetismo.pdf

9. Campos Dinamicos - Lei de Faraday 69

obtemos

WH = 5/50 = 0, 1 J/kg

KCB2m = 3/502 = 0, 066 Js/kg

e, substituindo para f = 200Hz

P = 0, 1× 200 + 0, 066× 2002 = 20 + 2640 = 2660 W/kg

Observa-se um crescimento muito grande da perda por Foucault, que certamente inviabilizaria o projeto.

PR 9.32: Considerando que a energia dissipada por histerese seja a equacao:

W = Wh +Wc = KSBβ +KCfB

2

onde KS = 3, 70W/kg (em 50,0 Hz) e o coeficiente de Steinmetz, β = 1, 600 e o expoente da inducao, e

KC = 0, 216E − 02Js/kg/T2 e o coeficiente de perdas por correntes induzidas de Foucault. Determinar

a potencia dissipada em 0,00378 m3 de material magnetico; quando esta operando em 68.10 Hz, com

inducao de pico igual a 0.8 T. A densidade do ferro e 7,652 kg/litro.

PR 9.33: Considerando a curva de histerese dada na tabela, determinar a densidade de energia magnetica

absorvida pelo material, em J/m3, para passar do ponto 1 ao ponto 8.

Ponto Induc~ao (T) H (A/cm)

1 .00 .00

2 .28 4.86

3 .73 9.60

4 .55 3.88

5 .37 -.58

6 .55 3.15

7 .83 7.07

8 1.10 9.82

PR 9.34: Uma bobina de um rele possui uma resistencia de 10Ω e uma indutancia de 50mH. Qual sera

o tempo necessario para atingir a corrente de atuacao de i = 0, 4A, quando este rele for ligado a uma

fonte CC de V = 5V?

Solucao: A soma das tensoes da indutancia L e da resistencia R e igual a tensao da fonte

Ldi

dt+Ri = V

Page 70: Problemas de Eletromagnetismo.pdf

9. Campos Dinamicos - Lei de Faraday 70

Resolvendo esta equacao diferencial obtem-se

i =V

R(1− e−

RL

t)

Substituindo-se os valores

0, 4 = 0, 5(

1− e−tLIGA0,005

)encontra-se o tempo tLIGA = 1, 11ms. Vemos, entao, que este sera o intervalo de tempo desde que

uma chave for ligada, ate o fechamento e/ou abertura dos contatos do rele. Entretanto, na pratica,

este tempo sera muito . . .muito bem diferente, pois existe a inercia mecanica, alem de que os valores

de resistencia, indutancia, e a corrente de atuacao (proporcional a forca de reacao da mola), nao sao

constantes. Mas, a solucao da equacao diferencial permite-nos fazer uma analise qualitativa. ♦

Page 71: Problemas de Eletromagnetismo.pdf

10. CORRENTE DE DESLOCAMENTO

PR 10.1: Explicar o funcionamento de um circuito ressonante LC.

Resposta: Um oscilador LC e composto por um indutor e um capacitor. Seu funcionamento se

baseia no armazenamento de energia em forma de diferenca de potencial e cargas eletricas no capacitor

e em forma de fluxo magnetico e corrente no indutor.

O capacitor, em um tempo igual a zero, oferece uma impedancia proxima a zero ohms, o que permite

fluir uma grande intensidade de corrente eletrica atraves do qual vai diminuindo ate que suas placas

tenham cargas eletricas positivas e negativas como permite o tamanho do mesmo e a permissividade

eletrica do isolante que tem entre as placas do capacitor.

Num instante o capacitor funciona como um isolante, ja que nao pode permitir a passagem de

corrente, e se cria um campo eletrico entre as duas placas, que cria a forca necessaria para manter

armazenadas as cargas eletricas positivas e negativas, em suas respectivas placas.

Por outra parte, num tempo igual a zero o indutor possui uma impedancia quase infinita, que nao

permite o fluxo de corrente atraves dele e, a medida que passa o tempo, a corrente comeca a fluir,

criando-se entao um campo magnetico proporcional a magnitude da mesma. Passado um tempo, o

indutor atua praticamente como um condutor eletrico, pelo que a sua impedancia tende a zero.

Por estar o condensador e o indutor em paralelo, a energia armazenada pelo campo eletrico do

capacitor (em formas de cargas eletrostaticas), e absorvida pelo indutor, que armazena em seu campo

magnetico, porem a continuacao e absorvida e armazenada pelo capacitor, para novamente ser absorvido

pelo indutor, e assim sucessivamente. Isto cria um vai e vem de corrente entre o capacitor e o indutor.

Este vai e vem constitui uma oscilacao eletromagnetica, no qual o campo eletrico e o magnetico sao

perpendiculares entre si, o que significa que nunca existe os dois ao mesmo tempo, ja que quando o

campo eletrico esta no capacitor existe campo magnetico no indutor, e vice-versa.

PR 10.2: Explique como se formam as ondas eletromagneticas.

Resposta: Ondas eletromagneticas sao ondas que se formam a partir da combinacao dos campos

magnetico e eletrico, que se propagam no espaco transportando energia. O conceito de onda eletro-

magnetica foi postulado pelo famoso fısico escoces James C. Maxwell. E dele o trabalho mais notavel

no campo do eletromagnetismo. Utilizando-se das leis experimentais de Coulomb, Faraday, Ampere e

tambem das suas proprias concepcoes, Maxwell construiu um conjunto de equacoes que resume os con-

hecimentos sobre o eletromagnetismo. Hoje conhecemos essas equacoes como as equacoes de Maxwell

e sabemos que foram elas que possibilitaram a existencia das ondas eletromagneticas. Essas equacoes

Page 72: Problemas de Eletromagnetismo.pdf

10. Corrente de Deslocamento 72

sao importantes para o estudo da eletricidade, assim como as leis de Newton sao importantes para a

mecanica.

Maxwell provou, atraves das suas equacoes, que o disturbio eletromagnetico, o qual e causado pela

superposicao do campo eletrico e campo magnetico, apresenta todas as caracterısticas ondulatorias e

que, sendo assim, a radiacao eletromagnetica tambem deveria sofrer os fenomenos da reflexao, refracao,

difracao e a interferencia, assim como acontece em uma onda. Foi por esse motivo que o disturbio

causado pelo campo eletrico e magnetico acabou por ser denominado de ondas eletromagneticas.

Os campos eletrico e magnetico que dao origem as ondas eletromagneticas se propagam perpen-

dicularmente um ao outro. E importante saber que, ao contrario das ondas mecanicas, a onda eletro-

magnetica nao necessita de um meio material para se propagar, pois o campo eletrico e o campo

magnetico podem ser estabelecidos na ausencia de materia, ou seja, no vacuo. Sendo assim, a radiacao

eletromagnetica pode se propagar no espaco vazio.

Um resultado muito importante obtido por James Maxwell foi o da velocidade com que as ondas

eletromagneticas se propagam. Utilizando suas equacoes e por meio de calculos, ele mostrou que no

vacuo, como tambem no ar, a velocidade de propagacao da radiacao eletromagnetica e igual a: c =

3, 0× 108 m/s.

Essa descoberta foi muito importante porque esse valor coincide com a velocidade da luz, fato esse que

levou Maxwell a suspeitar que a luz era uma onda eletromagnetica. Ja no seculo XIX, os fısicos sabiam

que a luz se tratava de um fenomeno ondulatorio, mas nao sabiam qual a natureza dela. Hoje ja se sabe

que a suspeita de Maxwell e verdadeira: a luz e uma onda eletromagnetica. A descoberta da natureza

da luz foi um fato muito importante, o qual possibilitou a unificacao da Otica e do Eletromagnetismo.

Como os fenomenos luminosos tem origem no eletromagnetismo, por consequencia a otica pode ser

considerada um ramo do eletromagnetismo e suas leis podem ser deduzidas a partir das equacoes de

Maxwell.

Maxwell morreu muito cedo e por isso nao viu suas ideias serem confirmadas. Foi somente no final

do seculo XIX que o fısico alemao H. Hertz conseguiu, em laboratorio, obter ondas eletromagneticas

com todas as propriedades e caracterısticas propostas por Maxwell. As experiencias que Hertz realizou

confirmaram as hipoteses elaboradas por Maxwell, confirmando, dessa forma, que a luz e uma onda

eletromagnetica.

PR 10.3: Deduzir a equacao diferencial da carga de um circuito RLC serie, alimentado com a tenao

v(t) = Vm cosωt, em regime permanente.

Resposta:

Ld2q

dt2+R

dq

dt+

1Cq = Vm cosωt

PR 10.4: Demonstrar a equacao da velocidade angular de ressonancia para o circuito RLC serie, e

desenhar o diagrama fasorial da corrente e das tensoes.

Page 73: Problemas de Eletromagnetismo.pdf

10. Corrente de Deslocamento 73

Resposta:

ω =1√LC

PR 10.5: (Halliday, Ex.1, pg.1070) Um capacitor de capacitancia C = 1, 0 µFarads, e carregado ate

atingir a tensao de 50 Volts. Desliga-se o gerador e liga-se uma bobina com indutancia L = 10 mH aos

terminais do capacitor, de modo a estabelecer-se oscilacoes no circuito LC. Qual sera a corrente maxima

na bobina, supondo-se o circuito com resistencia nula?

Resposta: De acordo com o princıpio da conservacao da energia, a energia maxima armazenada no

capacitor deve ser igual a energia maxima armazenada no indutor.

UE =12q2mC

=12L i2m = Um

im = V0

√C

L= 0, 50 A

PR 10.6: (Halliday, Fig.38-8, pg.1084) Fazer um histograma com oito estagios de um ciclo de oscilacao

de uma cavidade ressonante eletromagnetica cilındrica, representando as energias eletrica e magnetica

armazenadas. Os pontos e cruzes indicam as linhas de inducao magnetica ~B, e as linhas horizontais

representam ~E.

PR 10.7: Inicialmente, lembramos da equacao ∇×H = ~J , onde afirma que o campo ~H e produzido pela

densidade de corrente eletrica ~J . Agora, considere uma cavidade eletromagnetica cilındrica, como na

Fig. 38.10, Halliday, pg.1094, e explique como e possıvel ter campo magnetico no interior da cavidade

ressonante, sem a presenca de corrente eletrica no seu interior?

Resposta: As correntes de conducao estao nas paredes da cavidade, enquanto as correntes de deslo-

camento estao no interior da cavidade, na mesma direcao do campo eletrico.

PR 10.8: (Halliday P37-14) Somente 60 anos depois de Maxwell ter enunciando o conceito de corrente

de deslocamento, foi que, em 1929, M.R. Van Cauwenberghe conseguiu medir diretamente, pela primeira

vez, a corrente de deslocamento id entre as placas de um capacitor de placas paralelas, submetido a

uma diferenca de potencial alternado v(t) = Vmsen ωt. Ele usou placas circulares cujo raio efetivo era

de 40 cm e cuja capacitancia era 100 pF. A diferenca de potencial aplicada tinha um valor maximo de

174 kV na frequencia 50 Hz. Determinar: (a) Qual foi a corrente de deslocamento maxima obtida entre

as placas? (b) Por que foi escolhida uma diferenca de potencial tao elevada?

Solucao:

id = CdV

dt= 2πf C Vm cos(2πf t)

idmax = 2πf C Vm = 5, 47× 10−3 A

A corrente de deslocamento maxima e diretamente proporcional a maxima diferenca de potencial apli-

cada. Um valor grande de Vm produz um valor de corrente de deslocamento mais facilmente mensu-

ravavel do que com um Vm menor. ♦

Page 74: Problemas de Eletromagnetismo.pdf

10. Corrente de Deslocamento 74

PR 10.9: (Wentworth 5.22) Suponha que um campo vetorial qualquer seja ~A = 3x2yz3ax. Verifique

se o divergente do rotacional deste campo vetorial e igual a zero.

Solucao: Vamos fazer ∇ · (∇× ~A), iniciando pelo rotacional, que e o produto vetorial ...

∇× ~A =

∣∣∣∣∣∣∣∣∣ax ay az

∂/∂x ∂/∂y ∂/∂z

3x2yz3 0 0

∣∣∣∣∣∣∣∣∣ = 9x2yz2ay − 3x2z3az

. . . e agora, fazendo o seu divergente:

∇ · (∇× ~A) =∂

∂y(9x2yz2)− ∂

∂z(3x2z3) = 9x2z2 − 9x2z2 = 0

Comprovando que o divergente do rotacional de qualquer vetor e sempre nulo. ♦

PR 10.10: (Wentworth 5.23) Suponha que um campo vetorial qualquer seja ~A = ρ2 cosφaz. Verifique

se o divergente do rotacional deste campo vetorial e igual a zero.

Solucao: Vamos fazer ∇ · (∇× ~A), iniciando pelo rotacional, que e o produto vetorial ...

∇× ~A =1ρ

∂φ(ρ2 cosφ)aρ −

∂φ(ρ2 cosφ)aφ = −ρsen φaρ − 2ρ cosφaφ

. . . e agora, fazendo o seu divergente:

∇ · (∇× ~A) =1ρ

∂ρ[ρ(−ρsen φ)] +

∂φ(−2ρ cosφ)

= −1ρ2ρsen φ+ 2sen φ = 0

Comprovando que o divergente do rotacional e nulo. ♦

PR 10.11: (Wentworth 5.24) Duas placas planas paralelas com area de 60 cm2 estao separadas com

um dieletrico com espessura 2,0 mm, e caracterizado pela constante dieletrica εr = 9, 0.. Considerando

a tensao v(t) = 1, 0sen (2π × 103t) Volts aplicada nas placas, determinar a corrente de deslocamento.

Solucao:

C = ε0εrS

d= 239 pF

id = Cdv

dt= 239× 10−12 × 2π × 103 cos(2π × 103t) = 1, 5 cos(2π × 103t) µA ♦

PR 10.12: (Wentworth 5.26) Um cabo coaxial com 1,0 m de comprimento tem um condutor interno

com diametro 2,0 mm e externo com diametro 6,0 mm, que sao separados por um isolante com constante

dieletrica εr = 10, 2.. Considerando a tensao v(t) = 10 cos(6π × 106t) mV aplicada nos terminais do

cabo, determinar a corrente de deslocamento entre os condutores interior e exterior.

Solucao:

C = 2πε0εr`

ln(b/a)= 516, 27 pF

id = Cdv

dt= −97sen (6π × 106t) µA ♦

Page 75: Problemas de Eletromagnetismo.pdf

10. Corrente de Deslocamento 75

PR 10.13: (Ulaby Ex 6.5) Um mau condutor caracteriza-se pela condutividade σ = 100 S/m e per-

missividade ε = 4ε0. Qual e a frequencia em que a amplitude das densidades de corrente de conducao~Jc e deslocamento ~Jd sao iguais? Comentar o resultado.

Solucao:

| ~Jc| = σ| ~E|

| ~Jd| =∣∣∣∣∣ ∂ ~D∂t

∣∣∣∣∣ = |jωε ~E| = ωε| ~E|

ω =σ

ε= 2, 82× 1012 rad/s

f =ω

2π= 448 GHz

Assim, em baixas frequencias, normalmente se desconsidera a corrente de deslocamento. ♦

PR 10.14: (Hayt 10.10a) Demonstrar que a relacao da amplitude da densidade de corrente de conducao

pela amplitude da densidade de corrente de deslocamento e σ/ωε, quando o campo eletrico for E =

Em cosωt. Assumir µ = µ0.

Solucao: Inicialmente, D = εE = εEm cosωt. Entao a densidade de corrente de deslocamento e:

Jd =∂D

∂t= −ωεEmsen ωt

Em segundo lugar, calculamos a densidade de corrente de conducao:

Jc = σE = σEm cosωt

Usando estes resultados encontra-se:|Jc||Jd|

ωε♦

PR 10.15: (Hayt 10.11) Um capacitor formado por um cabo coaxial tem a = 1, 2 cm, b = 4 cm, e

material homogeneo com ε = 10−11 F/m, µ = 10−5 H/m, e σ = 10−5 S/m. Considerando o campo

eletrico ~E(ρ, t) = (106/ρ) cos(105t)aρ V/m, determinar: (a) A densidade de corrente de conducao; (b)

A corrente de conducao; (c) A densidade de corrente de deslocamento; (d) a corrente de deslocamento;

e, (e) o fator de qualidade do capacitor, que e a relacao |Id|/|Ic|.Solucao: (a) ~J = σ ~E = (10/ρ) cos(105t)aρ A/m2

(b) Ic =∫ ∫

~J · d~S = 8π cos(105t) A

(c) ~Jd =∂ ~D

∂t=∂ε ~E

∂t= −1

ρsen (105t) A/m2

(d) Id = 2πρ`Jd = −0, 8πsen (105t) A

(e)|Id||Ic|

=0, 88

= 0, 1 ♦

Page 76: Problemas de Eletromagnetismo.pdf

10. Corrente de Deslocamento 76

PR 10.16: (Hayt 10.12) Seja uma linha de transmissao coaxial com b/a = e2,5, µr = εr = 1, e a

intensidade do campo eletrico ~E = (200/ρ) cos(109t − 3, 336z)aρ V/m. Determinar: (a) A tensao Vab

entre os condutores; e, (b) a densidade de corrente de deslocamento.

Solucao:

(a) Vab = −∫ a

b

200ρ

cos(109t− 3, 33z) dρ = 500 cos(109t− 3, 33z) V

(b) ~Jd =∂ ~D

∂t=−200× 109 ε0

ρsen (109t− 3, 33z)aρ = −1, 77

ρsen (109t− 3, 33z)aρ A/m2 ♦

PR 10.17: Uma fonte de tensao V0sen ωt esta conectada entre duas esferas condutoras concentricas,

r = a e r = b, b > a, e a regiao entre elas esta preenchida com um material para o qual ε = ε0εr, µ = µ0

e σ = 0. Encontre a corrente total de deslocamento atraves do dieletrico e compare-a com a corrente

da fonte, determinada a partir da capacitancia e metodos de analise de circuitos.

PR 10.18: A densidade de corrente de deslocamento e dada por 2 cos(ωt−5z)ax µA/m2 em um material

para o qual σ = 0, ε = 4ε0 e µ = 5µ0.

1. Use a definicao da densidade de corrente de deslocamento para encontrar ~D e ~E.

2. Agora utilize a forma pontual da Lei de Faraday e uma integracao no tempo, para encontrar ~B e~H.

3. Finalmente, utilize a forma pontual da lei circuital de Ampere para achar a densidade de corrente

de deslocamento. Qual deve ser o valor de ω?

PR 10.19: Um material para o qual εr = 1, 5 e µr = 1 tem uma condutividade σ. Sendo E =

60 cos 105tax V/m, ache: (a) ~Jc; (b) ~Jd; e, (c) a condutividade para a qual sao iguais amplitudes

das densidades de corrente de deslocamento e de conducao.

PR 10.20: Demonstre e comente as equacoes de Maxwell no vacuo em termos de ~E e ~B.

Resposta: No vacuo, as equacoes de Maxwell sao:

∇× ~E = −1c

d ~B

dt

e

∇× ~B =1c

d ~E

dt

Estas equacoes definem uma onda eletromagnetica para os campos ~E e ~B situados a 90 e se deslocando

com velocidade c no espaco.

PR 10.21: Deduzir a equacao da onda TEM, com campo eletrico no eixo x.

Solucao: Vamos considerar uma Onda Eletromagnetica Transversal (TEM), onde ~E e ~H ficam num

plano perpendicular a direcao de propagacao. Tomamos a onda que se propaga segundo o eixo z, o

Page 77: Problemas de Eletromagnetismo.pdf

10. Corrente de Deslocamento 77

campo ~E tem somente componente na direcao de x e o campo ~H tem componente somente na direcao

de y. E, como o meio e nao condutor ~J = 0 a equacao de Maxwell

~∇× ~H =∂ ~D

∂t

se reduz a

−∂Hy

∂z= −ε∂Ex

∂t(1)

A equacao de Maxwell obtida da lei de Faraday e

~∇× ~E = −∂~B

∂t

e, procedendo como na equacao anterior temos:

∂Ex

∂z= −∂By

∂t

como B = µH, temos∂Ex

∂z= −µ∂Hy

∂t(2)

(1) relaciona a derivada de ~H em relacao a posicao com a derivada no tempo de ~E.

(2) relaciona a derivada de ~E em relacao a posicao com a derivada no tempo de ~H.

Derivando (1) em relacao ao tempo e (2) em relacao a posicao.

∂t[∂Hy

∂z] = −ε∂

2Ex

∂t2

e

− 1µ

∂2Ex

∂z2=

∂t[∂Hy

∂z]

Comparando as duas equacoes anteriores tem-se

∂2Ex

∂t2=

1µε

∂2Ex

∂z2

Esta equacao diferencial descreve a variacao da grandeza Ex (intensidade do campo eletrico) na posicao

e no tempo. E chamada equacao da onda em Ex ou Equacao de D’Alembert.

PR 10.22: Calcule a razao entre as amplitudes das densidades de corrente de conducao e de deslocamento

para o campo eletrico E = E0 cosωt V/m no:

1. Cobre, σ = 5, 8× 107(Ωm)−1, ε = ε0, ω = 1.000rad/s;

2. Agua destilada, σ = 2× 10−4(Ωm)−1, εr = 80, ω = 1.000rad/s;

3. Polestireno, σ = 2× 10−16(Ωm)−1, εr = 2, 53, ω = 1.000rad/s.

Page 78: Problemas de Eletromagnetismo.pdf

10. Corrente de Deslocamento 78

PR 10.23: MIT Open Course1 Problem 1: The Displacement Current. What does the displacement

current term added to Amperes Law mean in practice? Discuss this in terms of an open surface and

the contour bounding that surface. Apply this meaning to the classic problem of a charging capacitor.

PR 10.24: MIT Open Course Problem 2: Comparing the Displacement Current to the Conduction

Current in a Wire. A wire with a circular cross-sectional area of 40 square millimeters carries a current

of 30 A. The resistivity of the wire is 2 X 10-8 Ohm-meter. What is the uniform electric field in the

wire? If the current changes at a rate of 6000 Amps/second, what rate is the electric field changing?

What is the displacement current density in the material? What is the magnetic field 5 centimeters from

the center of the wire? Note that you must include both the displacement current and the conduction

current in this calculation. Is the contribution from the displacement current in the case important?

1 http://ocw.mit.edu/courses/physics/8-02sc-physics-ii-electricity-and-magnetism-fall-2010/the-displacement-current-

and-maxwells-equations/

Page 79: Problemas de Eletromagnetismo.pdf

11. ONDAS ELETROMAGNETICAS

PR 11.1: (Ulaby Ex 7.16) Converter os valores seguintes de ganho de potencia G para o numero

natural em decibeis:

(a) G = 2, 3 → G(dB) = 10 log 2, 3 = 3, 6 dB

(b) G = 4× 103 → G(dB) = 10 log 4 + 10 log 103 = 6 + 30 = 36 dB

(c) G = 3× 10−2 → G(dB) = 10 log 3 + 10 log 10−2 = 4, 8− 20 = −15, 2 dB ♦

PR 11.2: (Ulaby Ex 7.17) Converter os valores seguintes de ganho natural em decibeis para o ganho

de potencia G e ganho de tensao g.

(a) G(dB) = 23 dB → 10 logG = 23 → G = 199, 53 → g =√G = 14, 13

(a) G(dB) = −14 dB → 10 logG = −14 → G = 0, 04 → g =√G = 0, 2

(a) G(dB) = −3, 6 dB → 10 logG = −3, 6 → G = 0, 436 → g =√G = 0, 66 ♦

PR 11.3: (Ulaby Ex 7.7) Uma onda plana uniforme com f = 10 MHz, esta situada num meio com

µ = µ0 e εr = 9. Obter: (a) a velocidade de fase; (b) o numero de onda ou a constante de fase; (c) o

comprimento de onda; e, (d) a impedancia intrınseca do meio.

Solucao:

(a) up =1√µε

=c√εr

= 108 m/s

(b) k = β =ω

up= 0, 2π rad/m

(c) λ =2πβ

= 10 m

(d) η =√µ

ε= 125, 67 Ω ♦

PR 11.4: (Ulaby Ex 7.8) O fasor do campo eletrico de uma onda plana uniforme num meio sem

perdas, com impedancia intrınseca de 188,5 Ω e ~Es = 10aze−j4πy mV/m. Determinar (a) o fasor de

campo magnetico associado, (b) a expressao para o campo ~E instantaneo, considerando µ = µ0.

Solucao: ~E se propaga na direcao az e esta polarizado no eixo ay. Assim, ~Hs = az × ~Es = axE.

Logo,

~Hs = ax10× 10−3

188.5e−j4πy = 53axe−j4πy µA/m

~E(z, t) = az10 cos(6π × 108t− 4πy) mV/m ♦

Page 80: Problemas de Eletromagnetismo.pdf

11. Ondas eletromagneticas 80

PR 11.5: (Ulaby Ex 7.9) Um meio com impedancia intrınseca η = 100 Ω tem campo magnetico~Hs = (10ay + 20az)e−j4x mA/m. Qual e o fasor do campo eletrico associado?

Solucao:~Es = −ηaz × ~Hs = −100[ax × (10ay + 20az)]e−j4x × 10−3

~Es = (−az + 2ay)e−j4x V/m. ♦

PR 11.6: (Ulaby Ex 7.10) Um meio com impedancia intrınseca η = 100 Ω tem campo magnetico~Hs = ay(10e−j3x − 20e+j3x) mA/m. Qual e o fasor do campo eletrico associado?

Solucao: ~Hs1 = 10aye−j3x mA/m. ~Hs2 = −20aye+j3x mA/m.

~Es1 = −ηaz × ~Hs1 = −100(ax × 10ay)e−j3x = −aze−j3x V/m.

~Es2 = −ηaz × ~Hs2 = +100(ax × 20ay)ej3x = −2azej3x V/m.

~Es = ~Es1 + ~Es2 = −az(e−j3x + 2ej3x) V/m. ♦

PR 11.7: (Ulaby Ex 7.11) O campo eletrico de uma onda plana e

~E(z, t) = ax3 cos(ωt− kz) + ay4 cos(ωt− kz) V/m

Determine: (a) o tipo de polarizacao; (b) o modulo de ~E; e, (c) o angulo de inclinacao.

Solucao: (a) Como as componentes de x e y estao em fase, a polarizacao tem que ser linear. Isto e

comprovado, obtendo-se χ = 0. Para a expressao dada, δx = δy = 0. Entao, δ = δy − δx = 0.

ψ0 = tan−1(ay

ax

)= tan−1

(43

)= 53, 1

sen 2χ = (sen 2ψ0)sen δ = 0 → χ = 0

(b) | ~E| = [E2x + E2

y ]1/2 = 5 cos(ωt− kz) V/m.

(c) Da parte (a), ψ0 = 53, 1. ♦

PR 11.8: (Ulaby Ex 7.13) Os parametros constitutivos do cobre sao µ = µ0 = 4π × 10−7 H/m, ε =

ε0 = (1/36π)× 10−9 F/m, e σ = 5, 8× 107 S/m. Considerando que estes parametros sao independentes

da frequencia, qual e a faixa do espectro eletromagnetico onde o cobre pode ser considerado um bom

condutor?

Solucao: Num bom condutorε′′

ε′=

σ

ωε> 100

f <σ

200πε= 1, 04× 1016 Hz ♦

Page 81: Problemas de Eletromagnetismo.pdf

11. Ondas eletromagneticas 81

PR 11.9: (Ulaby Ex 7.14) A partir de qual frequencia um solo seco, com εr = 3, µr = 1, e σ = 10−4

S/m, pode ser considerado como um meio dieletrico de baixa perda?

Solucao: Num dieletrico de baixa perda

ε′′

ε′=

σ

ωε< 0, 01 → f > 60 MHz ♦

PR 11.10: (Ulaby Ex 7.15) Qual e a amplitude percentual do campo ~E para uma onda que se propaga

uma distancia de 3δs, sabendo-se que o meio tem espessura de efeito skin ou pelicular δs?

Solucao: Sabendo-se que δs = 1/α, pode-se escrever:

|E(z)| = |E0|e−αz = |E0|e−z/δs

Para o ponto z = 3δs:|E(z = 3δs)||E0|

= e−3 = 0, 05 = 5% ♦

PR 11.11: (Hayt 11.6) O campo eletrico ~E(z, t) = (25ax − 30ay) cos(ωt− 50z) V/m esta presente no

vacuo. Determinar: (a) A velocidade angular; (b) A densidade de corrente de deslocamento; (c) O fluxo

magnetico passando atraves de um retangulo definido por 0 < x < 1, y = 0, 0 < z < 1, em t = 0.

Solucao: (a) ω = up β = (3× 108) (50) = 15, 0× 109 rad/s

(b) ~Jd(z, t) =∂ ~D

∂t= −ε0ω(25ax − 30ay) cos(ωt− 50z)

= (−3, 32ax + 3, 98ay)sen (1, 5× 1010t− 50z) A/m2

Usando a impedancia intrınseca, podemos obter o campo magnetico:

~H(z, t) =(

25η0ay +

30η0ax

)cos(ωt− 50z) A/m

~B(z, t) = (1/c)(25ay + 30ax) cos(ωt− 50z) Wb/m2

φ =∫ 1

0

∫ 1

0

~B · aydxdz = −0, 44 nWb ♦

PR 11.12: A principal caracterıstica de uma onda eletromagnetica e a capacidade de transportar energia

entre dois pontos. A quantidade dessa energia por unidade de tempo e por unidade de area e chamada

de vetor de Poynting ~S = ~E × ~H. O campo eletrico de uma onda eletromagnetica plana e dado pela

expressao:~E = E0 cosω (

√εµz − t) ax + E0sen ω (

√εµz − t) ay V/m

em que E0 e uma constante. Encontre o campo magnetico correspondente e o vetor de Poynting.

PR 11.13: Uma onda plana uniforme esta se propagando no polietileno (ver tabelas com suas pro-

priedades), com uma frequencia de 9,4GHz. Se a amplitude do campo magnetico for 0,007 A/m e o

material for considerado sem perdas, ache: (a) a velocidade de propagacao; (b) o comprimento de onda;

(c) a constante de fase; (d) a impedancia intrınseca; e (e) a amplitude do campo eletrico.

Page 82: Problemas de Eletromagnetismo.pdf

11. Ondas eletromagneticas 82

PR 11.14: Dado ~E = E0sen (ωt− βz) ay V/m no espaco livre, utilize as equacoes de Maxwell para

encontrar as equacoes dos campos vetoriais ~D, ~B e ~H e tracar o grafico dos campos eletrico e magnetico

em funcao de z, nos instantes t = 0 e t = T/12.

PR 11.15: Dado o campo magnetico ~H = H0e(ωt−βz)j A/m no espaco livre, determine ~E.

PR 11.16: No espaco livre:~D = D0sen (ωt+ βz)ax C/m2.

Use as equacoes de Maxwell para mostrar que

B = −D0β

ωε0sen (ωt− βz)ay T.

Esboce o grafico dos campos para t = 0 ao longo de z, supondo que D0 e β sao maiores que zero.

PR 11.17: A partir das equacoes de Maxwell, obtenha a equacao fasorial-vetorial de Helmholtz dada

por

∇2EF = −ω2µ0ε0EF

PR 11.18: O campo eletrico de uma onda plana uniforme no ar tem uma amplitude de 8 V/m no sentido

do eixo x. Se a onda se propaga no sentido do eixo z, com um comprimento de onda λ = 0, 5m, encontre:

(a) a frequencia; (b) o perıodo; (c) o valor de k se o campo for expresso por E0 cos(ωt − kz); e, (d) a

amplitude do campo magnetico.

PR 11.19: Calcule a razao entre as densidades de corrente de conducao e de deslocamento para o campo

eletrico E = E0sen (ωt) V/m, quando σ = 5, 8× 107 S/m, ε = ε0, µ = µ0 e uma frequencia de 1,0MHz.

PR 11.20: A densidade de corrente de deslocamento em um certo material em que σ = 0, ε = 4ε0,

µ = µ0 e dada por 2 cos(ωt − 5z)ax µA/m2. Encontre o campo deslocamento eletrico e o campo

magnetico.

PR 11.21: Sendo o campo eletrico ~E = 200e(4x−kt) ay V/m no vacuo, use as equacoes de Maxwell para

encontrar o campo magnetico e a expressao da constante k na equacao do campo eletrico.

PR 11.22: Sendo o campo eletrico ~E = 200e(4x−kt) ay V/m no vacuo, use as equacoes de Maxwell para

encontrar o campo magnetico e a expressao da constante k na equacao do campo eletrico.

PR 11.23: Sabendo-se que a radiacao eletromagnetica do sol que atinge a superfıcie terrestre e da ordem

de 1350 W/m2, determine o valor da amplitude do campo eletrico Eo a amplitude da inducao magnetica

B0.

Page 83: Problemas de Eletromagnetismo.pdf

11. Ondas eletromagneticas 83

Resposta: Com o vetor de Poynting e a impedancia intrınseca do meio, podemos calcular a amplitude

do campo eletrico e Eo igual a 1, 01× 103 V/m. Podemos calcular tambem a amplitude para o campo

magnetico, que e igual a 2,6 A/m. Usando a relacao B = µ0H encontramos a amplitude da inducao

magnetica B0 = 3, 37× 10−6 T. ♦

PR 11.24: Uma linha de transmissao coaxial com superfıcies condutoras em rc = 1mm, rc = 5mm e

z = 0, tem o ar como dieletrico e um campo ~H = (0, 031/rc) cos 6× 107πt cos 0, 2πz~uφA/m.

1. Ache a fem gerada ao longo do caminho (10−3, 0o, 0) a (5 × 10−3, 0o, 0) a (5 × 10−3, 0o,−2) a

(10−3, 0o, 0).

2. Ao longo de que segmentos do caminho temos∮~E · d~= 0?

PR 11.25: Uma onda eletromagnetica tem a densidade de potencia de 2 W/m2, e se propaga num meio

com impedancia caracterıstica de 200 Ω. Qual e a amplitude dos campos eletrico ~E e magnetico ~H?

PR 11.26: Para a agua destilada temos permissividade relativa εr = 50 e condutividade σ = 20(Ω m)−1.

Calcular:

1. constante de propagacao;

2. constante de atenuacao;

3. constante de fase;

4. comprimento de onda; e,

5. impedancia intrınseca do meio para a situacao onde ω = 1011rad/s.

PR 11.27: Uma linha de transmissao coaxial tem raio interno a = 1mm, raio externo b = 4mm, e

um dieletrico homogeneo com εr = 2, 25, µ0 = 1 e σ = 0. O campo eletrico e dado por ~E =

(100/rc) cos(108 t− βz)~urc V/m.

1. Use as equacoes de Maxwell que envolvem o rotacional para determinar β.

2. Ache ~H.

3. Determine a densidade superficial de carga no condutor interno em funcao de φ, z e t.

4. Calcule a amplitude da corrente total de deslocamento no comprimento 0 ≤ z ≤ 1m.

PR 11.28: O campo magnetico proximo ao motor de um secador de cabelos varia senoidalmente com

uma frequencia de 60 Hz.

1. Mostre que a expressao simples ~B = cos 2π60tax T nao satisfaz as equacoes de Maxwell no ar.

Page 84: Problemas de Eletromagnetismo.pdf

11. Ondas eletromagneticas 84

2. Ache o valor de k sabendo que ~B = cos(2π60t− ky)ax satisfaz as equacoes de Maxwell.

PR 11.29: Um campo eletrico no vacuo e dado, em coordenadas esfericas por ~E = (0, 1/r)sen θsen (15×108t − 5r)~uθ V/m. Ache ~H considerando que todos os campos variam senoidalmente no tempo com a

mesma frequencia.

PR 11.30: O campo eletrico na origem e dado por 2ax − 10ay + 3az V/m em t = 0.

1. Se a origem pertence a uma superfıcie condutora perfeita, enquanto que para o material adjacente

a origem εr = 10, µr = 2 e σ = 0, ache o modulo da densidade superficial de carga na origem em

t = 0.

2. Se εr = 8, µr = 3 e σ = 0, para x ≤ 0, enquanto que εr = 3, µr = 8 e σ = 0, para x > 0, ache | ~E|em t = 0 no ponto (0+, 0, 0).

PR 11.31: Michel Faraday mostrou que um campo magnetico variante, como por exemplo ~B = B0ektaz,

da origem ao campo eletrico

E =−12kB0e

kt~uφ.

1. Mostre que estes campos nao satisfazem a outra equacao de Maxwell onde aparece no rotacional.

2. Sendo B0 = 1 T e k =1/10s, nos estamos estabelecendo um campo magnetico razoavelmente

grande em 1µs. Use a equacao que envolve ~∇× ~H para mostrar que a taxa com a qual Bz deveria

(mas nao o faz) variar com rc seria cerca de 5×10−6T por metro de espaco livre (vacuo) em t = 0.

PR 11.32: Sejam εr1 = 1, µr1 = 1 e σ1 = 0 na regiao 1 (z < 0), enquanto que εr2 = 5, µr2 = 20 e

σ2 = 0 na regiao 2 (z > 0). Sabendo que o campo eletrico na regiao 1 e ~E1 = [60 cos(15× 108t− 5z) +

20 cos(15× 108t+ 5z)]ax V/m, e que na regiao 2, ~E2 = A cos(15× 108t+ 5z)ax V/m, determine: (a) A;

(b) ~H1; (c) ~H2; (d) Mostre que ~H1 e ~H2 satisfazem as condicoes de contorno necessarias em z = 0.

PR 11.33: Sejam ε1 = 10−11F/m, µ1 = 4x10−6H/m e σ1 = 10−3(Ωm)−1 na regiao 1 (x < 0), enquanto

que ε2 = 2 ε1, µ2 = µ1/2 e σ1 = 4σ1, na regiao 2 (x > 0). Sabendo que ~E1 = (10ax + 20ay +

30az) cos 109t V/m no ponto P (0−, 0, 0), ache: (a) En1, Et1, Dn1 e Dt1 em P1; (b) Jn1 e Jt1 em P1;

(c) Et2, Dt2 e ~J2 em P2(0+, 00); (d) Use a equacao da continuidade como partida para mostrar que

Jn1 − Jn2 = ∂Dn2/∂t− ∂Dn1/∂t, e entao determine Dn2, Jn2 e En2.

PR 11.34: Temos superfıcies condutoras perfeitas localizadas em rc = 5mm, rc = 20mm, z = 0 e

z = 50cm (coordenadas cilındricas). A regiao envolvida e um dieletrico para o qual εr = 2, 25, µr = 1 e

σ = 0. Nesta regiao ~H = (2/rc) cos 2πz cos 4π108t~uphi A/m. Determine:

1. A densidade superficial de corrente em rc = 5mm, φ = 0, z = 5cm;

Page 85: Problemas de Eletromagnetismo.pdf

11. Ondas eletromagneticas 85

2. ~E;

3. A densidade superficial de carga em rc = 20mm, φ = π/2, e z = 25cm;

4. A densidade de corrente de deslocamento em rc = 10mm, φ = 0, 2π, e z = 25cm.

PR 11.35: Um condutor filamentar se estende desde z = −5 ate z = 5m sobre o eixo x no vacuo, e

conduz uma corrente I = 4t A no sentido +az. Ache e esboce ~A(t) em (0, 0, 10) para −0, 1 ≤ t ≤ 0, 1 µs.

PR 11.36: O campo eletrico no interior de uma linha de transmissao em forma de duas laminas condu-

toras muito longas e de pequena largura (2 mm) e afastamento (0,2 mm), pode ser considerado como

sendo dado por~E = −105 cos(109t− 4z)~uy V/m.

Ache ~A(x, y, z, t) se ~A(x, 0, z, t) = 0.

PR 11.37: Estudar a polarizacao do campo:

~E = 4/ 0ax + 3/ − 90ay

Solucao:

E(t) = 4 cosωt+ 3sen ωt

Ex = 4 cosωt

Ey = 3sen ωt

eliminando o tempo, vem:E2

x

16+E2

y

3= 1

elipse sobre a qual a extremidade do vetor vai ficar, ao girar. A onda e dita elipticamente polarizada.

Caso as amplitudes das duas componentes fossem iguais, seria circularmente polarizada. O sentido de

rotacao depende da defasagem ser mais ou menos noventa graus. ♦

PR 11.38: Discutir a polarizacao da onda de campo eletrico dada por:

~E = 5/ − 90e−jβzax + 5/ 0e−jβzay

Solucao:~E(t, z) = 5sen (ωt− βz)ax + 5 cos(ωt− βz)az

Ex(t, z) = 5sen (ωt− βz)

Ey(t, z) = 5 cos(ωt− βz)

eliminando (ωt− βz), temos o cırculo:

E2x + E2

y = 25

Para determinado z, a variavel (ωt− βz) cresce com o tempo portanto a rotacao.

Page 86: Problemas de Eletromagnetismo.pdf

11. Ondas eletromagneticas 86

PR 11.39: Estudar a polarizacao do campo:

~E = 4/ 0ax + 3/ 0ay

Solucao: Passando para o domınio do tempo:

E(t) = Exax + Eyay

Ex = X = 4 cosωt

Ey = Y = 3 cosωt

eliminando o tempo vem:

Y =34X

Reta sobre a qual o campo vai ficar variando: a onda e dita linearmente polarizada. ♦

PR 11.40: Se σ = 0, ε = ε0 e µ = µ0, determine se os campos

~E = 100sen 6× 107tsen zay V/m

~H = −0, 1328 cos 6× 107t cos zay A/m

satisfazem as equacoes de Maxwell.

PR 11.41: (Wentworth 6.1) Partindo das Equacoes de Maxwell para um meio simples, livre de cargas,

obtenha a equacao de Helmoltz para ~H.

Solucao:

∇× (∇× ~H) = ∇×(σ ~E + ε

∂ ~E

∂t

)= σ∇× ~E + ε

∂t∇× ~E = −µσ∂

~H

∂t− µε∂

2 ~H

∂t2

Usando a identidade vetorial ∇× (∇× ~H) = ∇ · ~H −∇2 ~H sabendo-se que ∇ · ~H = 0 obtem-se:

∇2 ~H = µσ∂ ~H

∂t+ µε

∂2 ~H

∂t2♦

PR 11.42: (Wentworth 6.5) Dado σ = 1, 0× 10−5 S/m, εr = 2, 0, µr = 50, 0 e f = 10 MHz, calcule γ,

α, β e η.

Solucao:

γ =√jωµrµ0(σ + jωεrε0) = 9, 4× 10−3 + j2, 11 1/m

α = 9, 4× 10−3 Np/m e β = 2, 11 rad/m.

η =

√jωµrµ0

σ + jωεrε0= 18806 257 Ω ♦

Page 87: Problemas de Eletromagnetismo.pdf

11. Ondas eletromagneticas 87

PR 11.43: (Wentworth 6.8) Suponha no espaco livre, ~H(x, t) = 100 cos(2π × 107t − βx + π/4)az

mA/m. Determine ~E(x, t).

Solucao:~Es = −ηap × ~Hs = −120πax × 0, 100e−jβxejπ/4 = 12πe−jβxejπ/4ay

~E(x, t) = 12π cos(

2π × 107t− 2π30x+

π

4

)ay V/m ♦

PR 11.44: (Wentworth 6.16) Em um meio com propriedades σ = 0, 00964 S/m, εr = 1, 0, µr = 100, 0

e f = 100 MHz, uma onda eletromagnetica se propaga na direcao +x com seu vetor campo na direcao

z. Determine a forma instantanea da intensidade do campo eletrico associada.

Solucao:~Hs = H0e−αxe−jβxaz

~Es = −ηap × ~Hs = −ηax ×H0e−αxe−jβxaz = ηH0e−αxe−jβxay

η =

√jωµ

σ + jωε= 2664ej30 Ω

γ =√jωµ(σ + jωε) = 14, 8 + j25, 7 1/m

~E(x, t) = 2, 66e−15x cos(200π × 106t− 26x+ 30

)ay V/m ♦

PR 11.45: (Wentworth 6.20) Calcular a profundidade pelicular a 1 GHz para os seguintes materiais:

(a) cobre; (b) prata; (c) ouro; e (d) nıquel.

Solucao:

σ (S/m) µr δ (mm)

Cobre 5,8E7 1 2,1

Prata 6,2E7 1 2,0

Ouro 4,1E7 1 2,5

Nıquel 1,5E7 600 0,17

PR 11.46: (Wentworth 6.23) Em um material condutor nao magnetico

~E(z, t) = 10e−200z cos(2π × 109t− 200z)ax V/m

Determine ~H(z, t).

Solucao: Como α = β o meio e um bom condutor. Com µr = 1 pode-se isolar a condutividade σ na

espessura de efeito pelicular δ = α:

σ =α2

πfµ0= 10, 13 S/m

η =√

σej45 Ω

Page 88: Problemas de Eletromagnetismo.pdf

11. Ondas eletromagneticas 88

~Hs =1ηap × 10e−αze−jβzax =

10η

e−αze−jβzay

~H(z, t) = 360e−200z cos(2π × 109t− 200z + 45

)ay mA/metro ♦

PR 11.47: (Wentworth 6.28) Assuma agua destilada (σ = 1, 0 × 10−45 S/m, εr = 81, µr = 1, 0)

preenchendo uma regiao z > 0. Na superfıcie, temos

~E(0, t) = 80 cos(2π × 109t)ax V/m

Determine: (a) ~E(z, t), (b) ~H(z, t), (c) Pave e (d) a potencia passando por uma superfıcie de 10 m2

localizada em z = 1, 0 m.

Solucao: Tangente de perdas: σ/ω/ε << 1 (dieletrico de baixa perda).

α =σ

2

õ

ε= 0, 0021 Np/m

β = ω√µε = 18, 88 rad/m

η =√µ/ε = 41, 9 Ω

~E(z, t) = 8e−0,0021z cos(2π × 108t− 18, 8z

)ax V/metro

~Hs =1ηap × ~Es = 191e−0,0021ze−j18,8zay mA/metro

~H(z, t) = 191e−0,0021z cos(2π × 108t− 18, 8z

)ay mA/metro

~Pavg =E2

x0

ηe−αzaz → 0, 761 W/m2

P (10m2) = 7, 6 W ♦

PR 11.48: (Wentworth 6.33) Dado ~H(z, t) = 2 cos(ωt − βz)ax + 6 cos(ωt − βz − 120)ay A/m. De-

termine a polarizacao e o sentido de rotacao.

Solucao:

~Es = −ηap × ~Hs = −η0az ×(2e−jβzax + 6e−jβze−j120 ay

)= −2η0e−jβzay + 6η0e−jβze−j120 ax

~E(z, t) = η06 cos (ωt− βz − 120) ax + η02 cos (ωt− βz − 180) ay V/m

Esta e uma polarizacao elıptica, em sentido horario. ♦

PR 11.49: Uma onda propagante e descrita por y = 10sen (βz − ωt). Esbocar a onda em t = 0 e em

t = 1, quando ela avancou λ/8 considerando sua velocidade com 3 × 108m/s e a frequencia angular

ω = 106rad/s. Repetir para ω = 2× 106rad/s.

PR 11.50: No espaco livre~E(z, t) = 103 cos(ωt− βz)ay V/m.

Obter H(z, t).

Page 89: Problemas de Eletromagnetismo.pdf

11. Ondas eletromagneticas 89

PR 11.51: No espaco livre E = 50 cos(ωt−βz)V/m. Calcular a potencia media que atravessa uma area

circular de raio 2,5 m pertencente a um plano Z constante.

Page 90: Problemas de Eletromagnetismo.pdf

12. LINHAS DE TRANSMISSAO

PR 12.1: (Halliday, pg.1105) Aplica-se uma diferenca de potencial dada por V0 = Vmsen ωt entre os

terminais de uma longa linha de transmissao, suposta sem resistencia. A frequencia e igual a 3 × 109

Hertz. Escrever a expressao para V (t) num ponto P , distante 1,5 comprimentos de onda.

Resposta: A equacao geral de uma onda progressiva na direcao x pode ser escrita como:

V = Vmsen (ωt− βx)

onde β = 2π/λ e a constante de fase ou numero de onda. Assim,

V = Vmsen [ωt− 2πλ

(1, 5λ)] = −Vmsen ωt

PR 12.2: (Ulaby Ex 8.1) Calcular os parametros de uma LT a dois fios, isolada com ar com distancia

de 2 cm, tendo raio de 1 mm em cada condutor. Os fios podem ser considerados condutores ideais com

condutividade infinita.

Solucao:

Rs = R′ = 0 ← σc = 0

L′ =µ0

πln

( d

2a

)+

√(d

2a

)2

− 1

= 1, 2 µH/m

G′ = 0 porque σ = 0.

C ′ =πε0

ln

[(d2a

)+√(

d2a

)2− 1

] = 9, 29 pF/m ♦

PR 12.3: (Ulaby Ex 8.2) Calcular os parametros de uma LT em 1 MHz para um condutor coaxial

rıgido, isolado com ar, tendo diametro interno de 0,6 cm e externo de 1,2 cm. Os condutores usados

sao de cobre.

Solucao:

Rs =√πfµc/σc = [π × 106 × 4π10−7/(5, 8× 107)]1/2 = 2, 6× 10−4 Ω

R′ =Rs

(1a

+1b

)= 2, 08× 10−2 Ω/m

L′ =µ0

2πln(b

a

)= 0, 14 µH/m

G′ = 0 porque σ = 0.

C ′ =2πε

ln(b/a)= 80, 3 pF/m ♦

Page 91: Problemas de Eletromagnetismo.pdf

12. Linhas de Transmissao 91

PR 12.4: (Ulaby Ex 8.3) Comprovar que V (z) = V +0 e−γz + V −0 e+γz e uma solucao da equacao da

onda.

Solucao:d2Vs(z)dz2

− γ2Vs(z) ∝ 0

d2

dz2

(V +

0 e−γz + V −0 e+γz)− γ2

(V +

0 e−γz + V −0 e+γz)∝ 0

γ2V +0 e−γz + γ2V −0 e+γz − γ2V +

0 e−γz − γ2V −0 e+γz = 0 ♦

PR 12.5: (Ulaby Ex 8.4) Uma LT com dois fios paralelos, isolada com ar, tem os seguintes parametros:

R′ = 0, 404 mΩ/m, L′ = 2 µH/m, G′ = 0, e C ′ = 5, 56 PF/m. Determinar, para a frequencia de operacao

de 5 kHz, os seguintes valores: (a) constante de atenuacao α; (b) constante de fase β; (c) a velocidade

de fase up; e, (d) a impedancia caracterıstica Z0.

γs =√

(R′ + jωL′) (G′ + jωC ′) = 3, 37× 10−7 + j1, 05× 10−4 1/m

α = 3, 37× 10−7 Np/m

β = 1, 05× 10−4 rad/m

up =ω

β= 3× 108 m/s

Z0 =R′ + jωL′

α+ jβ= (600− j2) Ω ♦

PR 12.6: (Ulaby Ex 8.5) Uma LT sem perdas, tem λ = 20, 7 cm na frequencia de 1 GHz. Qual e o

valor εr do material isolante?

Solucao:

λ =λ0√εr

εr =(λ0

λ

)2

=(c

)2

= 2, 1 ♦

PR 12.7: (Ulaby Ex 8.7) Uma LT sem perdas de 50 Ω e terminada com uma impedancia ZL =

(30− j200) Ω. Calcular o coeficiente de reflexao de tensao na carga.

Solucao:

Γ =ZL − Z0

ZL + Z0=

30− j200− 5030− j200 + 50

= 0, 936 − 27, 5 ♦

PR 12.8: (Ulaby Ex 8.9) Se Γ = 0, 5 6 −60 e λ = 24 cm, encontre a localizacao do maximo e mınimo,

da onda estacionaria, mais proximos da carga.

Solucao:

lMax =θΓλ

4π+λ

2= 10 cm

lMin = lMax −λ

4= 4 cm ♦

Page 92: Problemas de Eletromagnetismo.pdf

12. Linhas de Transmissao 92

PR 12.9: Supoe-se que uma LT seja infinitamente longa e que se aplica uma diferenca de potencial dada

por

Vs = Vmsen ωt

entre os terminais. A frequencia f = ω/2π e igual a 3× 109Hz. Escrever a expressao de v(t) num ponto

P , distante 1,5 comprimentos de onda.

Solucao: A equacao geral de uma onda progressiva na direcao do eixo dos x pode ser escrita como:

v = Vmsen (ωt− kx)

onde k = 2π/λ e denominado numero de onda. No ponto x = 1, 5λ, tem-se

vP = Vmsen (ωt− 2πλ

1, 5λ) = Vmsen (ωt− 3π) = −Vmsen (ωt)

Logo, vP sera sempre igual em modulo a tensao da fonte, mas de sinal oposto. ♦

PR 12.10: Calcular a tensao na carga para a LT, considerando que a onda eletromagnetica de tensao

se propaga com v = 3× 108m/s, para os seguintes casos de comprimento e frequencia:

Solucao:

a) ` = 5m e f = 60Hz:

Da teoria de circuitos:

vi(t) = 5 cos(2πft)

Entretanto, o atraso de propagacao introduz uma defasagem ∆φ:

∆φ = β` = (2π/λ)` = 2π × 10−6 rad = 0, 00036o

onde

λ =v

f=

3× 108

60= 5× 106 m

Assim,

vi(t) = 5 cos(2πft− 0, 00036o)

b) ` = 1000km e f = 60Hz:

∆φ = β` =2πλ` =

2π5

rad = 72o

vi(t) = 5 cos(2πft− 72o)

c) ` = 5m e f = 10MHz:

λ =v

f= 3× 10810× 106 = 30 m

∆φ = β` = (2π × 5/30) =π

3rad = 60o

vi(t) = 5 cos(2πft− 60o)

Conclusao: A teoria de circuitos, que e uma aproximacao da teoria mais geral de LT’s, apresenta bons

resultados somente quando ` << λ. ♦

Page 93: Problemas de Eletromagnetismo.pdf

12. Linhas de Transmissao 93

PR 12.11: (Wentworth 2.2) Calcule os parametros distribuıdos a 1MHz para um cabo com dois con-

dutores paralelos gemeos, formado por condutores de cobre AWG 26 com os centros separados, no ar,

por 1,0 cm.

Solucao: Usamos as formulas da Tabela 2.1 do livro, para dois condutores paralelos, para um cabo

AWG26 de cobre, que tem diametro 25,94 mil, e a = 7, 97 mils ou a = 202 µm. Assim:

R′ =1α

√fµ

σc= 0, 729 Ω/m

L′ =µ

πcosh− 1

(d

2a

)= 1, 16 µH/m

G′ =πσd

cosh− 1(d/2a)= 0

C ′ =πε

cosh− 1(d/2a)= 7, 12× 10−12 pF/m ♦

PR 12.12: (Wentworth 2.8) As especificacoes de um cabo coaxial RG-214 sao as seguintes:

(1) condutor interno de cobre com diametro de 2,21 mm;

(2) condutor externo com diametro interno de 7,24 mm;

(3) condutor externo com diametro externo de 9,14 mm; e,

(4) dieletrico Teflon (εr = 2, 10).

Calcule a impedancia caracterıstica e a velocidade de propagacao para esse cabo.

Solucao:

Z0 =60√εr

ln(b

a

)= 49, 1 Ω

up =c√εr

= 2, 07× 108 m/s ♦

PR 12.13: (Wentworth 2.10) Se uma potencia de 1,0 W e injetada em um cabo coaxial e 1,0 µW de

potencia e medido a 100 m do ponto de injecao de potencia na linha, qual e a atenuacao da linha em

dB/m?

Solucao:

A = −10 log

(1× 10−6 W

1 W

)= +60 dB ♦

PR 12.14: (Wentworth 2.16) Uma fonte com impedancia de 50 Ω alimenta uma LT de 50 Ω que possui

1/8 de um comprimento de onda, terminada com uma carga ZL = 50− j25 Ω. Calcule o coeficiente de

reflexao (complexo); a relacao de onda estacionaria (ROE); e a impedancia de entrada vista pela fonte.

Solucao:

ΓL =ZL − Z0

ZL + Z0=

50− j25− 5050− j25 + 50

= 0, 24e−j76 = 0, 246 76

V SWR = ROTE = ROE = s =1 + |ΓL|1− |ΓL|

= 1, 64

Page 94: Problemas de Eletromagnetismo.pdf

12. Linhas de Transmissao 94

β ` =2πλ

π

8=π

4← tan

4

)= 1

Zin = Z0ZL + jZ0 tan(β`)Z0 + jZL tan(β`)

= 5050− j25 + j50× 1

50 + j(50− j25)× 1= 30, 8− j3, 8 Ω ♦

PR 12.15: (Wentworth 2.19) A impedancia de entrada para uma LT sem perdas, de comprimento 30

cm e 100 Ω de impedancia, operando em 2 GHz, e Zin = 92, 3− j67, 5 Ω. A velocidade de propagacao

e 0,70c. Determine a impedancia da carga.

Solucao: Isolando-se ZL na equacao da impedancia de entrada de uma LT:

Zin = Z0ZL + jZ0 tan(β`)Z0 + jZL tan(β`)

⇒ ZL = Z0Zin − jZ0 tan(β`)Z0 − jZin tan(β`)

ZL = 50 + j0, 912 Ω = 50 + j2π (2× 109) L → L = 73 pH

Esta e uma indutancia muito baixa, de modo que ZL∼= 50 Ω. ♦

PR 12.16: (Wentworth 2.24) Suponha que uma LT da Fig.2.13 seja caracterizada pelos seguintes

parametros distribuıdos em 100 MHz: R = 5, 0 Ω/m; L = 0, 010 µH/m; G = 0, 010 S/m; e, C = 0, 020

nF/m. Se ZL = 50−j25 Ω, vs = 10 cosωt V; Zs = 50 Ω (impedancia de saıda da fonte); e o comprimento

da linha e 1,0 metro, determine a tensao para cada extremidade da linha de transmissao.

Solucao: Inicialmente, calcula-se os parametros da LT:

γ =√

(R′ + jωL′) (G′ + jωC ′) = 0, 2236 + j0, 2810 m−1

Z0 =

√R′ + jωL′

G′ + jωC ′= 22, 3607 Ω

ΓL = 0, 4479− j0, 1908

Zin = Z0ZL + jZ0 tan(β`)Z0 + jZL tan(β`)

= 27, 2079− j15, 413 Ω

Fazendo a divisao da tensao da fonte VSS entre as impedancias ZS e Zin, tem-se a tensao Vin na entrada

da LT:

Vin = VSSZin

Zin + ZSS= 3, 97e−j18,2 V . . . vin(t) = 4, 0 cos(ωt− 18, 2) V

Entretanto, a tensao na entrada da LT pode ser escrita em funcao dos seus parametros:

Vin = V +0

(eγ` + ΓLe−γ`

)= V +

0 (1, 504 + j0, 101) = V +0

(1, 507ej3,84

)Isolando-se a amplitude da onda progressiva para a direita V +

0 , encontra-se:

V +0 =

3, 97 6 − 18, 2

1, 5076 3, 84= 2, 636 − 22 = 2, 63e−j22 Volts

E, assim obtem-se facilmente a tensao VL nos terminais da carga:

VL = V +0 (1 + ΓL) = 3, 85e−j29,6 Volts . . . vL(t) = 3, 85 cos(ωt− 29, 6) V ♦

Page 95: Problemas de Eletromagnetismo.pdf

13. CARTA DE SMITH

A carta de Smith, inventada por Phillip H. Smith(1905-1987), e uma ajuda grafica ou nomograma pro-

jetado para engenheiros eletricistas e eletronicos especializados em radio frequencia(RF), para ajudar na

resolucao de problemas com linhas de transmissao e circuitos associados. O uso da Carta de Smith tem

crescido ao longo dos anos e ainda e amplamente utilizado hoje em dia, nao so como ajuda para resolucao

de um problema, mas como um demonstrador grafico do comportamento em uma ou mais frequencias

de um dispositivo em RF, ou ainda uma alternativa ao uso deinformacoes tabulares ou numericas.

A carta de Smith pode ser usada para representar muitos parametros, incluindo impedanciase ad-

mitancias,coeficientes de reflexao, parametros de espalhamento, e figura de cırculos de ruıdo, contornos

de ganho constante, regioes para a estabilidade incondicional, incluindo analise de vibracoes mecanicas.

O analisador de rede (HP8720A)(Fig. 13.1), e um dos instrumentos mais importantes em RF, porque

apresenta o grafico de Smith.

Usando a carta de Smith para analisar circuitos de elementos concentrados

Os circuitos concentrados tem o comprimento de onda (da frequencia de operacao) muito maior do

que as dimensoes dos proprios componentes. O grafico de Smith pode ser utilizado para analisar tais

circuitos, caso em que os movimentos de todo o grafico sao gerados pelas impedancias (normalizadas)

e admitancias dos componentes com uma frequencia de operacao. Neste caso, o comprimento de onda

de escala na circunferencia grafico de Smith nao e utilizado. O circuito da Fig. 8.1 sera analisado

utilizando a carta de Smith, com uma frequencia de funcionamento de 100 MHz. Neste frequencia o

comprimento de onda no espaco livre e de 3 m. As dimensoes dos componentes proprios estara na

ordem de milımetros de modo que o pressuposto de componentes concentrados sera valido. Apesar

de nao existir nenhuma linha de transmissao, como tal, uma impedancia do sistema deve ainda ser

definidas para permitir calculos de normalizacao e desnormalizacao e e uma boa opcao neste caso.

A analise comeca com o grafico de Smith Z olhando para R1, apenas com outros componentes

presentes. Comoe o mesmo que a impedancia do sistema, isto e representado por um ponto no centro do

grafico de Smith. A primeira transformacao e OP1 ao longo da linha de resistencia normalizada constante

neste caso, a adicao de uma reactancia normalizada de -j0,80, correspondendo a um condensador em

serie de 40 pF. Pontos com P de sufixo sao no plano Z e pontos com sufixo Q estao no planoY. Portanto

transformacoes P1 a Q1 e P3 a Q3 sao do grafico de Smith Z para a Smith Y e Q grafico transformacao

2 2 para Pe a partir do grafico de Smith Y para o grafico de Smith Z. A tabela seguinte mostra os passos

dados para trabalhar com os componentes restantes e transformacoes, retornando eventualmente, de

volta para o centro do grafico de Smith e uma correspondencia perfeita de 50 Ω.

Page 96: Problemas de Eletromagnetismo.pdf

13. Carta de Smith 96

Fig. 13.1: Network Analyser ou Analisador de Redes, tracando a Carta de Smith.

Fig. 13.2: Calculo da impedancia de entrada do circuito RLC, usando a Carta de Smith.

Page 97: Problemas de Eletromagnetismo.pdf

13. Carta de Smith 97

Tab. 13.1: Pontos da grade, e comparacao das solucoes analıtica e numerica.Transform. Y ou Z x ou y normal. Capac./ indut. Formula para resol. Resultado

O − P1 Z -j80 Capacitncia (Srie) −j80 = −j/ωC1Z0 40 pF

Q1 −Q2 Y -j1,49 Indutancia (Paralelo) −j1, 49 = −j/ωL1Y0 53 nH

P2 − P3 Z -j0,23 Capacitancia (Serie) −j0, 23 = −j/ωC2Z0 138 pF

Q3−O Y +j1,14 Capacitancia (Shunt) +j1, 14 = jωC3/Y0 36 pF

PR 13.1: Determinar a impedancia de entrada da LT, sabendo que e terminada por uma impedancia

de 15 + j25Ω. Um trecho de 27, 4cm de cabo coaxial de 50 ohms de impedancia caracterıstica tem

a frequencia de 146 MHz, uma velocidade de transmissao v = 0, 67c = 20 cm/nano segundos, e um

comprimento de onda λ = 20× 1000/146 cm = 1, 37 metros.

Solucao: Procedimento: a impedancia de carga normalizada, calculada como:

ZR = 0, 3 + j0, 5 =1550

+ j2550

p.u.

Como o comprimento da LT e 27, 4/137 = 0, 12λ, deve-se ‘caminhar’ no sentido horario na carga de

Smith, significando da carga para o gerador, ate a nova impedancia. Observarmos na carta que ela

mede 1, 6 + j1, 7 p.u. Entao, a impedancia de entrada, vista pela fontes, vale

ZS = (1, 6 + j1, 7)× 50 = 80 + j85 Ω

Observa-se o efeito dramatico de alguns centımetros de cabo. A carta de Smith tambem fornece o

coeficiente de reflexao Γ. Seu modulo corresponde ao raio da carta, e seu angulo ao angulo medido a

partir do ponto de maxima impedancia (direita). ♦

PR 13.2: Uma L.T. tem constantes iguais a R = 10×10−3Ω/m, G = 1, 5×10−6S/m, L = 1, 2×10−6H/m,

e C = 1, 5 × 10−9F/m. Qual e a impedancia caracterıstica, a atenuacao em dB/km, a defasagem em

rad/km, e a velocidade da onda, para a frequencia f = 10 000Hz?

PR 13.3: Um cabo coaxial com raio interno igual a 0,5mm e raio externo de 2,8mm, tem µr = 1 e

εr = 3, 1, e atua com uma frequencia de 400MHz e uma impedancia de carga igual a 100Ω. Qual e o

comprimento de onda e o coeficiente de reflexao desta linha?

PR 13.4: Uma linha de 50Ω sem perdas tem comprimento de 1, 5λ, uma impedancia de carga ZL = 60Ω

e uma tensao na carga de 206 40oV. Calcular: (a) a potencia media fornecida a carga; (b) o modulo da

tensao mınima na linha; e, (c) o modulo da corrente maxima na linha.

PR 13.5: Uma L.T. casada de 50Ω e alimentada por um transmissor que e equivalente a uma fonte de

amplitude 300V com resistencia interna de 50Ω. O comprimento da linha e 5m e a frequencia e de

30MHz. Obter as ondas de tensao e corrente, usando o cosseno como referencia.

Page 98: Problemas de Eletromagnetismo.pdf

13. Carta de Smith 98

PR 13.6: Uma L.T. corretamente terminada tem um coeficiente de atenuacao de 2dB/km e um coefi-

ciente de deslocamento de fase de 0,2rad/km. Qual a tensao no ponto de 10km, ao longo da linha, se

houver uma entrada de 1Vrms?

PR 13.7: Uma linha de transmissao casada, de comprimento 10 km, possui: R = 50Ω/km, G =

1, 0µS/km, L = 600µH/km, e C = 40nF/km. Se a potencia de entrada da L.T. for 1W na frequencia

de 1kHz, calcular: (a) a impedancia caracterıstica da linha; (b) o coeficiente de propagacao; (c) o coe-

ficiente de atenuacao; (d) o coeficiente de defasagem; (e) os modulos da tensao e da corrente nos lados

da fonte e de recepcao da linha; e, (f) a potencia recebida no final da linha.

PR 13.8: Duas L.T. sem perdas, perfeitamente terminadas, tem resistencias caracterısticas iguais a

200Ω e 400Ω. Qual deve ser a resistencia caracterıstica de uma secao de um quarto de comprimento de

onda para conecta-las a um casamento perfeito?

PR 13.9: Encontre a impedancia de entrada de uma linha cuja impedancia caracterıstica vale 500Ω,

tendo comprimento de 60 cm, e operando em um comprimento de onda de 2m. Para isso use a

impedancia de terminacao ZL = 25 + j50Ω.

PR 13.10: (Wentworth 2.25) Posicione em uma Carta de Smith as seguintes impedancias de carga,

terminando uma LT de 50 Ω: (a) ZL = 200 Ω; (b) ZL = j25 Ω; (c) ZL = 50+j50 Ω; e, (d) ZL = 25−j200

Ω.

Solucao: ♦

PR 13.11: (Wentworth 2.26) Repita o problema Usando a Carta de Smith: Uma fonte com impedancia

de 50 Ω alimenta uma LT de 50 Ω que possui 1/8 de um comprimento de onda, terminada com uma

carga ZL = 50 − j25 Ω. Calcule o coeficiente de reflexao (complexo); a relacao de onda estacionaria

(ROE); e a impedancia de entrada vista pela fonte.

Solucao: ♦

PR 13.12: (Wentworth 2.27) Uma LT sem perdas com Z0 = 75 Ω e 0, 690λ de comprimento e termi-

nada em uma carga ZL = 15 + j67 Ω. Utilize a carta de Smith para determinar: (a) ΓL; (b) ROTE ou

ROE; (c) Zin; e, (d) a distancia entre a extremidade de entrada da LT e o primeiro maximo de tensao

a partir da extremidade de entrada da linha.

Solucao: ♦

PR 13.13: (Wentworth 2.30) Em uma LT sem perdas de Z0 = 50 Ω, o ROTE e medido como sendo

3,4. Um maximo de tensao se localiza 0, 079λ a partir da extremidade da carga. Determine a carga.

Solucao: ♦

Page 99: Problemas de Eletromagnetismo.pdf

13. Carta de Smith 99

PR 13.14: (Wentworth 2.35) Uma rede de casamento de impedancia, utilizando um elemento reativo

em serie com um comprimento d de uma LT, e utilizado para casar uma carga de 35− j50 Ω em uma

LT de 100 Ω a 1 GHz. Determine o comprimento completo da linha d e o valor do elemento reativo se:

(a) um capacitor serie for utilizado; e, (b) um indutor serie for utilizado.

Solucao: ♦

PR 13.15: (Wentworth 2.39) 2.37 Deseja-se casar uma carga de 170 Ω em uma LT de 50 Ω. Determine:

(a) A impedancia caracterıstica exigida para um transformador de quarto de onda; e, (b) Quais os

comprimentos de linha e de stub exigidos para uma rede de casamento com stub em paralelo?

Solucao: ♦

PR 13.16: (Wentworth 2.38) Uma impedancia de carga ZL = 200 + j160 Ω e casada a uma linha

de 100 Ω utilizando um sintonizador de Stub paralelo em curto. Determine a solucao que minimiza o

comprimento do Stub em curto.

Solucao: ♦

PR 13.17: (Wentworth 2.39) Repita o problema 2.38 do Wentworth para um sintonizador de Stub

paralelo em aberto.

Solucao: ♦

Page 100: Problemas de Eletromagnetismo.pdf

14. EQUACOES DE MAXWELL

PR 14.1: Sendo E = 200e4x−ktay V/m no vacuo, use as equacoes de Maxwell para encontrar k e ~H,

sabendo que todos os campos variam com e−kt.

PR 14.2: Um problema unidimensional de valores de contorno - Dois pontos extremos de um intervalo,

x = 0 e x = L, possuem valores conhecidos V (0) e V (L), respectivamente. A equacao diferencial que

rege a distribuicao da funcao V ao longo de x e

d2V

dx2= AV +B

Dividir o intervalo 0 ≤ x ≤ L em n+1 intervalos igualmente espacados de ∆x = xj+1−xj , j = 0, n+1,

e resolver o problema usando diferencas finitas.

Solucao: Substituindo a derivada d2V/dx2 por diferencas finitas centrais, a equacao diferencial para

o ponto j pode ser representada por

Vj+1 − 2Vj + Vj−1

(∆x)2= AVj +B

Entao nos temos n equacoes lineares simultaneas, para os V1, V2, . . .Vn valores da funcao V em cada

ponto j = 1, n. Apos multiplicar cada equacao por (∆x)2, este conjunto de equacoes pode ser escrito

como

V2 − 2V1 + V (0) = (AV1 +B) (∆x)2

V3 − 2V2 + V (1) = (AV2 +B) (∆x)2

. . . . . .

Vn − 2Vn−1 + Vn−2 = (AVn−1 +B) (∆x)2

V (L)− 2Vn + Vn−1 = (AVn +B) (∆x)2

(14.1)

Se V (0) = V (L) = 0, α = −2 − A(∆x)2 e β = B(∆x)2, o conjunto de equacoes resulta no sistema de

equacoes:

Page 101: Problemas de Eletromagnetismo.pdf

14. Equacoes de Maxwell 101

α 1

1 α 1

1 α 1

. . . . . . . . .

. . . . . . . . .

1 α 1

1 α

V1

V2

V3

. . .

. . .

Vn−1

Vn

=

β

β

β

. . .

. . .

β

β

(14.2)

A matriz dos coeficientes e tridiagonal, e pode ser resolvida por metodos numericos rapidos para

ordem muito grande (na faixa de n = 1000). Enquanto que uma solucao analıtica e viavel apenas para

problemas muito simples, as tecnicas numericas sao fundamentais para considerar nao linearidades nas

equacoes, tais como as advindas das propriedades dos materiais. ♦

PR 14.3: Considere o desenho em 2D, onde tem-se duas superfıcies equipotenciais S1 e S2, e um fluxo

que vai de S1 para S2. Desenhar a forma das linhas equipotenciais e das linhas de fluxo.

PR 14.4: Um bloco de material semi-condutor foi inserido entre duas placas metalicas, como mostra-se

na figura. Fazer um esboco das linhas equipotenciais e das linhas de fluxo da corrente no semicondutor.

PR 14.5: Dois materiais condutores foram inseridos entre duas placas metalicas, como mostra-se na

figura. Fazer um esboco das linhas equipotenciais e das linhas de fluxo da corrente no semicondutor.

PR 14.6: Metodo de Euler. Escolher um intervalo de tempo ∆t = tj+1 − tj , fazer a aproximacao

dy

dt= df(yj , tj) =

yj+1 − yj

tj+1 − tj

e resolver as equacoes diferenciais anteriores passo a passo no tempo, para um determinado intervalo

de tempo, e comparar os valores numericos e analıticos. Solucao: Apesar desta tecnica numerica ser

muito simples, voce necessita de valores de ∆t muito pequenos para obter uma precisao razoavel. Por

isto, esta tecnica nao e muito usada na pratica, quando se necessita de exatidao. ♦

Page 102: Problemas de Eletromagnetismo.pdf

BIBLIOGRAFIA

[1] WENTWORTH, Stuart M., Eletromagnetismo Aplicado, Bookman, Porto Alegre, 2009.

[2] ULABY, Fawwaz. T., Eletromagnetismo para Engenheiros, Bookmann, Porto Alegre, 2007.

[3] HAYT, W. H., Eletromagnetismo, LTC Editora, 1983.

[4] KRAUS, J.D., CARVER,K.R.. Eletromagnetismo. Guanabara Dois. 1953.

[5] BASTOS, J. P. A., Eletromagnetismo para Engenharia: Estatica e Quase-estatica, Editora da

UFSC, 1989.